Zuku Sample Questions Flashcards

1
Q

A 22-year-old Thoroughbred mare is presented for bilateral green nasal discharge, retching, coughing, and inappetence noted after feeding this evening.
The owner recently switched the mare to a pelleted diet because of the mare?s poor dentition.
Physical exam reveals bilateral green nasal discharge. The mucous membranes are pink and moist with a capillary refill time of 1.5 seconds.
A slight swelling is notable in the proximal-left jugular groove just caudal to the vertical ramus of the mandible.

Based on the most likely diagnosis, what initial therapy would be recommended?
Value
Normal
100.6ºF (38.1°C) 99.1?100.8°F (37.3?38.2°C)
HR=40 bpm
28-40 bpm
R=12 brpm
10-14 brpm

A - Ultrasound thorax, perform transtracheal wash for cytology and culture, begin broad-spectrum antimicrobials
B - Administer antimicrobials, refer horse immediately for endoscopy and lavage under general anesthesia
C - Cautious guttural pouch endoscopy with lavage for Streptococcus equi spp. equi PCR, quarantine horse until results are available
D - Pass nasogastric tube;_give atropine; lavage the esophagus until the obstruction clears
E - Withhold food and water; give sedatives and oxytocin; recheck in 2 hours

A

Answer: Cautious guttural pouch endoscopy with lavage for Streptococcus equi spp. equi PCR, quarantine horse until results are available

The most appropriate initial therapy for this systemically stable horse with an acute esophageal obstruction (also incorrectly called “choke”) is to withhold food and water, give sedatives and oxytocin (and n-buty|scopolammonium bromide) to relax the esophagus, and recheck in a couple hours.
This is often successful in uncomplicated cases.
If obstruction has not resolved within 2-4 hours, sedate to lower the head, pass a nasogastric tube to the level of the obstruction, and GENTLY lavage the esophagus with water.
This is typically successful.
In cases that do not resolve with this Tx, refer for ultrasound, esophageal endoscopy, and/or general anesthesia with esophageal lavage at a referral center.
Click here to see an endoscopic image of an esophagus after resolution of a 12-hour obstruction.
Obstructions lasting more than several hours require IV fluids and electrolytes, anti-inflammatories, and antimicrobials (for secondary aspiration pneumonia).
Complications include aspiration pneumonia, repeat obstruction, and esophageal stricture.

How well did you know this?
1
Not at all
2
3
4
5
Perfectly
2
Q

Several ornamental freshwater fish (Betta splendens) have recently developed a velvety, rust-colored appearance, especially around the eyes and gills (see below).
Which of the following is an appropriate treatment in pet fish for the presumptive diagnosis?
A - Oxytetracycline
B - None; this condition is self-limiting.
C - Potassium permanganate
D - Chloroquine
E - Tricaine methanesulfonate (MS222).

A

Answer: Chloroquine

Treat these fish with suspected “velvet disease” with chloroquine. Velvet disease a parasitic dinoflagellate infection by Piscinoodinium spp. in freshwater fish and Amyloodinium spp. in saltwater species.
The Tx in ornamental fish is typically chloroquine or copper sulfate (provided alkalinity is > 50 mg/L), given as a bath or in a quarantine tank. The Tx in fish raised for food in the U.S. is repeated Txs of copper sulfate (assuming alkalinity is > 50 mg/L) (similar to Ich, Ichthyophthirius multifliis) or hypersalinity.
“Velvet disease” is one of the biggest health concerns in captive marine fish, especially clownfish, and can look like white spots. The nonmotile trophont (feeding) stage of this parasite attaches to the skin and gills.
After feeding, the trophont gives rise to the reproductive stage (tomont) which encysts and forms hundreds (up to 256!) flagellated, free-swimming stages (dinospores) that go on to initiate new infections. This exponential reproduction makes the prognosis guarded if Dz is not caught and Tx early.
Potassium permanganate is one Tx for oomycete infections in fish (primarily fresh-water), along with hypersalinity and formalin. Clinical presentation includes gray-white, cotton-like growths on the skin, eyes, gills, or fins.
Tricaine methanesulfonate (MS222, Syncaine®, tricaine mesylate) is the only anesthetic licensed in the United States and Canada for food fish intended for human consumption.

Merck Veterinary Manual:

Velvet disease, caused by Piscinoodinium in freshwater and Amyloodinium in marine fish, is a parasitic infection leading to mortality and lethargy. It presents as small golden spots on the skin and gills, which are often not visible without a microscope. Diagnosis is made by wet mount examination. Treatments include copper sulfate, chloroquine, and freshwater dips for marine fish. Control of the disease is challenging, requiring repeated treatments and careful management of the fish environment to break the parasite’s life cycle. Effective treatment and prevention are critical for managing this disease in aquaria and fish farms.

[Link] (https://www.merckvetmanual.com/exotic-and-laboratory-animals/aquarium-fish/parasitic-diseases-of-fish?autoredirectid=18693)

How well did you know this?
1
Not at all
2
3
4
5
Perfectly
3
Q

A five-year-old Siberian Husky is presented with erythema, depigmentation, scaling, and crusting of the nasal planum. The dog seems otherwise healthy.
Which one of the following choices is the most likely diagnosis?
Image courtesy, Caroldermoid.
A - Discoid lupus
B - Dermatomyositis
C - Calcinosis cutis
D - Food allergy
E - Demodecosis

A

Answer: Discoid Lupus

This is a classic example of discoid lupus. There is a breed disposition including German Shepherds,
Siberian Huskies, Brittanys, Collies, and Australian herding breeds.
Exposure to ultraviolet light is another risk factor.
Dermatomyositis is a hereditary skin disease of Collies and Shetland Sheepdogs. Lesions are usually over bony prominences, tip of tail, and pinnae, rather than the nasal planum.

Discoid Lupus Erythematosus (DLE) in Dogs: NAVLE Preparation

Pathophysiology:

•	DLE is an autoimmune disease affecting the skin, especially the nasal planum. The immune system targets skin cells, causing inflammation and tissue damage.

Clinical Signs:

•	Depigmentation, erythema, scaling, crusting, and ulceration of the nasal planum.
•	Lesions may extend to the bridge of the nose and periocular areas.
•	Exacerbated by sunlight.

Diagnostic Tests:

•	Skin biopsy for histopathology.
•	Immunofluorescence to detect immune complexes.
•	Rule out other causes like infections or neoplasia.

Affected Dogs:

•	Common in breeds like Collies, Shetland Sheepdogs, German Shepherds, and Siberian Huskies.
•	Typically affects middle-aged dogs.

Complications:

•	Secondary bacterial infections.
•	Permanent scarring and depigmentation.

Possible Treatments:

•	Topical corticosteroids and tacrolimus.
•	Systemic immunosuppressants (e.g., prednisone).
•	Avoidance of sunlight and use of sunscreens.

Management:

•	Regular follow-ups to monitor disease progression and treatment efficacy.
How well did you know this?
1
Not at all
2
3
4
5
Perfectly
4
Q

A six-year-old miniature Dachshund is presented with progressively worsening carpal and tarsal laxity.
Which one of the following choices is the most likely diagnosis?

A - Infectious synovitis
B - Multiple myeloma
C - Metacarpal fracture
D - Rheumatoid arthritis
E - Systemic lupus erythematosus

A

Answer: Rheumatoid arthritis

This dog has rheumatoid arthritis (erosive polyarthritis). There is mild to moderate destruction of the cortex of the distal radius.
There is decreased mineral opacity in the distal carpal bones and large lucent areas are visible within them. The cortices of the distal carpal bones are not well visualized.
The cortical margins of the carpal bones are irregular. The proximal and distal intercarpal joints are collapsed.
There is increased intracapsular soft tissue opacity in the carpal joint. There is a moderate soft tissue swelling at the level of the carpal joint.
There is cranial curvature of the radius.
Click here to see normal canine carpal radiographs.
Radiographic interpretation and images courtesy, Dr A. Zwingenberger and Veterinary Radiology.
Normal radiograph links courtesy, Imaging Anatomy Univ. of Illinois Vet Med.

[https://www.merckvetmanual.com/musculoskeletal-system/arthropathies-and-related-disorders-in-small-animals/immune-mediated-arthritis-in-dogs-and-cats?autoredirectid=14293]

How well did you know this?
1
Not at all
2
3
4
5
Perfectly
5
Q

A nine-year-old female spayed German Shepherd is presented for non-productive retching and collapse.
Physical examination reveals severe abdominal pain, tachycardia, pale mucous membranes, poor pulse quality, and tachypnea.
The dog is stabilized with intravenous fluid therapy, and an abdominal radiograph is shown below. The second image reveals the findings on exploratory laparotomy.
What is the most likely diagnosis?

A - Gastrointestinal ulceration
B - Mesenteric volvulus
C - Linear foreign body.
D - Intussusception
E - Intestinal lymphosarcoma

A

Answer: Mesenteric volvulus

This is an example of mesenteric volvulus. Note the marked generalized gas distention of the bowel.
Obstruction of blood supply to the intestine causes ileus, and the discoloration of the serosa seen in the second image.
Mesenteric volvulus is uncommon and is nearly always fatal due to irreversible severe diffuse bowel wall ischemia. German shepherds, English pointers and other large breed dogs are predisposed.
The presentation can resemble gastric dilatation and volvulus (i.e., acute abdominal pain and distention, unproductive retching and shock).
A linear foreign body causes intestinal plication visible of radiographs and at exploratory laparotomy.
Image courtesy, Dr Kalumet.

[https://www.merckvetmanual.com/digestive-system/diseases-of-the-stomach-and-intestines-in-small-animals/gastric-dilation-and-volvulus-in-small-animals]

How well did you know this?
1
Not at all
2
3
4
5
Perfectly
6
Q

A young adult terrier cross dog is presented for inability to gain weight, vomiting, and poor appetite.
The owner had adopted the dog from a shelter several days earlier after he had been found in an emaciated state.
On physical examination, he had poor body condition (3/9) and abdominal palpation revealed a tense abdomen with gassy enlarged loops of bowel and a firm linear structure.
Abdominal radiographs are shown below.
Which one of the following choices is the most likely diagnosis?

A - Foreign body
B - Megacolon
C - Normal abdomen for age
D - Retroperitoneal mass
E - Microhepatica

A

Answer: Foreign body.

This dog has a chronic linear foreign body with multiple intussusceptions. The foreign material in the stomach and duodenum, gravel sign (mineral opacities), and plication suggest a linear foreign body and chronic partial obstruction.
Distended bowel loops with a convex soft tissue appearance could indicate intussusception, however, ultrasound is a better imaging modality to confirm this. Chronic partially obstructing linear foreign bodies can cause intussusception as the peristaltic motion of the intestines attempts to move the foreign material forward.
Decreased serosal detail is likely in part due to poor body condition although the presence of peritoneal effusion is also likely.
Click here to see normal canine abdominal radiographs.
Click here to see a postmortem image showing multiple intussusceptions and plication pattern with markedly distended bowel loops.
Radiographic interpretation and images courtesy, Dr A. Zwingenberger and Veterinary Radiology.
Normal radiograph links courtesy, Imaging Anatomy Univ. of Illinois Vet Med.

https://www.merckvetmanual.com/digestive-system/diseases-of-the-stomach-and-intestines-in-small-animals/gastrointestinal-obstruction-in-small-animals

How well did you know this?
1
Not at all
2
3
4
5
Perfectly
7
Q

A 12-year-old miniature horse stallion is presented for inappetance and strange behavior. He typically lives in a pasture with a herd of seven other horses, including mares and foals, supplemented with round bales of grass hay.
Five days prior to presentation he had his feet trimmed and was subsequently very sore so was kept indoors, away from the herd, on 2.2 mg/kg of phenylbutazone once daily.
On examination he is head-pressing and obtunded.
During the exam he goes down and has a seizure, and is then euthanized.
At necropsy the liver looks like the image below.
What histopathological finding is most likely to be described from this liver?

A - Fluke migration
B - Portal hepatitis associated with a toxin
C - Hepatic lipidosis
D - Megalocytosis, bridging fibrosis from pyrrolizidine alkaloids
E - Oxidative injury due to phenylbutazone

A

Answer: Hepatic lipidosis.

The appearance of this liver (yellow, swollen, friable with rounded edges) plus the history and presentation are consistent with hepatic lipidosis. Any horse with decreased feed intake is at risk for developing hyperlipemia or hyperlipidemia and hepatic lipidosis but ponies and miniature horses, especially those that are off feed, pregnant, or lactating, are at much higher risk.
When negative energy balance leads to fat mobilization, the liver must process the fat by gluconeogenesis into glucose. If the liver is overwhelmed by mobilized free fatty acids, triglycerides are deposited within the hepatocytes and accumulate in the plasma. This leads to hepatic dysfunction.

https://www.merckvetmanual.com/digestive-system/hepatic-disease-in-large-animals/hyperlipemia-and-hepatic-lipidosis-in-large-animals

How well did you know this?
1
Not at all
2
3
4
5
Perfectly
8
Q

A 12-year-old male neutered West Highland White terrier is presented after he disappeared from home for six hours and returned with a left hind limb lameness.
A pelvic radiograph taken on presentation is shown below.
What is the most appropriate initial treatment?

Nottingliam Vet School
Image courtesy, Nottingham Vet School
A - Tibial tubercle transposition and Robert-Jones bandage
B - Triple pelvic osteotomy
C - Non-steroidal anti-inflammatory medication and rest
D - Femoral head and neck excision
E - Closed reduction and Ehmer sling

A

Answer: Closed reduction and Elmer sling

Conservatively treat coxofemoral (hip) luxation with closed reduction and an Ehmer sling. Typically presents as a non-weight-bearing lameness, usually after blunt force trauma, (e.g., being hit by a car). Craniodorsal luxation of the femoral head is most common.
Tx: Closed or open reduction. Closed reduction and Ehmer sling if injury <48h old, patient is stable for anesthesia, and hip structure is normal. 50% success rate.
Open reduction if closed reduction is unstable or has failed.
Carefully monitor post-reduction for irritation/wounds and ischemia caused by Ehmer sling.
Here is some useful information from the University of Illinois about closed reduction of hip luxations. And here is more helpful information from the American College of Veterinary Surgeons.

https://www.merckvetmanual.com/musculoskeletal-system/arthropathies-and-related-disorders-in-small-animals/joint-trauma-in-dogs-and-cats

https://vetmed.illinois.edu/2021/04/13/coxofemoral-luxation-tips-for-closed-reductions/

https://www.acvs.org/small-animal/hip-luxation/

How well did you know this?
1
Not at all
2
3
4
5
Perfectly
9
Q

Which of the following correctly describes the cells on this canine blood smear?

A - Polycytosis, macrochromasia, one Heinz body
B - Heterochromic, microcytic red blood cells, Babesia canis parasitism
C - Macrocytic, hypochromic red blood cells, plasma cell,. Anaplasma marginale
D - Polychromasia, anisocytosis, spherocytosis
E - Anisochromasia, red blood cell ghosts, rubricytes

A

Answer: Polychromasia, anisocytosis, spherocytosis

(Zuku) [https://zukureview.com/zuku-qod/navle/1880?chosen=ErFNNRTN8TNCWXdORc3x1EoGJcCyV9RmsqiLVgvwR8NKY1IUaWPZNAp4Wzx7H1Nw&utm_source=Zukureview+Subscribers&utm_campaign=dc6d9d9c4b-EMAIL_CAMPAIGN_2024_07_05_01_54&utm_medium=email&utm_term=0_1c9568dbdc-dc6d9d9c4b-%5BLIST_EMAIL_ID%5D]

This canine blood smear shows polychromasia, anisocytosis, and spheroctosis.

Polychromasia and anisocytosis are evidence of a regenerative anemia, so perform a reticulocyte count to confirm a regenerative response.

Polychromasia is variation in red blood cell (RBC) color; anisocytosis is variation in RBC size.

Spherocytosis typically occurs due to immune-mediated damage to the BC membrane resulting in a rounded (i.e., spherical) appearance to the RBC. This is seen most commonly in dogs with immune-mediated hemolytic anemia (IMHA), but may occur secondary to blood transfusion, so always interpret with caution.

Reticulocytes are immature RBCs, released early from the bone marrow; use new methylene blue staining to identify these cells. Look for reticulocytosis with regenerative anemia in most species except horses. The reticulocyte count is a measure of the bone marrow response to anemia.

For more info and images, see the Cornell Clin Path website: BC Morphology and Hematology Atlas.

https://www.merckvetmanual.com/circulatory-system/anemia/anemia-in-animals

https://www.merckvetmanual.com/circulatory-system/anemia/hemolytic-anemia-in-animals

https://eclinpath.com/hematology/tests/absolute-reticulocyte-count/

https://www.merckvetmanual.com/circulatory-system/hematopoietic-system-introduction/red-blood-cells-in-animals

How well did you know this?
1
Not at all
2
3
4
5
Perfectly
10
Q

A ten-year-old male neutered orange tabby cat is presented with asymmetric crusty, pruritic dermatoses around his eyes, towards his ears. The remainder of his dermatological exam is unremarkable.
The cat lives indoors but likes to sleep in a sunny window seat.
What condition is most likely?
A - Frostbite
B - Squamous cell carcinoma
C - Pemphigus foliaceus
D - Feline proliferative and necrotizing_otitis externa
E - Ceruminous aland carcinoma
/L

A

Answer: Squamous cell carcinoma

Squamous cell carcinoma is commonly associated with solar keratosis, especially in white/sparsely-haired locations in cats. Lesions are common on the ears, nose, lips, and frontal ridges. There is no breed or sex predilection.
Surgical excision is the treatment of choice, but advanced tumors may have invaded surrounding bony structures making surgical excision impossible. Adjunctive radiation therapy may be considered. These lesions can be prevented by minimizing UV radiation exposure in at-risk animals.
Ceruminous gland carcinoma can range in appearance from fibrotic dermal nodules to ulcerated plaques. It would not occur on the tips of the pinnae and would not be bilateral.
Frostbite would not occur in a cat that lives indoors. It can cause a similar presentation.
Pemphigus foliaceus is an immune-mediated disorder of the skin. Look for erosions, ulcerations, and thick encrustations of the mucocutaneous junctions and skin.
Feline proliferative and necrotizing otitis externa is rare and occurs on the inner aspect of the pinnae, the ear canal, or the external aural orifice (not the tips). It has an unknown etiology.

https://www.merckvetmanual.com/integumentary-system/tumors-of-the-skin-and-soft-tissues/epidermal-and-hair-follicle-tumors-in-animals?autoredirectid=14257

How well did you know this?
1
Not at all
2
3
4
5
Perfectly
11
Q

An eight-year-old female Saint Bernard is presented with a six-week history of right pelvic limb lameness and swelling above the hock.
A radiograph of the affected limb is shown below.
Three-view thoracic radiographs are unremarkable.
Based on the top differential, what is the treatment of choice?

A - 6 - 8 weeks of oral clindamycin
B - Intralesional mitomycin D
C - Limb amputation
D - Lipid complex amphotericin B IV
E - Intravenous cyclophosphamide

A

Answer: Limb amputation.

Limb amputation is the treatment of choice in this dog with osteosarcoma. Also consider adjunctive chemotherapy due to the likelihood of metastatic disease at the time of diagnosis.
On radiographic review the lesions are characteristic of a primary bone tumor. There is a large, expansile, destructive osseous lesion of the distal right tibia with cortical destruction, most severe at the caudal and medial margins. There is also mild, irregular periosteal proliferation and soft tissue thickening at the site.
Osteosarcoma is a top differential for an older large-breed dog with an expansile, destructive osseous lesion.
Click here to see normal canine tibial radiographs.
Radiographic interpretation and images courtesy, Dr. A. Zwingenberger and Veterinary Radiology.

https://www.merckvetmanual.com/musculoskeletal-system/osteopathies-in-small-animals/bone-tumors-in-dogs-and-cats

How well did you know this?
1
Not at all
2
3
4
5
Perfectly
12
Q

Many hens in a large backyard flock recently became ill after the owner purchased four new chickens. The sick hens are gasping, coughing, and sneezing.
They have poor appetite and act depressed. Some affected birds have watery green diarrhea and swelling of the head and neck.
A few have paralyzed legs and wings, twisted necks, are circling, and have tremors or clonic spasms.
Birds are not laying well and some eggs are misshapen with watery albumen. Many of the sickest birds have died.
Necropsy of a dead chicken shows petechial hemorrhages on the mucosal surface of the proventriculus and gizzard (left).
What should be done next to address this problem?

A - Report outbreak to regulatory authorities
B - Increase poultry house temperatures to reduce morbidity
C - Cull affected birds and vaccinate the remainder
D - Disinfect housing with phenolic compounds, barrier precautions for staff
E - Treat all affected chickens with amprolium

A

Answr: Report outbreak to regulatory authorities

This history and the necropsy findings are consistent with viscerotropic velogenic Newcastle disease (VVND), which is reportable. Contact the appropriate regulatory authorities immediately.
Gross lesions such as the petechiae seen here in the proventricular mucosa are not usually seen with low-virulence Newcastle disease (loNDV). The clinical history is also suggestive of VVND.
The acute form of NDV is clinically indistinguishable from highly pathogenic avian influenza and differentiated by PCR or isolating the hemagglutinating virus identified by inhibition with Newcastle disease antiserum.
NDV occurs worldwide and chickens usually present with acute respiratory disease. Occasionally diarrhea, neurological problems, or depression predominate. Virulence varies between the VND and loNDV forms.
Vaccines against NDV decrease clinical signs and death. Depopulate infected poultry.
Sometimes producers increase the temperature of the poultry house to decrease morbidity and mortality in flocks with suspected avian influenza.

Amprolium is a treatment for coccidiosis.

https://www.merckvetmanual.com/poultry/newcastle-disease-and-other-paramyxovirusinfections/newcastle-disease-in-poultry?autoredirectid=21574

[Link] (https://www.merckvetmanual.com/poultry/newcastle-disease-and-other-paramyxovirusinfections/newcastle-disease-in-poultry?autoredirectid=21574)

[Link] (https://www.merckvetmanual.com/poultry/avian-influenza-in-poultry-and-wild-birds/avian-influenza-in-poultry-and-wild-birds?autoredirectid=24360&autoredirectid=12669)

How well did you know this?
1
Not at all
2
3
4
5
Perfectly
13
Q

A chicken carcass is presented for necropsy from a backyard layer flock. The client has noticed several birds with transient paralysis, and reports the birds received no hatchery vaccinations.
This bird was depressed for a couple days before death. Necropsy reveals the findings shown below.
What type of etiologic agent is associated with this disease?
A - Birnavirus
B - Paramyxovirus
C - Herpesvirus
D - Pneumovirus
E - Adenovirus

A

Answer: Herpesvirus.

This is an example of Marek’s disease, caused by a highly contagious and cell-associated herpesvirus. It is very common and most birds are presumed infected; subclinical disease can lower both growth and egg production rates.
Fully infective viruses are released from feather follicles and the enveloped virus can be found in dander and dust. Clinical presentation is often as described in this case.
Clinical disease is highly variable, and depends on virus and vaccine strain and dose, host sex and genetics, maternal antibody, age at exposure, and underlying stress.
The most common finding at necropsy is enlarged nerves that lose their striations, especially the sciatic (as seen here), vagus, and brachial. Tumors (diffuse or nodular) may be seen in various organs.
Dx: Classically, look for tumors AND enlarged nerves at necropsy. Use histology and viral PCR to confirm. Prevent with vaccination and strict sanitation control.
Viral inclusion body hepatitis is an example of a condition caused by an adenovirus.
Swollen head syndrome is an example of a condition caused by a pneumovirus.
Infectious bursal syndrome is an example of an infection caused by a birnavirus.
Newcastle disease is an example of an infection caused by a paramyxovirus.

[Link] (https://www.merckvetmanual.com/poultry/neoplasms-in-poultry/marek-s-disease-in-poultry?autoredirectid=23789&autoredirectid=23789&autoredirectid=16879)

https://zukureview.com/zuku-qod/navle/1856?chosen=ofLe9THvupR3%2BuRFJTK0qg%3D%3D&utm_source=Zukureview+Subscribers&utm_campaign=0d399fedcd-EMAIL_CAMPAIGN_2024_06_28_01_54&utm_medium=email&utm_term=0_1c9568dbdc-0d399fedcd-%5BLIST_EMAIL_ID%5D

How well did you know this?
1
Not at all
2
3
4
5
Perfectly
14
Q

Several cattle within a herd present with high fevers, up to 106°F (41°C), feet-stomping, and salivation.
Some of the cows have vesicles in the interdigital space, and vesicles and denuded areas in the mouth.
Necropsy of one of the cows shows white streaks on the heart in addition to the above findings.
Which one of the following choices is a top differential?

A - Bovine papillar stomatitis
B - Foot-and-mouth disease
C - Bovine respiratory syncytial virus
D - Bluetongue
E - Q fever

A

Answer: Foot-and-mouth disease

The top differentials for vesicular disease in cattle include foot-and-mouth disease (FMD) and vesicular stomatitis.
The US has been free of FMD since 1929, therefore FMD is a reportable disease. The disease affects cloven-hoofed animals. Pigs and cattle are most severely affected.
Vesicular stomatitis is clinically indistinguishable from FMD but also can affect horses.
Bovine papillar stomatitis causes proliferative raised lesions.
Bluetongue primarily affects sheep (typically subclinical in cattle). It causes vasculitis with clinical presentation of fever, edema, reluctance to eat/move.

https://www.merckvetmanual.com/generalized-conditions/foot-and-mouth-disease/foot-and-mouth-disease-in-animals?autoredirectid=14225

https://www.merckvetmanual.com/generalized-conditions/vesicular-stomatitis-in-large-animals/vesicular-stomatitis-in-large-animals?autoredirectid=23195&autoredirectid=23195&autoredirectid=23195&autoredirectid=23195&autoredirectid=23195

https://zukureview.com/zuku-qod/navle/1855?chosen=icu3e%2BDR6efKdg7ceam0z%2BwzUSDDL5MS%2B3/ukxNn4yZxyx82oCVfcsOP56gJum1G&utm_source=Zukureview+Subscribers&utm_campaign=be132ef7d1-EMAIL_CAMPAIGN_2024_06_27_01_54&utm_medium=email&utm_term=0_1c9568dbdc-be132ef7d1-%5BLIST_EMAIL_ID%5D

How well did you know this?
1
Not at all
2
3
4
5
Perfectly
15
Q

A two-year-old female intact nulliparous cat is presented three weeks post-estrus for an acutely enlarged mammary gland that is painful upon palpation.
The cat is bright, alert, and responsive with otherwise normal physical examination findings. It is not possible to express any milk or discharge from the swollen gland.
What is the recommended treatment?

A - Ovariohysterectomy
B - Warm compresses
C - Broad-spectrum antimicrobials
D - Domperidone
E - Progesterone therapy

A

Answer: Ovariohysterectomy.

This is feline mammary hyperplasia (a.k.a. hypertrophy, fibroadenomatosis), perform ovariohysterectomy (OVH) to prevent recurrence. It can resolve on its own within a few weeks but is likely to recur without OVH. Medical Tx includes progesterone blockers.
Mammary hyperplasia occurs secondary to high progestogen concentrations (endogenous or exogenous) and can occur in males or females, intact or sterilized.
DDX include mastitis and mammary gland neoplasia. Mastitis typically occurs in lactating queens and presents with systemic signs. Neoplasia is more common in older cats (average 11 y) and mammary swelling is typically not acute.
Tx mastitis with warm compresses and broad-spectrum antimicrobials.
Domperidone and warm compresses are contraindicated as these can stimulate lactation.

https://www.merckvetmanual.com/reproductive-system/reproductive-diseases-of-the-female-small-animal/mammary-hypertrophy-in-cats

https://www.merckvetmanual.com/reproductive-system/reproductive-diseases-of-the-female-small-animal/mastitis-in-small-animals

https://www.merckvetmanual.com/reproductive-system/mammary-tumors-in-cats/mammary-tumors-in-cats?autoredirectid=23405

How well did you know this?
1
Not at all
2
3
4
5
Perfectly
16
Q

A ten-year-old male neutered mixed breed dog is presented with a two month history of progressive left thoracic limb lameness.
Radiographs and an ultrasound of the affected limb are shown below.
What is the most likely diagnosis?

A - Rheumatoid arthritis
B - Joint neoplasia
C - Degenerative joint disease
D - Septic arthritis
E - Fragmented coronoid process

A

Answer: Joint neoplasia

This is neoplasia involving the elbow joint.
Radiographic interpretation: There is geographic lysis involving the distal humerus, proximal ulna, and to a lesser degree the proximal radius. The areas of bone loss are multifocal with relatively well circumscribed borders. The anconeal process is absent. There is a large soft tissue swelling centered around the left elbow.
Ultrasound: There is a heterogeneous, hyperechoic lesion surrounding the caudal aspect of the elbow and also extending medially and laterally. There are severe defects within the bone and the mass lesion extends into the bone.
Fine needle aspirate and cytology of the lesion revealed mesenchymal proliferation and marked atypia with increased plasma cells and recent hemorrhage - probable sarcoma.
Click here to see normal canine elbow radiographs.
Radiographic interpretation and images courtesy, Dr A. Zwingenberger and Veterinary Radiology.
Normal radiograph links courtesy, Imaging Anatomy Univ. of Illinois Vet Med.

https://www.merckvetmanual.com/musculoskeletal-system/osteopathies-in-small-animals/bone-tumors-in-dogs-and-cats

http://mirc.veterinaryradiology.net/storage/ss1/docs/20090624000302739/1/MIRCdocument.xml

https://vetmed.illinois.edu/imaging_anatomy/index.html

How well did you know this?
1
Not at all
2
3
4
5
Perfectly
17
Q

An 11-year-old spayed female German shepherd is presented for pelvic limb weakness.
Which one of the following choices can be discerned from these radiographs?

A - T3-L3 intervertebral disc disease
B - Degenerative myelopathy
C - Cardiomegaly
D - Caval syndrome
E - Normal radiographs

A

Answer: Normal radiographs

These are normal thoracic radiographs. There is a small amount of air in the thoracic esophagus dorsal to the carina. This is normal and usually transient.
There is moderate degenerative joint disease of the scapulohumeral joints as well as mild ventral spondylosis at multiple sites of the thoracic and thoracolumbar spine.
Click here to see more normal canine thoracic radiographs.
Radiographic interpretation and images courtesy, Dr. A. Zwingenberger and Veterinary Radiology.

https://vetmed.illinois.edu/imaging_anatomy/canine/thorax/ex02/thorax02.html

How well did you know this?
1
Not at all
2
3
4
5
Perfectly
18
Q

What is the purpose of placing a permanent rumen fistula in some cattle?

A - Treat traumatic reticuloperitonitis in the fistulated animal
B - Manage vagal indigestion in the fistulated animal
C - Create transfaunation donors
D - Allow for direct administration of medications into rumen long-term
E - This procedure is not done on a permanent basis in cows

A

Answer: Create transfaunation donors

The rumen is permanently fistulated in the left paralumbar fossa to allow cows to be donors for transfaunation of rumen contents to other cows.
Transfaunation is used to treat other cows with conditions like ruminal acidosis, vagal indigestion, and peritonitis.
Fresh ruminal fluid contains microbes (bacteria and protozoa), volatile fatty acids (VFAs), microbial proteins, vitamins, minerals, and other buffers.
Ruminal fluid can also be collected via siphon from a stomach tube or from animals at the slaughter house.
Ruminal fistulas are also used to study ruminant physiology.
Click here to see a video of a fistulated steer from Oregon State University Extension Service.
Ref: The Oregon State University Agricultural Extension. Photo courtesy of National Nuclear Security Administration/Nevada Field Office.

https://www.merckvetmanual.com/pharmacology/systemic-pharmacotherapeutics-of-the-digestive-system/drugs-for-specific-purposes-in-the-ruminant-digestive-system

How well did you know this?
1
Not at all
2
3
4
5
Perfectly
19
Q

Which one of the following sets of conditions predispose a cow to metritis?
A - Agalactia, milk fever, bovine vibriosis
B - Contaminated calving environment, abortion, hypomagnesemia
C - Milk fever, malnutrition, excess dietary zinc
D - Strep. agalactiae mastitis, retained placenta, laminitis
E - Dystocia, overfeeding in dry period, Ca-P imbalance in feed

A

Answer: Dystocia, overfeeding in dry period, Ca-P imbalance in feed.

Cows are predisposed to post-parturient metritis by many things, including:
-Dystocia
-Overfeeding in dry period
-Ca-P imbalance in feed
Cows are also predisposed to metritis by:
-Retained placenta
-Contaminated calving environment
-Abortion
-Malnutrition
In cattle, the causative bacterial organisms isolated most often are Trueperella pyogenes alone or with Fusobacterium necrophorum or other gram-negative anaerobes.
Note the name changes - Trueperella pyogenes used to be called Arcanobacterium pyogenes and before that it was classified as Corynebacterium pyogenes.
Specific diseases associated with bovine metritis or endometritis include brucellosis, leptospirosis, trichomoniasis, and bovine campylobacter.

https://www.merckvetmanual.com/reproductive-system/uterine-diseases-in-production-animals/metritis-in-production-animals?ruleredirectid=463

https://zukureview.com/zuku-qod/navle/512?chosen=/D6urn6ARUuFMQMx6IQuUpGefSmQNn8ujgQ7d4bkAC3UiI8ygQhpHLyeoPcYTiv2AO8zpV8l9u6nEG1yZ27Zlw%3D%3D&utm_source=Zukureview+Subscribers&utm_campaign=7290f8470c-EMAIL_CAMPAIGN_2022_11_24_02_50&utm_medium=email&utm_term=0_1c9568dbdc-7290f8470c-%5BLIST_EMAIL_ID%5D

How well did you know this?
1
Not at all
2
3
4
5
Perfectly
20
Q

An adult male neutered mixed breed dog is presented for epiphora, conjunctivitis, and periocular pruritus.
An image of the right eye following administration of local anesthetic is shown below.
What is the most likely diagnosis?
A - Onchocerca cervicalis
B - Dirofilaria immitis
C - Theileria spp.
D - Thelazia californiensis
E - Spirocerca lupi

A

Answer: Thelazia californinsis.

This is Thelazia californiensis in the western U.S. (in Europe and Asia it is
T. callipaeda). Clinical signs include epiphora, conjunctivitis, ocular pruritus, keratitis w/ corneal opacity and ulceration, and blindness (occasionally).
T. californiensis is zoonotic and carried by muscid and fruit flies. The flies act as intermediate hosts and deposit infective larvae on the eye while feeding on ocular secretions.
Click here to see a video of T. californiensis in a dog’s eye.
Tx: Manually remove worms with forceps after sedation and local anesthetic. Additional Tx options include ivermectin subQ, spot-on moxidectin, or oral milbemycin oxime. Ocular moxidectin or levamisole solution are an option but may cause irritation.
Spirocera lupi cause parasitic nodules in the aorta, esophagus, and stomach of dogs worldwide.
Theileria spp. are tick-borne blood parasites affecting horses and cattle.
Onchocerca cervicalis causes ophthalmic lesions and ventral midline dermatitis in equids.
Dirofilaria immitis is the causative agent of heartworm in dogs and cats.

https://www.merckvetmanual.com/eye-diseases-and-disorders/eyeworm-disease/eyeworms-of-small-animals

Eyeworms of Small Animals (Thelaziasis)

Definitions and Causative Agents

•	Thelaziasis: Parasitic infection caused by nematodes of the genus Thelazia.
•	Species:
•	Thelazia callipaeda: Found in Europe and Asia.
•	Thelazia californiensis: Found in the western US.

Hosts

•	Definitive hosts: Dogs, cats, foxes, wolves, martens, badgers, lagomorphs, and humans for T. callipaeda; dogs, cats, and deer for T. californiensis.
•	Vectors: Zoophilic fruit flies, such as Phortica variegata (Europe) and Phortica okadai (Asia).

Transmission

•	Transmission occurs when infected male flies harboring L3 larvae feed on the eye of a suitable host.

Clinical Signs

•	Mild Infections: Mild inflammation, lacrimation, and foreign-body sensation.
•	Severe Infections: Edema, corneal ulceration, conjunctivitis, blindness, excessive lacrimation, ocular pruritus, keratitis, corneal opacity and ulceration, hyperemia.

Diagnosis

•	Direct observation of parasites on the conjunctiva or in tear ducts.
•	Removal of parasites with forceps after local anesthesia.

Treatment

•	Medications:
•	Ivermectin: 0.2 mg/kg SC.
•	Milbemycin oxime: 0.5 mg/kg PO, two treatments one week apart.
•	Moxidectin: 2.5% spot-on treatment or 0.17 mg/kg SC.
•	Ocular solutions: 1% moxidectin or 2% levamisole.
•	Ointments: 1% levamisole or 4% morantel.

Prevention

•	Seasonal Prevention:
•	Moxidectin: Sustained-release, 0.17 mg/kg SC.
•	Milbemycin oxime: Dosage recommended for heartworm prevention.
•	Ivermectin: 0.2 mg/kg PO.

Pathophysiology

•	Adult nematodes cause mechanical irritation and inflammation of the ocular tissues, leading to various degrees of clinical signs depending on the worm burden.

Assessment

•	Clinical examination of the eyes for signs of inflammation and presence of worms.
•	Confirmatory diagnosis through visualization and removal of worms.
How well did you know this?
1
Not at all
2
3
4
5
Perfectly
21
Q

A six-year-old female spayed cocker spaniel is presented with a two-day history of lethargy.
Exam reveals petechiae and ecchymoses on the buccal mucosa, ventral abdomen, and inner pinnae.
There is hyphema OD.
Click the labwork icon to review hematology, blood chemistry, and urinalysis test results. Prothrombin time (PT) and activated partial thromboplastin time (aPTT) are normal.
What is the most likely diagnosis?
A - Immune-mediated thrombocytopenia
B - Disseminated intravascular coagulation
C - Hepatic insufficiency
D - Anticoagulant rodenticide toxicity_
E - Von Willebrands disease

A

Answer: Immune-mediated thrombocytopenia

This is immune-mediated thrombocytopenia, the most common cause of spontaneous bleeding in dogs. It may be primary (idiopathic, most common) or secondary to infection, neoplasia, or certain drugs.
Most common in middle-aged females; cockers, poodles, and old English sheepdogs predisposed.
Dogs with disseminated intravascular coagulation (DIC) have thrombocytopenia with a prolonged PT and aPTT.
Follow this link to see a table of the four most important coagulation disorder patterns.

Immune-Mediated Thrombocytopenia (ITP) in Animals

Definition

•	ITP: Immune-mediated destruction of platelets or marrow megakaryocytes.
•	Also Known As: Idiopathic thrombocytopenic purpura.

Causes and Risk Factors

•	Idiopathic: No identifiable underlying cause.
•	Risk Factors: Middle-aged, spayed female dogs, especially Cocker Spaniels.

Pathophysiology

•	Mechanism: Immune system targets and destroys platelets or megakaryocytes.
•	Autoantibodies: Directed against platelet surface antigens.

Clinical Signs

•	Bleeding: Petechiae, ecchymoses, melena, epistaxis.
•	Severe Thrombocytopenia: Platelet counts <30,000/μL, often <10,000/μL.

Diagnosis

•	Exclusion: Rule out other causes of thrombocytopenia.
•	Tests: Bone marrow aspirate (rarely needed), platelet count, clinical signs.

Treatment

•	Corticosteroids: High-dose initial treatment, tapered gradually.
•	Vincristine: 0.01-0.02 mg/kg IV, shortens recovery time.
•	Transfusion: Fresh whole blood for severe anemia.
•	Splenectomy: For recurrent cases.
•	Monitoring: Regular platelet count checks during steroid tapering.
•	Avoid: Drugs interfering with coagulation.

Disseminated Intravascular Coagulation (DIC) in Animals - Comprehensive Study Guide

Definition

•	Disseminated Intravascular Coagulation (DIC): A secondary condition characterized by systemic activation of blood coagulation, leading to widespread clotting, bleeding, and organ damage.

Causative Agents

•	Primary Diseases: Bacterial, viral, rickettsial, protozoal, parasitic infections, heat stroke, burns, neoplasia, and severe trauma.

Pathophysiology

•	Mechanism: Systemic inflammatory response activates coagulation, leading to widespread clot formation. This results in consumption of coagulation factors and platelets, causing bleeding and organ failure due to microvascular thrombosis.

Symptoms

•	Clinical Signs: Variable, ranging from no overt signs to severe bleeding, organ failure, and microvascular thrombosis.

Diagnosis

•	Laboratory Tests: Prolonged APTT, PT, elevated D-dimer, reduced fibrinogen, and platelet count. Thromboelastography can help differentiate stages.

Treatment

•	Underlying Cause: Identify and treat the primary disease.
•	Medications: Heparin in hypercoagulable stages; fresh frozen plasma in hypocoagulable stages.
•	Supportive Care: Fluids, plasma expanders to maintain effective circulating volume.

Zuku Coagulation disorder

https://zukureview.com/node/100828

How well did you know this?
1
Not at all
2
3
4
5
Perfectly
22
Q

Which one of the following sets of conditions predispose a cow to metritis?
A - Contaminated calving environment, abortion,. hypomagnesemia
B - Strep. agalactiae mastitis, retained placenta, laminitis
C - Dystocia, overfeeding in dry_period, Ca-P imbalance in feed
D - Milk fever, malnutrition, excess dietary zinc
E - Agalactia, milk fever, bovine vibriosis

A

Answer: C - Dystocia, overfeeding in dry_period, Ca-P imbalance in feed

Cows are predisposed to post-parturient metritis by many things, including:
-Dystocia
-Overfeeding in dry period
-Ca-P imbalance in feed
Cows are also predisposed to metritis by:
-Retained placenta
-Contaminated calving environment
-Abortion
-Malnutrition
In cattle, the causative bacterial organisms isolated most often are Trueperella pyogenes alone or with Fusobacterium necrophorum or other gram-negative anaerobes.

Note the name changes - Trueperella pyogenes used to be called Arcanobacterium pyogenes and before that it was classified as
Corynebacterium pyogenes.
Specific diseases associated with bovine metritis or endometritis include brucellosis, leptospirosis, trichomoniasis, and bovine campylobacter.

Metritis in Production Animals - Comprehensive Study Guide

Definition

•	Metritis: Inflammation of the uterus, typically occurring postpartum, associated with bacterial infection.

Causative Agents

•	Bacteria: Escherichia coli, Trueperella pyogenes, Fusobacterium necrophorum, Porphyromonas levii.

Pathophysiology

•	Mechanism: Infection leads to systemic illness, characterized by a dysbiosis in the uterine microbiome favoring pathogenic bacteria. The inflammation results in an abnormally enlarged uterus and fetid uterine discharge.

Symptoms

•	Acute Puerperal Metritis: Systemic signs such as fever >39.5°C, decreased milk production, anorexia, depression, and fetid, watery, reddish-brownish uterine discharge.
•	Clinical Metritis: Enlarged uterus with purulent uterine discharge without systemic illness.

Diagnosis

•	Methods: Clinical examination, visual observation of discharge, vaginal discharge scoring, bacterial culture, and PCR.

Treatment

•	Antibiotics:
•	Ceftiofur hydrochloride: 2.2 mg/kg IM every 24 hours for 5 days.
•	Ceftiofur crystalline free acid: 6.6 mg/kg SC twice, 72 hours apart.
•	Other antibiotics: Procaine G penicillin, oxytetracycline, and ampicillin trihydrate.
•	Supportive Care: Fluid therapy, anti-inflammatory treatment (flunixin meglumine, ketoprofen, aspirin).

Prevention

•	Management: Good hygiene, clean maternity areas, use of vaccines, and feeding antioxidants like vitamin E, selenium, and beta-carotene.
How well did you know this?
1
Not at all
2
3
4
5
Perfectly
23
Q

Two eight-week old calves are presented down and extremely weak.
They are depressed and lying in pools of foul-smelling brown diarrhea with a small amount of blood.
Another calf died suddenly the previous night with no prior signs.
The down calves are dehydrated, with rectal temperatures of 105.2° and 105.6° F (40.7° - 40.9°
C) [N=101.5°-103.5°F, N=37.8°-39.7° C], respectively.
What is the treatment plan?

A - Immunize calves and adult cattle with
MLV vaccine; probiotics for sick calves
B - Isolate sick calves; oral electrolytes
C - Cull sick calves; prophylactic oxytetracycline in feed for well animals
D - Corticosteroids; amprolium in water; rumensin in feed
E - Oral electrolytes, injectable broad-spectrum antimicrobials

A

Answer: E - Oral electrolytes, injectable broad-spectrum antimicrobials

Treat these calves with oral electrolytes and injectable broad-spectrum antimicrobials. Milk feeding will not make diarrhea worse and provides an important source of nutrients and fluids for sick animals.
Fever, diarrhea and sudden death in eight-week-old calves is highly suggestive of septicemia due to salmonellosis.
Isolate sick calves to prevent transmission.

Use of antibiotics is controversial as they may prolong recovery and shedding and yield a carrier calf.
However, if an animal is septic it needs parenteral antibiotics.
Prognosis is poor with neonatal salmonella and deaths can approach 100% in affected calves.
In adults, antibiotics may yield a clinical cure, but Salmonellae can establish in the biliary system and intermittently shed into the Gl system, leading to environmental contamination.
Prevention is dependent on which species of Salmonella is causing the problems-host adapted or environmental.

Comprehensive Summary on Salmonellosis in Animals for BCSE Test Preparation

Definition and Etiology

Salmonellosis is an infection caused by bacteria of the genus Salmonella. It affects many animal species and humans, manifesting in various forms from asymptomatic carriers to severe septicemia. The disease is of significant concern due to its zoonotic potential.

Causative Agents

•	Salmonella enterica subspecies include various serovars such as:
•	S. Typhimurium
•	S. Dublin
•	S. Choleraesuis
•	S. Enteritidis
•	S. Heidelberg

Epidemiology

•	Cattle: S. Typhimurium causes enteritis in young calves, while S. Dublin is more common in older calves and adults, often becoming endemic on farms.
•	Sheep: Outbreaks are common during cold seasons and involve high-density housing conditions.
•	Pigs: Septicemic salmonellosis usually traces back to infected pigs or contaminated environments.
•	Horses: Stress-related factors such as surgery and transport can trigger clinical salmonellosis. Mares can shed the bacteria at parturition, infecting foals.
•	Dogs and Cats: Often asymptomatic carriers, though clinical disease can occur in stressed or young animals.

Pathophysiology

•	Salmonella invades the intestinal mucosa, leading to enteritis and systemic infection. The bacteria produce toxins that disrupt cellular function and cause inflammation.
•	Enteritis and Septicemia: Acute infection can lead to severe enteritis and septicemia, particularly in neonates and immunocompromised animals.

Clinical Signs

•	General Symptoms: Fever, diarrhea, abdominal pain, dehydration, and depression.
•	Acute Enteritis: Fever followed by severe watery diarrhea, sometimes with blood and mucus. In horses, signs include colic, severe dehydration, and metabolic acidosis.
•	Septicemia: High fever, lethargy, diarrhea, and rapid deterioration. In calves, respiratory symptoms and septic arthritis may occur.
•	Carrier State: Animals may intermittently shed Salmonella without showing symptoms, posing a risk for outbreaks.

Diagnosis

•	Clinical Signs: Observation of diarrhea, fever, and other systemic symptoms.
•	Laboratory Tests: Repeated isolation of Salmonella from feces, blood, or tissues. Fecal cultures, PCR assays, and serotyping are used for confirmation.
•	Postmortem Findings: Lesions in the lower ileum, cecum, and colon, characterized by inflammation, necrosis, and sometimes hemorrhage.

Treatment

•	Supportive Care: IV fluid and electrolyte replacement to manage dehydration and electrolyte imbalances. Polyionic isotonic fluids are commonly used.
•	Antimicrobials: Selection based on sensitivity testing. Commonly used antimicrobials include enrofloxacin, penicillin, gentamicin, and metronidazole. Note the potential nephrotoxicity of aminoglycosides in dehydrated animals.
•	NSAIDs: Used to manage pain and inflammation; examples include flunixin meglumine, meloxicam, and firocoxib.
•	Plasma and Colloids: Administered to address hypoproteinemia and provide coagulation factors.

Prevention and Control

•	Hygiene and Biosecurity: Strict hygiene practices in calving areas, isolation of infected animals, and thorough cleaning and disinfection of facilities. An “all-in/all-out” management system can help prevent spread.
•	Vaccination: Not commonly used in all species but may be beneficial in pigs to reduce infection and shedding.
•	Environmental Management: Control of rodents and wildlife that can contaminate feed and water sources.

Zoonotic Risk

Salmonella infections can be transmitted from animals to humans, particularly through contact with contaminated feces or environments. This is especially concerning in immunocompromised individuals. Strict biosecurity protocols and personal hygiene are essential when handling infected animals.

How well did you know this?
1
Not at all
2
3
4
5
Perfectly
24
Q

What kind of organism causes equine granulocytic anaplasmosis?
A - Anaplasma
B - Spirochete
C - Ehrlichia
D - Chlamydia
E - Protozoa

A

Answer: Anaplasma

This is a tricky question to help you remember that two diseases
FORMERLY classified as Ehrlichia have now been RE-classified.
The causative organism of equine granulocytic anaplasmosis (a.k.a. anaplasmosis; formerly equine granulocytic ehrlichiosis) was
originally classified as Ehrlichia equi, but is now called
ANAPLASMA phagocytophilum due to DNA sequencing studies.
Do not confuse equine anaplasmosis, a necrotizing vasculitis, with BOVINE anaplasmosis, which primarily causes an anemia with icterus
and fever.

Another name change occurred with the causative organism of Potomac horse fever (PHF), from Ehrlichia risticii to NEOrickettsia
risticil.
PHF presents as a febrile
colitis/diarrhea, with laminitis 3-5 d after diarrhea in horses of all
ages: A big rule out is salmonella (think septicemia/fever + diarrhea).

Key Information on Equine Granulocytic Anaplasmosis

Etiology and Transmission

•	Causative Agent: Anaplasma phagocytophilum.
•	Vector: Ixodes ticks (e.g., I. pacificus, I. scapularis).
•	Transmission: Tick bites; seasonal occurrence.

Clinical Signs

•	Fever: Up to 107°–108°F.
•	Symptoms: Depression, limb edema, ataxia, anorexia, icterus, petechiation.
•	Blood Abnormalities: Leukocytopenia, pancytopenia, thrombocytopenia, cytoplasmic inclusion bodies in neutrophils.

Diagnosis

•	Tests: PCR, serology, blood smear (Giemsa or Wright-Leishman stain).
•	Differential Diagnoses: Viral encephalitis, equine infectious anemia, liver disease.

Treatment and Control

•	Antibiotics: Oxytetracycline.
•	Supportive Care: Corticosteroids, fluid therapy, pain management.
•	Prevention: Tick control; no vaccine available.

Anaplasmosis in Ruminants - Comprehensive Veterinary Information

Definitions and Terminology:

•	Anaplasmosis: A tick-borne disease of ruminants caused by intracellular bacteria infecting red blood cells, leading to severe anemia and fever.

Causative Agents:

•	Pathogens: Anaplasma marginale, A. centrale, A. ovis, A. phagocytophilum.

Physiopathology:

•	Transmission: Ticks (Dermacentor, Rhipicephalus), mechanical via biting dipterans, contaminated needles.
•	Pathogenesis: Bacteria infect erythrocytes, causing their destruction by the immune system, leading to anemia and jaundice.

Clinical Findings:

•	Symptoms: Progressive anemia, fever, weight loss, decreased milk production, icterus. Severe cases may lead to death.

Diagnosis:

•	Tests: Blood smears (Giemsa-stain), serologic tests (ELISA), PCR.
•	Microscopic Findings: Anaplasma organisms in erythrocytes.

Treatment:

•	Antibiotics: Tetracyclines (e.g., oxytetracycline 20 mg/kg IM), imidocarb (1.5 mg/kg SC).
•	Supportive Care: Blood transfusions for severely affected animals.

Control and Prevention:

•	Vaccination: Use of live vaccines (e.g., A. centrale) in endemic areas.
•	Tick Control: Acaricides and environmental management.

Key Information on Potomac Horse Fever

Etiology and Transmission

•	Causative Agent: Neorickettsia risticii.
•	Transmission: Ingestion of aquatic insects (e.g., mayflies, caddisflies) containing infected trematodes.

Clinical Signs

•	Fever: Up to 105°F.
•	Symptoms: Depression, anorexia, colic, diarrhea, laminitis.
•	Blood Abnormalities: Leukopenia followed by leukocytosis, thrombocytopenia.

Diagnosis

•	PCR Testing: On blood or feces.
•	Serology: Paired serum samples showing rising titers.

Treatment and Control

•	Antibiotics: Oxytetracycline.
•	Supportive Care: IV fluids, NSAIDs.
•	Vaccination: Provides partial protection.
•	Prevention: Minimize exposure to aquatic habitats and insects.
How well did you know this?
1
Not at all
2
3
4
5
Perfectly
25
Q

Which canine parasite can cause cutaneous larva migrans in people?
A - Habronema spP.
B - Trichostrongylus spp.
C - Ancylostoma spp.
D - Spirocerca sp.
E - Trichuris spp.

A

Answer: Ancylostoma

Hookworms (Ancylostoma spp.)
may cause cutaneous larva migrans in people.
Note that roundworms (Toxocara spp., Toxasacaris spp., Baylisascaris spp.) are also zoonotic, causing visceral and ocular larva migrans in people.
In dogs, whipworms, Trichuris spp. are associated with a hypoadrenocorticism-like
syndrome (hyponatremia, hyperkalemia, azotemia, metabolic acidosis).
Whipworm infection has been suggested as one cause of cecocolic intussusception.
Habronema spp. in horses can cause tumorlike stomach nodules
and sometimes cutaneous lesions.
Trichostrongylus spp. cause parasitic gastritis and enteritis in sheep, goats, and cattle.

Hookworms in Small Animals - Comprehensive Veterinary Information

Definitions and Terminology:

•	Hookworms: Intestinal nematodes, primarily affecting dogs and cats, leading to significant health issues.

Causative Agents:

•	Species:
•	Ancylostoma caninum (dogs)
•	Ancylostoma tubaeforme (cats)
•	Ancylostoma braziliense (dogs and cats)
•	Ancylostoma ceylanicum (dogs and cats)
•	Uncinaria stenocephala (dogs and cats in cooler regions)

Physiopathology:

•	Life Cycle: Eggs pass in feces, hatch in soil, and larvae infect hosts through ingestion, skin penetration, or transmammary routes.
•	Larval Migration: Skin penetration, blood to lungs, coughed up, swallowed, mature in intestines. Arrested larvae in tissues can reactivate.

Clinical Findings:

•	Acute Anemia: Normocytic, normochromic progressing to hypochromic, microcytic in puppies.
•	Chronic Infections: Anemia, melena, hypoproteinemia, weakness, diarrhea.
•	Lesions: Hemorrhagic enteritis, pneumonia in pups, dermatitis (interdigital spaces).

Diagnosis:

•	Fecal Flotation: Detects thin-shelled, oval eggs.
•	Antigen Tests: For hookworm detection.
•	Postmortem: Examination of intestines for adult worms.

Treatment:

•	Anthelmintics:
•	Fenbendazole
•	Moxidectin
•	Pyrantel pamoate
•	Milbemycin
•	Nitroscanate
•	Supportive Care: Blood transfusions, iron supplements, high-protein diet for severe anemia.
•	Drug Resistance: Monitor efficacy post-treatment, especially in A. caninum cases.

Control and Prevention:

•	Regular Deworming: Based on age and risk factors.
•	Sanitation: Dispose of feces promptly, maintain clean environments.
•	Preventive Measures: Treat pregnant bitches, routine fecal checks.

Key Points:

•	Zoonotic Potential: A. braziliense and A. ceylanicum can infect humans.
•	Drug Resistance: Emerging problem, particularly in the southeastern US.

https://www.merckvetmanual.com/digestive-system/gastrointestinal-parasites-of-small-animals/roundworms-in-small-animals

https://zukureview.com/zuku-qod/navle/1699?chosen=27%2BrMcuytoyi7l9pz3P%2BT2EQjV1w9o05nPa77jLkjhc%3D&utm_source=Zukureview+Subscribers&utm_campaign=6ea6a9efb9-EMAIL_CAMPAIGN_2024_04_16_01_52&utm_medium=email&utm_term=0_1c9568dbdc-6ea6a9efb9-%5BLIST_EMAIL_ID%5D

How well did you know this?
1
Not at all
2
3
4
5
Perfectly
26
Q

A female veterinarian is pregnant and her physician tests her serologically for toxoplasmosis.
IgM is negative.
IgG is positive.
What is the most appropriate interpretation?
A - Mother is safe, baby at risk
B - Both mother and baby are safe
C - Both mother and baby at risk
D - Mother at risk, baby is safe
E - Need to re-check in 2 weeks for rising titers

A

Answer: Both mother and baby are safe

Both mother and baby are safe.
The toxoplasmosis organism causes birth defects in a developing fetus if a mother is infected for the first time in her life while pregnant (ie: IgM
positive while pregnant).
Toxoplasmosis is not generally dangerous to immune-competent people and a positive IgG result suggests a previous infection.
There are challenges to toxoplasmosis testing in pregnant women (false positives).
If a pregnant woman is IgM positive, confirmatory tests must be done.
Click here for more on pregnancy and toxoplasmosis.

Comprehensive Summary on Toxoplasmosis in Animals for BCSE Test Preparation

Definition and Etiology

Toxoplasmosis is a zoonotic infection caused by Toxoplasma gondii, an apicomplexan protozoan. It infects all warm-blooded animals, including humans. Cats are the only definitive hosts, shedding oocysts in their feces.

Transmission

•	Ingestion: of sporulated oocysts from contaminated environment or tissue cysts in undercooked meat.
•	Vertical Transmission: Transplacental transfer from mother to fetus.

Pathophysiology

•	Stages of Infection:
•	Tachyzoites: Rapidly multiplying form causing tissue damage.
•	Bradyzoites: Slow division form in tissue cysts.
•	Sporozoites: Infectious form in mature oocysts.
•	Replication: After ingestion, the parasite invades intestinal epithelium, replicates, and disseminates via blood and lymph, causing tissue necrosis.

Clinical Findings

•	General Signs: Often asymptomatic in immunocompetent hosts.
•	Acute Infection: Fever, diarrhea, cough, dyspnea, icterus, seizures, and death in young or immunocompromised animals.
•	Reproductive Issues: Abortion, stillbirth in sheep, goats, and pigs.
•	Chronic Infection: Subclinical with bradyzoites in tissue cysts.

Diagnosis

•	Serologic Testing: Indirect hemagglutination, ELISA, and other serological assays to detect IgM and IgG antibodies.
•	Histology and PCR: Identification of tachyzoites or bradyzoites in tissues; PCR for definitive diagnosis.

Treatment

•	Anticoccidial Drugs: Sulfadiazine (15–25 mg/kg PO every 12 hours) and pyrimethamine (0.44 mg/kg PO every 12 hours) for 4 weeks.
•	Combination Therapy: Trimethoprim-sulfamethoxazole (15 mg/kg PO every 12 hours) for 4 weeks.
•	Clindamycin: Preferred treatment for dogs and cats (10–12.5 mg/kg PO for dogs, 25–50 mg/kg PO for cats every 12 hours for 3-4 weeks).
•	Other Options: Diaminodiphenylsulfone, atovaquone, spiramycin, toltrazuril, ponazuril, and diclazuril for acute infections and to reduce oocyst shedding in cats.

Prevention and Zoonotic Risk

•	Hygiene: Wash hands thoroughly after handling raw meat or contaminated objects. Clean and disinfect surfaces.
•	Food Safety: Cook meat to 67°C (152.6°F), avoid raw or undercooked meat, and ensure good kitchen hygiene.
•	Pet Care: Feed cats commercially prepared food, clean litter boxes daily, and avoid exposure of pregnant women to cat feces.

https://zukureview.com/zuku-qod/navle/1693?chosen=880IhrDfWfwmivh/GPpo3Q9kdgXg86Zr7ZxFWq4pUeE%3D&utm_source=Zukureview+Subscribers&utm_campaign=b41fc05a7a-EMAIL_CAMPAIGN_2024_04_15_01_52&utm_medium=email&utm_term=0_1c9568dbdc-b41fc05a7a-%5BLIST_EMAIL_ID%5D

How well did you know this?
1
Not at all
2
3
4
5
Perfectly
27
Q

A six-year-old female spayed cocker spaniel is presented with a two-day history of lethargy. Upon physical exam, mucosal petechiae and ecchymoses and an ocular hemorrhage in the right eye are noted.
Prothrombin time (PT) and activated partial thromboplastin time (aPTT) testing are normal. A buccal mucosal bleeding time (BMBT) is performed and is prolonged. Click the labwork icon to review hematology, blood chemistry, and urinalysis test results.
Which one of the following diseases is the most likely diagnosis?

A - Disseminated intravascular coagulation (DIC)
B - Anticoagulant rodenticide toxicity
C - Hepatic insufficiency
D - Von Willebrand’s disease
E - Immune-mediated thrombocytopenia

A

Answer: Immune-mediated thrombocytopenia

This is immune-mediated thrombocytopenia, the most common cause of spontaneous bleeding in dogs. It may be primary (idiopathic, most common) or secondary to infection, neoplasia, or certain drugs.
Most common in middle-aged females; cockers, poodles, and old English sheepdogs predisposed.
Dogs with disseminated intravascular coagulation (DIC) have thrombocytopenia with a prolonged
PT and aPTT.
Follow this link to see a table of the four most important coagulation disorder patterns.

https://zukureview.com/node/100828

Immune-Mediated Thrombocytopenia (ITP) in Animals

Definition

•	ITP: Immune-mediated destruction of platelets or marrow megakaryocytes.
•	Also Known As: Idiopathic thrombocytopenic purpura.

Causes and Risk Factors

•	Idiopathic: No identifiable underlying cause.
•	Risk Factors: Middle-aged, spayed female dogs, especially Cocker Spaniels.

Pathophysiology

•	Mechanism: Immune system targets and destroys platelets or megakaryocytes.
•	Autoantibodies: Directed against platelet surface antigens.

Clinical Signs

•	Bleeding: Petechiae, ecchymoses, melena, epistaxis.
•	Severe Thrombocytopenia: Platelet counts <30,000/μL, often <10,000/μL.

Diagnosis

•	Exclusion: Rule out other causes of thrombocytopenia.
•	Tests: Bone marrow aspirate (rarely needed), platelet count, clinical signs.

Treatment

•	Corticosteroids: High-dose initial treatment, tapered gradually.
•	Vincristine: 0.01-0.02 mg/kg IV, shortens recovery time.
•	Transfusion: Fresh whole blood for severe anemia.
•	Splenectomy: For recurrent cases.
•	Monitoring: Regular platelet count checks during steroid tapering.
•	Avoid: Drugs interfering with coagulation.

Disseminated Intravascular Coagulation (DIC) in Animals - Comprehensive Study Guide

Definition

•	Disseminated Intravascular Coagulation (DIC): A secondary condition characterized by systemic activation of blood coagulation, leading to widespread clotting, bleeding, and organ damage.

Causative Agents

•	Primary Diseases: Bacterial, viral, rickettsial, protozoal, parasitic infections, heat stroke, burns, neoplasia, and severe trauma.

Pathophysiology

•	Mechanism: Systemic inflammatory response activates coagulation, leading to widespread clot formation. This results in consumption of coagulation factors and platelets, causing bleeding and organ failure due to microvascular thrombosis.

Symptoms

•	Clinical Signs: Variable, ranging from no overt signs to severe bleeding, organ failure, and microvascular thrombosis.

Diagnosis

•	Laboratory Tests: Prolonged APTT, PT, elevated D-dimer, reduced fibrinogen, and platelet count. Thromboelastography can help differentiate stages.

Treatment

•	Underlying Cause: Identify and treat the primary disease.
•	Medications: Heparin in hypercoagulable stages; fresh frozen plasma in hypocoagulable stages.
•	Supportive Care: Fluids, plasma expanders to maintain effective circulating volume.
How well did you know this?
1
Not at all
2
3
4
5
Perfectly
28
Q

Pemphigus foliaceus, pemphigus vulgaris, and bullous pemphigoid are examples of which type of immune-mediated disease?
Pemphigus foliaceus in a dog (abdomen).
A - Delayed hypersensitivity
В - Type III -  Antigen-antibody complexes deposited in tissues
C - Type IV - Antigen triggers cell-mediated cytokine release,  activates macrophages and production of cytotoxic T cells
D - Type II - Antibody binds cell antigen and activates complement
E - Type I - Immediate IGE-mediated hypersensitivity

A

Answer: Type II - Antibody binds cell antigen and activates complement.

Pemphigus foliaceus, pemphigus vulgaris, and bullous pemphigoid are generally considered type lI reactions.
Antibody binds antigen on a cell, then the antibody-antigen complex activates complement, causing cell lysis.
In general, think of rare, autoimmune skin diseases characterized by varying presentations of ulceration, crusting, pustules, vesicles.
Click here to see images of:
Moderate pemphigus foliaceus
Severe pemphigus foliaceus
Bullous pemphigoid
FYI: “Delayed hypersensitivity” is the same as a type IV reaction.

Hypersensitivity Diseases in Animals

Definitions and Terminology

•	Hypersensitivity: An exaggerated or inappropriate immune response to an antigen, causing tissue damage.
•	Types of Hypersensitivity:
•	Type I (Immediate): IgE-mediated reaction causing mast cell degranulation (e.g., anaphylaxis, atopy).
•	Type II (Cytotoxic): Antibody-mediated destruction of cells (e.g., autoimmune hemolytic anemia).
•	Type III (Immune Complex): Immune complex deposition in tissues (e.g., glomerulonephritis).
•	Type IV (Delayed-Type): T-cell-mediated response causing tissue damage (e.g., contact dermatitis).

Pathophysiology

1.	Type I (Immediate Hypersensitivity):
•	Sensitization Phase: Initial exposure to an allergen causes IgE production.
•	Effector Phase: Re-exposure to the allergen leads to IgE binding on mast cells, causing degranulation and release of histamine and other mediators.
•	Clinical Manifestations: Anaphylaxis, urticaria, atopic dermatitis.
2.	Type II (Cytotoxic Hypersensitivity):
•	Mechanism: IgG or IgM antibodies bind to antigens on cell surfaces, leading to complement activation or antibody-dependent cell-mediated cytotoxicity (ADCC).
•	Clinical Manifestations: Autoimmune hemolytic anemia, immune-mediated thrombocytopenia.
3.	Type III (Immune Complex Hypersensitivity):
•	Mechanism: Immune complexes formed in the bloodstream are deposited in tissues, causing complement activation and inflammation.
•	Clinical Manifestations: Systemic lupus erythematosus (SLE), glomerulonephritis.
4.	Type IV (Delayed-Type Hypersensitivity):
•	Mechanism: Sensitized T cells release cytokines upon antigen re-exposure, attracting macrophages and causing tissue damage.
•	Clinical Manifestations: Contact dermatitis, tuberculosis skin test reaction.

Clinical Changes and Symptoms

•	Type I:
•	Anaphylaxis: Rapid onset, difficulty breathing, collapse, hives, swelling.
•	Atopic Dermatitis: Pruritus, erythema, chronic skin infections.
•	Type II:
•	Autoimmune Hemolytic Anemia: Pale mucous membranes, jaundice, tachycardia, weakness.
•	Immune-Mediated Thrombocytopenia: Petechiae, ecchymoses, bleeding tendencies.
•	Type III:
•	Systemic Lupus Erythematosus: Polyarthritis, skin lesions, renal disease.
•	Glomerulonephritis: Proteinuria, edema, hypertension.
•	Type IV:
•	Contact Dermatitis: Red, itchy, and inflamed skin at the site of contact.
•	Tuberculosis Skin Test Reaction: Local swelling and induration at the test site.

Diagnosis

1.	Clinical Examination:
•	History: Detailed history of clinical signs, potential allergens, and previous reactions.
•	Physical Examination: Identifying characteristic signs based on the type of hypersensitivity.
2.	Laboratory Tests:
•	CBC and Serum Biochemistry: Assessing overall health, anemia, thrombocytopenia, renal function.
•	Allergy Testing: Intradermal skin tests or serum IgE testing for Type I hypersensitivity.
•	Coombs Test: Detects antibodies against red blood cells for Type II hypersensitivity.
•	Immunofluorescence: Identifies immune complexes in tissues for Type III hypersensitivity.
•	Patch Testing: Identifies contact allergens for Type IV hypersensitivity.
3.	Imaging Studies:
•	Radiography and Ultrasound: Assessing organ involvement in systemic hypersensitivity reactions.

Management Strategies

1.	Type I (Immediate Hypersensitivity):
•	Anaphylaxis: Immediate administration of epinephrine, antihistamines, and corticosteroids. Supportive care includes oxygen therapy and IV fluids.
•	Atopic Dermatitis: Allergen avoidance, antihistamines, corticosteroids, immunotherapy (allergen-specific desensitization), and anti-inflammatory medications.
2.	Type II (Cytotoxic Hypersensitivity):
•	Autoimmune Hemolytic Anemia: Immunosuppressive therapy (corticosteroids, azathioprine), blood transfusions if needed.
•	Immune-Mediated Thrombocytopenia: Immunosuppressive therapy, platelet transfusions in severe cases.
3.	Type III (Immune Complex Hypersensitivity):
•	Systemic Lupus Erythematosus: Immunosuppressive therapy, NSAIDs for pain and inflammation, supportive care for organ involvement.
•	Glomerulonephritis: Immunosuppressive therapy, management of hypertension and proteinuria, renal support.
4.	Type IV (Delayed-Type Hypersensitivity):
•	Contact Dermatitis: Allergen avoidance, topical corticosteroids, and systemic corticosteroids for severe reactions.
•	Tuberculosis Skin Test Reaction: Monitoring and management based on underlying TB infection.

Medications

•	Antihistamines:
•	Diphenhydramine: 1-2 mg/kg orally or intramuscularly every 8-12 hours.
•	Corticosteroids:
•	Prednisolone: 0.5-2 mg/kg orally or intravenously once daily for immunosuppression.
•	Epinephrine:
•	Anaphylaxis: 0.01 mg/kg intramuscularly or intravenously, repeat as needed.
•	Immunosuppressive Drugs:
•	Azathioprine: 2 mg/kg orally once daily.
•	Cyclosporine: 5-10 mg/kg orally once daily for atopic dermatitis.
•	NSAIDs:
•	Carprofen: 2.2 mg/kg orally once daily for inflammation and pain.

Prognosis

•	Type I Hypersensitivity: Good with prompt treatment, but recurrent exposure to allergens can cause repeated reactions.
•	Type II Hypersensitivity: Variable; early and aggressive treatment improves outcomes.
•	Type III Hypersensitivity: Dependent on the extent of organ involvement and response to immunosuppressive therapy.
•	Type IV Hypersensitivity: Good with allergen avoidance and appropriate treatment.
How well did you know this?
1
Not at all
2
3
4
5
Perfectly
29
Q

Which of the following crystalloid fluids is hypertonic?
A - Lactated Ringer’s solution
B - 0.45% saline in 2.5% dextrose
C - 7% saline
D - 5% Dextrose
E - Normosol@R

A

Answer: 7% Saline

7% saline is a hypertonic solution. Use it for fluid resuscitation in patients with shock.
Hypertonic saline solution (HSS) contains 1283 mEq/L of sodium and chloride compared to 154 mEq/L in normal saline (0.9%). The osmolality of HSS is 2567 milliosmoles/L, compared to 300 mOsm/L of plasma.
HSS is effective at small doses, typically 1-4 ml/kg over 15-20 mins. This is 1-20 mL for a 10 lb cat,
23-92 mL for a 50 lb dog and 1-2 L for a 1100 lb horse.

Fluid Resuscitation Plan in Animals

1.	Concepts:
•	Fluid Resuscitation: Restoring blood volume and correcting electrolyte imbalances in animals.
•	Physiological/Anatomical Considerations: Fluid compartments (intravascular, interstitial, intracellular) and maintaining perfusion.
2.	Steps of Resuscitation:
•	Assess Perfusion Parameters: Heart rate, capillary refill time, mucous membrane color.
•	Calculate Deficits: Estimate dehydration percentage, calculate fluid deficits.
•	Fluid Administration: Bolus or continuous rate infusion depending on severity.
•	Reassessment: Continuous monitoring of clinical parameters.
3.	Determining Deficits:
•	Physical Examination: Skin turgor, mucous membrane moisture, body weight changes.
•	Laboratory Tests: Packed cell volume (PCV), total protein, serum electrolytes.
4.	Selecting Solutions:
•	Crystalloids:
•	Isotonic: 0.9% saline, Lactated Ringer’s Solution (LRS) for general fluid replacement.
•	Hypotonic: 5% dextrose in water (D5W) for free water replacement.
•	Hypertonic: 7% NaCl for rapid intravascular volume expansion.
•	Colloids:
•	Synthetic: Hydroxyethyl starch (HES), Dextrans for maintaining intravascular volume.
•	Natural: Plasma, albumin for volume expansion and protein replacement.
5.	Characteristics of Solutions:
•	Crystalloids: Rapidly distribute across extracellular space, short-term volume expansion.
•	Colloids: Stay within intravascular space longer, provide longer-term volume support.
6.	Electrolytes:
•	Sodium (Na+): Essential for osmolality and volume status, monitor to prevent hypernatremia or hyponatremia.
•	Potassium (K+): Vital for cellular function, supplement cautiously to avoid hyperkalemia.
7.	Types of Colloids:
•	Synthetic Colloids: HES, dextrans used for hypoproteinemia, shock.
•	Natural Colloids: Plasma, albumin for hypoproteinemia, coagulopathies.
8.	Fluid Selection Criteria:
•	Patient Condition: Dehydration, hypovolemia, shock.
•	Electrolyte Imbalance: Specific deficits or excesses.
9.	Determining End Points:
•	Clinical Signs: Normalization of heart rate, blood pressure, urine output, mucous membrane color.
•	Laboratory Values: Normalization of PCV, total protein, serum electrolytes.
10.	Calculating Resuscitation:
•	Dehydration Calculation: Body weight (kg) x % dehydration = liters of deficit.
•	Shock Doses: Dogs: 90 ml/kg, Cats: 60 ml/kg (initial bolus of 20-30 ml/kg).
•	Maintenance Fluids: 40-60 ml/kg/day.
How well did you know this?
1
Not at all
2
3
4
5
Perfectly
30
Q

Which species uses sweating as the primary method of cooling?

A - Sheep
B - Horse
C - Pig
D - Cat
E - Dog

A

Answer: Horse.

The horse is the only domestic species to sweat as a primary method of heat dissipation. Higher primates (i.e., monkeys, apes, and humans) also cool off by sweating.
The other species do not sweat and use different methods to cool down. Pigs wallow in mud to cool by evaporation. Cats and dogs sweat a small amount through their paws, but dogs predominantly pant to cool off.
Cattle and sheep do not sweat efficiently and rely on respiration to cool themselves.

How well did you know this?
1
Not at all
2
3
4
5
Perfectly
31
Q

Which one of the following choices can be an underlying cause for the problem seen in this turtle?

A - Mycoplasmosis
B - Vitamin A deficiency
C - Septicemic cutaneous ulcerative disease (SCUD)
D - Pasteurella multocida
E - Nutritional secondary hyperparathyroidism

A

Answer: Vitamin A deficiency

This is an aural abscess, which can occur secondary to vitamin A deficiency.
Captive terrestrial box turtles are most at risk, usually due to diets containing little vitamin A.
Other presentations of hypovitaminosis A include froth from the nose (a sign of respiratory disease)
and renal disease.
For information on wild turtle conservation (and stunning photography), have a look at the Turtle Conservancy.

https://www.turtleconservancy.org/news/

Bacterial Diseases of Reptiles

Definitions and Causative Agents

•	Septicemia: Often caused by Aeromonas and Pseudomonas spp.. Common in reptiles, leading to systemic infections.
•	SCUD: Septicemic Cutaneous Ulcerative Disease primarily affects aquatic turtles, often caused by Citrobacter freundii.
•	Ulcerative Dermatitis: Also known as scale rot, common in snakes and lizards.
•	Abscesses: Frequently caused by traumatic injuries and can involve various bacteria.
•	Infectious Stomatitis: “Mouth rot” seen in snakes, lizards, and turtles.
•	Pneumonia: Respiratory infections, often involving mixed bacterial populations.
•	Mycoplasmosis: Notably affects chelonians and crocodilians.
•	Otitis: Ear infections, particularly common in turtles.
•	Cloacitis: Infection of the cloaca, often traumatic.
•	Spinal Osteopathy/Osteomyelitis: Chronic bacterial osteomyelitis in reptiles, especially snakes.
•	Mycobacteriosis: Chronic infections with mycobacterial species.
•	Salmonella enterica Infection: Common in reptiles, often asymptomatic carriers.

Pathophysiology and Transmission

•	Septicemia: Bacteria enter the bloodstream, leading to widespread infection.
•	SCUD: Bacteria invade through the skin or shell lesions, leading to systemic spread.
•	Ulcerative Dermatitis: Infections often start with minor skin lesions, exacerbated by poor environmental conditions.
•	Abscesses: Localized infections that can spread systemically if not treated.
•	Infectious Stomatitis: Starts with oral lesions, can progress to systemic infection.
•	Pneumonia: Often linked to poor husbandry conditions, including temperature and humidity imbalances.
•	Mycoplasmosis: Chronic respiratory disease in reptiles, often leading to systemic infection.
•	Otitis: Inflammation and infection of the ear, often related to vitamin deficiencies.
•	Cloacitis: Infections in the cloacal area, often related to trauma or parasites.
•	Spinal Osteopathy: Chronic infection leading to bone deformities.
•	Mycobacteriosis: Chronic granulomatous disease affecting various organs.
•	Salmonella: Common commensal organism, zoonotic potential.

Clinical Signs

•	Septicemia: Respiratory distress, lethargy, petechiae.
•	SCUD: Pitted scutes, anorexia, lethargy, liver necrosis.
•	Ulcerative Dermatitis: Erythema, necrosis, ulceration.
•	Abscesses: Nodules or swellings, often on the face or limbs.
•	Infectious Stomatitis: Oral petechiae, caseous material along dental arcades.
•	Pneumonia: Open-mouth breathing, nasal discharge, dyspnea.
•	Mycoplasmosis: Rhinitis, upper respiratory tract disease.
•	Otitis: Swelling at the tympanic membrane, caseous material.
•	Cloacitis: Edema, hemopurulent discharge.
•	Spinal Osteopathy: Progressive spinal lesions.
•	Mycobacteriosis: Granulomas, often with chronic wasting.
•	Salmonella: Often asymptomatic but can cause enteritis.

Diagnosis

•	Septicemia: Blood culture, cytology, and histopathology.
•	SCUD: Culture from lesions, biopsy.
•	Ulcerative Dermatitis: Clinical signs, culture, and sensitivity tests.
•	Abscesses: Fine needle aspiration, culture.
•	Infectious Stomatitis: Clinical examination, culture.
•	Pneumonia: Radiographs, culture from respiratory secretions.
•	Mycoplasmosis: PCR, serology.
•	Otitis: Clinical examination, culture.
•	Cloacitis: Radiographs, culture, and sensitivity tests.
•	Spinal Osteopathy: Biopsy, radiographs.
•	Mycobacteriosis: Culture, histopathology.
•	Salmonella: Fecal culture.

Treatment

•	Septicemia: Systemic antibiotics, supportive care.
•	SCUD: Systemic antibiotics, wound debridement, sanitation.
•	Ulcerative Dermatitis: Systemic and topical antibiotics, debridement.
•	Abscesses: Surgical excision, local treatment.
•	Infectious Stomatitis: Surgical debridement, systemic antibiotics.
•	Pneumonia: Antibiotics, supportive care, nebulization.
•	Mycoplasmosis: Tetracyclines, macrolides.
•	Otitis: Surgical drainage, antiseptic lavage.
•	Cloacitis: Surgical debridement, systemic antibiotics.
•	Spinal Osteopathy: Long-term antibiotics, supportive care.
•	Mycobacteriosis: Often euthanasia due to poor prognosis.
•	Salmonella: Management of zoonotic risk, often asymptomatic.

Prevention and Management

•	Husbandry: Optimal environmental conditions, proper nutrition.
•	Sanitation: Clean and disinfect habitats regularly.
•	Isolation: Separate infected individuals to prevent the spread.
•	Biosecurity: Prevent introduction of new pathogens.
•	Public Health: Manage zoonotic risks, particularly with Salmonella.
How well did you know this?
1
Not at all
2
3
4
5
Perfectly
32
Q

A large herd of finishing swine is evaluated due to reduced weight gain. On examination many pigs have dry coughs which are more apparent when the animals are moving around.
A group of pigs recently sent to slaughter from this farm had a high incidence of pneumonic lesions ventrally in the cardiac and apical pulmonary lobes.
What is the most likely cause of pneumonia in these pigs?

A - Swine influenza virus
B - PRRS virus
C - Mycoplasma hyopneumoniae
D - Mannheimia hemolytica
E - Glaesserella parasuis

A

Answer: Mycoplasma hyopneumoniae

This is the smoldering low-level illness and cranioventral consolidation of Mycoplasma hyopneumoniae, also called “enzootic pneumonia.” Follow this link to see lung tip atelectasis (dark, collapsed areas on tips - you can’t breathe with lungs like that).
M. hyopneumoniae causes mild endemic respiratory disease in pigs characterized by a dry cough and reduced weight gain. Stressors (parasites, other infections, even the weather) can result in severe pneumonia.
Swine influenza A virus varies in severity, and is characterized by rapid onset in whole herd of fever, depression, anorexia, coughing, followed by almost-as-sudden recovery.
Mannheimia hemolytica is more a sheep/goat/cow pneumonia (think shipping fever complex), but is often thought of in same risk category as Pasteurellae.
Glaesserella (formerly Haemophilus) parasuis is the cause of Glasser’s disease, seen mostly in piglets
6-8 wk old. Characterized by short course, sudden death. Clinical signs include high fever, severe coughing, abdominal breathing, swollen joints, CNS signs like lateral decubitus, paddling, trembling.
In addition to respiratory illness, look for reproductive problems (abortions, stillbirths) in sows with porcine reproductive and respiratory syndrome (PRRS).

https://zukureview.com/zuku-qod/navle/529?chosen=fFr%2BwWbNIceoDyfGzlks%2BoGUOau0yfY/9E8gmELOw/4%3D&utm_source=Zukureview+Subscribers&utm_campaign=308b54fb2d-EMAIL_CAMPAIGN_2022_12_01_02_50&utm_medium=email&utm_term=0_1c9568dbdc-308b54fb2d-%5BLIST_EMAIL_ID%5D

Mycoplasmal Pneumonia in Pigs - Comprehensive Information for BCSE Test

Definitions and Causative Agents:

•	Mycoplasmal Pneumonia: Chronic respiratory disease caused by Mycoplasma hyopneumoniae.
•	Causative Agents: Mycoplasma hyopneumoniae, often complicated by other mycoplasmas, bacteria, and viruses.

Epidemiology:

•	Transmission: Aerosol, direct contact, from dam to piglets.
•	Age Susceptibility: Pigs of all ages, highest prevalence in pigs 3-5 months old.

Clinical Findings:

•	Signs: Persistent dry cough, reduced growth rate, sporadic severe pneumonia.
•	Lesions: Gray or purple, consolidated lung areas, primarily in apical and cardiac lobes, histological lymphoid hyperplasia.

Diagnosis:

•	Methods: Clinical signs, histopathology, PCR, serologic tests.
•	Samples: Tracheal, laryngeal, bronchial swabs, lung tissue.

Treatment:

•	Antibiotics: Effective against Mycoplasma spp (e.g., tetracyclines, macrolides).
•	Supportive Care: Improved ventilation, reduced stress.

Control:

•	Vaccination: Reduces clinical signs, does not prevent infection.
•	Management: All-in/all-out practices, gilt acclimation.
How well did you know this?
1
Not at all
2
3
4
5
Perfectly
33
Q

A sow farm is seeing a sudden onset of diarrhea followed by collapse and death in piglets one to five days of age in the farrowing rooms. Necropsy on one of the piglets finds the lesions shown below.
What should be done next?

A - Prevent neonates from drinking sow’s colostrum
B - Treat affected piglets with antibiotics
C - Tilmicosin IM, all piglets, isolate sick ones
D - Vaccinate dams in middle third of pregnancy with bacterin
E - Treat newborn piglets at birth with type C antitoxin

A

Answer: Treat newborns piglets at birth with type C antitoxin

In acute outbreak of Clostridium perfringens type C, prophylactic type C antitoxin and/or antibiotic is protective if given to piglets within 2 hours of birth.

The necropsy identified hemorrhagic intestines characteristic of the disease. Affects piglets up to 3 weeks old with a severe hemorrhagic necrotizing diarrhea and high mortality.

Treatment of sick piglets is rarely efficacious.

Best prevention in newborns is to vaccinate the pregnant dam in last third of pregnancy with the appropriate toxoid. Important to assure newborn gets sufficient colostrum.

Clostridium perfringens Type C Enteritis in Pigs - Comprehensive Veterinary Information

Definitions and Terminology:

•	Clostridium perfringens Type C Enteritis: A highly fatal necrohemorrhagic enteritis in piglets.

Causative Agent:

•	Pathogen: Clostridium perfringens type C.

Physiopathology:

•	Transmission: Fecal-oral route; spores ingested from contaminated environment.
•	Pathogenesis: Beta toxin causes necrosis of intestinal villi, leading to hemorrhage and necrosis of the mucosa.

Clinical Findings:

•	Symptoms: Sudden onset of hemorrhagic diarrhea, collapse, and death in piglets 1-3 days old. Brownish liquid feces, pasty-gray diarrhea, progressive emaciation in subacute cases.
•	Lesions: Dark red, hemorrhagic intestines, necrotic membrane in chronic cases.

Diagnosis:

•	Clinical Signs: Hemorrhagic diarrhea, sudden death.
•	Lab Tests: Detection of beta toxin in feces, culture, genotyping.
•	Postmortem Examination: Histopathology shows segmental hemorrhagic necrosis and presence of large gram-positive rods.

Treatment and Control:

•	Treatment: Limited value after onset; prophylactic antitoxin or antimicrobials if given early.
•	Prevention: Vaccination of sows with type C bacterin-toxoid during gestation.
How well did you know this?
1
Not at all
2
3
4
5
Perfectly
34
Q

A 12-year-old Belgian draft horse gelding is presented with severe hyperkeratosis and scaling on the caudal aspects of all four pasterns. The parasite shown below is found on a skin scraping of one of the lesions.
Which one of the following choices is the correct diagnosis?

A - Sarcoptic dermatitis
B - Trombiculiasis
C - Cheyletiellosis
D - Chorioptic mange
E - Demodecosis

A

Answer: Chorioptic mange

This is chorioptic mange - the most common mite in horses. Chorioptes equi is characterized by long legs and short pedicles.

Most commonly it is found on the lower limbs (esp. the hind legs), where it causes pruritus.
Chorioptes infestation is often seen as a component of pastern dermatitis, a multi-factorial disease that is very common in draft horses, especially those breeds with heavy feathering. In non-feathered breeds, it can occur elsewhere on the body.

It is also called scratches, greasy heel, and dermatitis verrucosa. It is a surface mite and causes extreme pruritus. It is very contagious horse-to-horse (occasionally on fomites) so if one in a herd has it, the others probably do as well.

Tx: Ideal to remove the feathers (clients rarely allow this) then use some combination of topical permethrin or coumaphos, systemic or topical ivermectin/moxidectin, selenium sulphide shampoos, or topical products w/ fpronil (usually used in small animal).

Here’s a great review of chorioptic mange, with images, from the University of Liverpool.

https://www.liverpool.ac.uk/media/livacuk/equine/equinepractice/Chorioptic,Mites.pdf

Chorioptic Mange in Horses
Chorioptic mange is caused by infestation with Chorioptes bovis (formerly (equi) and is the most common form of mange in horses. Draft horses are commonly infested, although all breeds are susceptible.
Lesions caused by C bovis start as a pruritic dermatitis affecting the distal limbs around the foot and fetlock. Papules are evident first, followed by alopecia, crusting, and thickening of the skin. A moist dermatitis of the fetlock develops in chronic cases. Infested horses may stamp their feet or rub one foot against the opposite leg or object. Chorioptic mange is a differential diagnosis for “greasy heel” in draft horses.
Clinical signs subside in summer but recur with the return of cold weather. The disease course is usually chronic without treatment; however, the prognosis is favorable when treated. Topical and oral treatments recommended for other types of mange are effective.
Hot lime sulfur is labeled for use against Chorioptes in horses. Treatment should be repeated every 12 days if needed, following the species-specific dilution on the label. Treatment is aided by clipping long hair from infested areas.
Although not labeled for treatment of mange in horses, oral ivermectin at 200 mcg/kg administered for two doses 14 days apart (field studies), or a single treatment with oral moxidectin at 400 mcg/kg, has effectively treated psoroptic, chorioptic, and sarcoptic mange in horses.

How well did you know this?
1
Not at all
2
3
4
5
Perfectly
35
Q

The calf shown below was presented with ataxia, intention tremors, and hypermetria; all signs were present since birth. The second image shows a normal bovine calf brain on the left and the brain from this calf on the right. Which one of the following choices is the most likely cause of the disorder in this calf?

A - Copper deficiency
B - Organophosphate toxicity
C - Bovine viral diarrhea virus
D - Border disease
E - Caudal occipital malformation syndrome

A

Answer: Bovine Viral Diarrhea Virus

Bovine viral diarrhea virus (BVD).
In utero infection with BVD is a common cause of cerebellar hypoplasia in cattle.
Other causes are bluetongue, Akabane, or Wesselsbron viruses.
Caudal occipital malformation syndrome is a disorder seen mainly in Cavalier King Charles Spaniels.
The malformed occipital bone causes crowding at the caudal fossa and cerebellar herniation at the foramen magnum.

Bovine Viral Diarrhea (BVD) is a significant viral disease in cattle caused by the Bovine Viral Diarrhea Virus (BVDV), a pestivirus. It has two biotypes: noncytopathic (NCP) and cytopathic (CP). BVD can lead to a wide range of symptoms from mild to severe, including diarrhea, respiratory issues, reproductive failure, immunosuppression, and mucosal disease. Persistent infection can occur if a fetus is infected before the immune system matures. Control measures include vaccination, biosecurity, and managing persistently infected animals.

How well did you know this?
1
Not at all
2
3
4
5
Perfectly
36
Q

A 12 week old chicken from a young backyard flock is presented for evaluation because of weight loss, decreased appetite, and diarrhea.
The irises in the affected bird are lighter than normal with irregular pupillary margins, and the hen does not have a normal pupillary light reflex (bottom chicken in image). Several other young birds appear to be growing poorly compared to the rest of the flock.
Based on the presumptive diagnosis, how can this problem be prevented going forward?

A - Buy only vaccinated chicks
B - Cull birds from the same genetic line
C - Collect pharyngeal swabs from affected birds;   Antibiotic Tx for entire flock based on culture and sensitivity
D - Isolate unaffected animals; Breed from survivors of disease
E - Use only mycoplasma-resistant bloodlines for new additions to flock

A

Answer: Buy only vaccinated chicks

Buy only chicks vaccinated against Marek disease. This chicken has ocular evidence of Marek disease. There is no treatment, but the problem is preventable when chicks are vaccinated at hatching or before 2 weeks of age.
Diffuse infiltration of mononuclear cells can turn the irises of affected chickens from a normal yellow color to a pale tan to gray color (a.k.a. “gray eye”). Cellular infiltration can also occur in many other tissues (skin, nerves, viscera), and there may be palpable tumors in the muscle and skin. ‘
Marek disease is common in commercial poultry. Dx usually based on enlarged nerves and lymphoid tumors in the viscera. Lymphoid leukosis is a key differential.
Absence of bursal tumors helps distinguish Marek disease from leukosis. But if you find bursal tumors, Marek disease is still on the DDX list pending other tests, like immunochemistry. Chicks as young as 3 weeks can show Marek disease, but lymphoid leukosis is typically seen in chickens more than 14 weeks old.

Marek’s Disease is a highly contagious viral disease affecting chickens, caused by an alphaherpesvirus. It primarily targets the nervous system and can cause tumors in various organs. Clinical signs include paralysis, weight loss, and immunosuppression, which predisposes birds to secondary infections. The disease is often fatal, especially in young birds. Diagnosis is typically based on clinical signs and post-mortem examination. Control measures include vaccination, which is effective in preventing disease but does not prevent infection or virus shedding.

How well did you know this?
1
Not at all
2
3
4
5
Perfectly
37
Q

A 12-year-old male neutered West Highland White terrier is presented after he disappeared from home for six hours and returned with a left hind limb lameness.
Pelvic radiographs taken on presentation are shown below.
What is the most appropriate initial treatment?

A - Femoral head and neck excision
B - Non-steroidal anti-inflammatory medication and rest
C - Need orthogonal radiographic view
D - Tibial tubercle transposition and
Robert-Jones bandage
E - Triple pelvic osteotomy

A

Answer: C - Need orthogonal radiographic view

An orthogonal view is necessary to determine the next best step because it is not possible to determine if this hip luxation is dorsal or ventral with only the VD.
Conservatively treat craniodorsal coxofemoral (hip) luxations (most common) with closed reduction and an Ehmer sling. Typically presents as a non-weight-bearing lameness, usually after blunt force trauma, (e.g., being hit by a car).
Occasionally ventral luxations occur and an Ehmer sling predisposes these to re-luxation, as it positions the limb in an abducted, externally rotated position. Instead, use hobbles to prevent abduction and re-luxation.
In this case closed reduction is appropriate as the injury is recent (less than 24-48 hr since it occurred). Additionally, the hip anatomy is normal, with no signs of avulsion fragments, hip dysplasia, or osteoarthritis.
Closed reduction is associated with a re-luxation rate of ~50%, treated with open reduction and stabilization: e.g., toggle pinning, iliofemoral suture, capsulorrhaphy, trochanteric transposition, and prosthetic joint capsule.
Risk of re-luxation after closed reduction is higher with chronic injuries or dysplastic hips, and these patients are at risk of progressive arthritis. Consider salvage options such as femoral head osteotomy or total hip replacement.
Ehmer slings have a high risk of bandage complications. Carefully monitor post-reduction for sling-related injuries and instruct owners on how to monitor.

https://pubmed.ncbi.nlm.nih.gov/31149876/

Here is some useful information from the University of Illinois about closed reduction of hip luxations, and here is more helpful information from the American College of Veterinary Surgeons.

https://vetmed.illinois.edu/2021/04/13/coxofemoral-luxation-tips-for-closed-reductions/

https://www.acvs.org/small-animal/hip-luxation/

Hip Luxation
Traumatic dislocation of the hip is most frequently a craniodorsal displacement of the femoral head relative to the acetabulum.
Clinical signs include lameness, pain during manipulation of the hip joint, and a shortened limb due to dorsal displacement of the femur. Radiography is useful in confirming the luxation and delineating the presence of other fractures in the femoral head or acetabulum. Treatment involves either closed manipulation and postoperative slings to maintain the reduction or open surgical stabilization using sutures or toggle pins. Femoral head and neck resection or total hip replacement can be performed after failed reductions. Prognosis for recovery is usually excellent.

How well did you know this?
1
Not at all
2
3
4
5
Perfectly
38
Q

Which of the following nerves are targeted with a paravertebral block used to perform a standing laparotomy in a cow?

A - L1-3, and S1-5
B - L1, L2, and L4
C - L2, L3, S1, and S2
D - L1, L2, L3
E - T13, L1, and L2

A

Answer: T13, L1and L2

The spinal nerves, T13, L1, and L2 must be blocked to completely desensitize the flank of a cow.
The paravertebral (PV) nerve block targets these nerves. It can be performed via two techniques â?? the proximal or distal PV block.

https://www.merckvetmanual.com/management-and-nutrition/pain-assessment-and-management/local-and-regional-analgesic-techniques-in-animals?autoredirectid=20983

The proximal block places local anesthetic in the space just caudal to the transverse processes of the vertebrae - T13, L1, and L2.
The distal block is placed at the ends of the transverse processes of the vertebrae â?? L1, L2, and L4 as the nerves gradually course caudally after they exit the spinal foramen.

Proper placement of the anesthetic results in warming of the skin from vasodilation, anesthesia of the skin and body wall, and a curvature of the spine in some cows.

The latter is caused by relaxation of the epaxial musculature on the affected side; the spine curves in a convex manner.

Click here for a useful review of Local Anesthesia and Analgesia by Lyon Lee DVM, PhD. pp. 12-14.

https://www.westernu.edu/mediafiles/veterinary/vet-anesthesia-analgesia/local-anesthesia-analgesia.pdf

How well did you know this?
1
Not at all
2
3
4
5
Perfectly
39
Q

A three-day-old female alpaca (cria) is presented in respiratory distress.
The cria’s cheeks flare noticeably during inspiration, and the distress is more pronounced during nursing, when the animal gasps and inhales milk.
What is the most likely diagnosis?
A - Choanal atresia
B - Wry face
C - Mitral stenosis
D - Lung lobe torsion
E - Diaphragmatic hernia

A

Answer: Choanal atresia

Choanal atresia is one of the most common congenital problems of South American camelids.
Choanal atresia occurs when the caudal nares (choanae) fail to open during embryologic development.
Can be unilateral or bilateral and may cause complete or partial blockage.

https://www.merckvetmanual.com/exotic-and-laboratory-animals/llamas-and-alpacas/diseases-of-llamas-and-alpacas

How well did you know this?
1
Not at all
2
3
4
5
Perfectly
40
Q

A four-year-old stallion is presented with urine scald, fecal retention, tailhead rubbing, and a right head tilt.
Rectal examination reveals atonia of the anus and rectum, however, no fracture is palpated.
Which one of the following tests help provide evidence of the top differential diagnosis?
A - Tibial muscle and nerve biopsies
B - Antibodies against P2-myelin protein
C - Immunoblot against DNA fragments from P. tenuis
D - Cervicothoracic spinal radiography
E - Search the pasture for yellow star thistle

A

Answer: Antibodies against P2-myelin protein.

Antibodies against P2-myelin protein.
The top differential for this horse with cauda equina signs PLUS cranial nerve signs is polyneuritis equi. Horses with this condition can have circulating antibodies against P2-myelin protein, but many false positives.
Other diseases to rule out in this case would be equine herpesvirus-1 and equine protozoal encephalomyelitis.
Unfortunately there is no definitive Tx and prognosis is poor for functional recovery.

Polyneuritis Equi
Neuritis of the cauda equina (polyneuritis equi) is characterized by inflammation of the sacrocaudal nerves and occasionally other nerves. It is seen in adult horses of all breeds in Europe and North America. The cause is unknown, although an immunologic reaction incited by a viral infection is possible. Affected horses have circulating antibodies against P2 myelin protein.
The most consistent clinical signs reflect involvement of the sacrocaudal nerves and include urinary and fecal incontinence, tail paralysis, perineal paresthesia or analgesia, atrophy of the gluteal muscles, mild pelvic limb ataxia, and, in male horses, penile paralysis. Affected horses may rub the tail. The thoracic limbs and cranial nerves may also be affected. Diagnosis can usually be based on clinical findings. CSF may be xanthochromic, with increased protein content and mononuclear pleocytosis. Sacral fracture should be excluded by rectal examination and radiography.
There is no treatment, and the prognosis for recovery is poor. Histopathologically, there is granulomatous inflammation primarily affecting the extradural portions of the sacrocaudal nerves.

https://www.merckvetmanual.com/respiratory-system/respiratory-diseases-of-horses/equine-herpesvirus-infection

https://www.merckvetmanual.com/horse-owners/brain,-spinal-cord,-and-nerve-disorders-of-horses/equine-protozoal-myeloencephalitis

How well did you know this?
1
Not at all
2
3
4
5
Perfectly
41
Q

A milk sample is sent into the clinic for culture from a third lactation Jersey cow with a history of decreased appetite, drop in milk production, and temperature of 103.5°F (39.7°C) [Normal = 100.4-102.8ºF (38.0?39.3°C)].
There is growth on both the blood agar and Macconkey culture plates. Growth on the Macconkey looks like the image below.
What is the best treatment recommendation for this cow?

A - Supportive therapy.
B - Iodine teat dip
C - Intramammary_pirlimycin
D - Vaccinate with bacterin
E - Topical insecticides

A

Answer: Supportive therapy

Supportive care is the first and best treatment for severe coliform mastitis (i.e., E.coli, Klebsiella, Enterobacter spp.). Lipopolysaccharide (LPS) endotoxins released from the bacteria trigger immune-related sepsis. Treat with IV fluids and oral calcium. NSAIDs administered early in the course of disease can decrease the severity of clinical signs and improve treatment outcomes.
The subsequent use of antimicrobial therapy in severe coliform mastitis cases is controversial, but about half of endotoxic cases will also be bacteremic. Some studies suggest improved patient outcomes with IV oxytetracycline or IM cephalosporins. Both are extra-label drug use and require Food and Drug Avoidance Residue Databank (FARAD) approved drug withdrawal times in the US.
Pirlimycin is a lincosamide that targets gram-positive Staph spp. and Strep spp. and would not be indicated in this case.
Prevent new cases of environmental mastitis pathogens with antiseptic teat dip before each milking, good bedding management, and fly control. Vaccinate dry cows with bacterin toxoid to aid in prevention of coliform mastitis and endotoxemia.
Expect 1-2 cases of clinical mastitis per 100 cows/month on well managed herds.

Coliform Mastitis in Cattle: Comprehensive Veterinary Guide

Definition:
Coliform mastitis is a form of mastitis in dairy cattle caused by gram-negative, lactose-fermenting bacteria, primarily Escherichia coli, Klebsiella spp., and Enterobacter spp.

Pathophysiology:

•	Invasion: Bacteria enter through the teat canal, proliferate in the mammary gland, and release endotoxins.
•	Immune Response: Endotoxins trigger an acute inflammatory response, characterized by neutrophil influx and cytokine release.
•	Systemic Effects: Severe cases can lead to systemic toxemia, including fever, shock, and even death.

Causes:

•	Environmental Factors: Contaminated bedding, water, and equipment; poor milking hygiene; and trauma to the teats.
•	Specific Pathogens: E. coli and Klebsiella are common environmental pathogens associated with coliform mastitis.

Symptoms:

•	Local: Swollen, firm udder, with watery, bloody, or pus-laden milk.
•	Systemic: Fever, anorexia, dehydration, and in severe cases, shock.

Clinical Changes:

•	Milk Changes: Reduced milk yield, abnormal milk appearance (watery or containing clots).
•	Udder Changes: Swelling, heat, and pain in the affected quarter(s).

Assessment:

•	Clinical Signs: Observation of milk and udder condition, systemic symptoms.
•	Laboratory Tests: Somatic cell count (SCC) in milk, bacterial culture to identify the causative agent.

Treatment:

•	Antibiotics: Systemic and intramammary antibiotics, ideally chosen based on sensitivity testing.
•	Cephalosporins: Commonly used due to efficacy against gram-negative bacteria.
•	Aminoglycosides and Fluoroquinolones: Used for severe cases but require careful monitoring for residues in milk.
•	Supportive Care: Fluid therapy to manage dehydration and shock, anti-inflammatory drugs to reduce fever and pain.

Prevention:

•	Hygiene: Proper milking procedures, clean and dry bedding, regular equipment maintenance.
•	Teat Care: Use of post-milking teat dips to reduce bacterial colonization.
•	Vaccination: Some vaccines are available for prevention against E. coli and Klebsiella spp.

Prognosis:

•	Severity: Varies based on the extent of infection and timeliness of treatment.
•	Outcomes: Potential for chronic infections, reduced milk production, and, in severe cases, death or culling.

Epidemiology:

•	Incidence: Higher in herds with poor environmental management; outbreaks can occur in warmer climates or with improper bedding materials.
•	Zoonotic Risk: Although rare, some coliform bacteria can be zoonotic, posing a risk to humans.

Key Points:

•	Early Detection: Critical for effective treatment; regular monitoring of SCC and milk quality is essential.
•	Veterinary Involvement: Regular herd health assessments and consultations for mastitis management and antibiotic stewardship.
How well did you know this?
1
Not at all
2
3
4
5
Perfectly
42
Q

An eight-year-old mixed breed female spayed dog presents with a mass on her head. A radiograph is shown below.
What are the three most likely differential diagnoses?

A - Hypertrophic osteopathy, hemangiosarcoma, meningioma
B - Multiple cartilaginous exocytosis, masticatory myositis,
rhabdomyosarcoma
C - Craniomandibular osteopathy, multiple cartilaginous exocytosis, squamous cell carcinoma
D - Multilobular osteochondrosarcoma, osteosarcoma,
chondrosarcoma
E - Osteosarcoma, fungal osteomyelitis, hypertrophic
osteodystrophy

A

Answer: Multilobular osteochondrosarcoma, osteosarcoma, chondrosarcoma

The top differential diagnoses of this skull tumor are osteosarcoma (OSA), chondrosarcoma (CSA), and multilobular osteochondrosarcoma (MLO).

https://vsso.org/skull-tumors

https://vsso.org/bone-axial-osa#general_considerations

https://vsso.org/bone-csa

https://vsso.org/bone-mlo

OSA is the most common primary bone tumor of dogs and skull OSA accounts for < 3% of all OSA.
Axial OSA carries a more favorable prognosis than appendicular OSA, and Tx consists of radiation and chemotherapy.

https://www.merckvetmanual.com/musculoskeletal-system/osteopathies-in-small-animals/osteomyelitis-in-dogs-and-cats

https://www.merckvetmanual.com/musculoskeletal-system/myopathies-in-small-animals/masticatory-myositis-in-dogs-and-cats

Chondrosarcoma is a malignant, cartilage-producing tumor, and the second-most common primary bone tumor. Unlike OSA, CSA is more common in the axial skeleton than appendicular skeleton.
Surgery is the Tx of choice for CSA, with radiation therapy recommended for unresectable tumors.
MLO presents as a firm, fixed mass arising from the periosteum. Clinical signs are referable to location. Tx is surgical with unknown effects of chemotherapy and radiation.
Osteomyelitis is infection and inflammation of bone. Masticatory myositis is immune-mediated inflammation of the muscles of mastication.
Hypertrophic osteodystrophy is a developmental disorder of the metaphysis of long bones of young, growing dogs. Craniomandibular osteopathy is a proliferative bone disorder of growing terrier dogs that affects the mandible and tympanic bullae. Multiple cartilaginous exocytosis affects young dogs and cats: see multiple ossified protuberances arising from the surfaces of long bones.
Hemangiosarcoma is a tumor arising from endothelial cells. Meningioma is a benign, space-occupying tumor arising from the dura of the brain or spinal cord. Squamous cell carcinoma is a skin tumor that can result in firm, nodular, +/- erosive mass lesions. Rhabdomyosarcoma is a malignant primary muscle tumor.
Hypertrophic osteopathy is a paraneoplastic proliferation of long bones secondary to intrathoracic or intraabdominal tumors.

https://zukureview.com/zuku-qod/navle/1842?chosen=LTO%2B3G5b006tyuXbvHfv6qpJDK%2BxzqnHE/QJZklazLvQzTfHyOpaSo/%2B3JL1ldoF2FIzgMvubO8NTjBG29T1eA%3D%3D&utm_source=Zukureview+Subscribers&utm_campaign=d2456ebbe6-EMAIL_CAMPAIGN_2024_06_20_01_54&utm_medium=email&utm_term=0_1c9568dbdc-d2456ebbe6-%5BLIST_EMAIL_ID%5D

How well did you know this?
1
Not at all
2
3
4
5
Perfectly
43
Q

A dead goat is presented to a veterinarian in the Caribbean.
The farmer reports that the goat hadn?t been eating well yesterday, then this morning was breathing hard and walking with a strange high-stepping gait.
She eventually went down and started paddling with her head twisted back just before death.
Necropsy reveals the following finding (blue arrow) adjacent to the heart:
He is worried about his other goats.
What vector is responsible for transmitting the infectious agent most likely responsible for this goat? s condition?

A - Amblyomma spp. ticks
B - Musca autumnalis (the face fly).
C - Dermacentor spp. ticks
D - Tabanids (e.g., the horse fly).
E - Damalinia caprae lice

A

Answer: Amblyoma spp ticks.

Amblyomma spp. are vectors for Ehrlichia ruminatium (formerly Cowdria ruminatium), the causative agent of heartwater disease in ruminants. The clinical progression of this case and the pericardial effusion seen on necropsy (as identified by the blue arrow) are classical for acute heartwater disease.
Animals may just be found dead with peracute disease.
Definitive diagnosis typically requires necropsy with microscopic evaluation of stained brain tissue.
This disease is endemic in parts of Africa and the Caribbean but is considered a foreign animal disease in the mainland Americas.
Dermacentor spp. are vectors for Babesia caballi in horses, Anaplasma marginale in cattle, Rickettsia rickettsii (Rocky Mountain spotted fever), Powassan virus, as well as the agents of Q fever and tularemia, among other infectious diseases throughout the world.
Face flies (Musca autumnalis) are an important vector for eyeworm (Thelazia spp.) and Mycoplasma bovis, one agent of infectious bovine keratoconjunctivitis of ruminants.
Horseflies (Tabanid spp.) can be vectors for bovine leukosis virus, the cause of enzootic bovine leukosis and lymphosarcoma.

https://zukureview.com/zuku-qod/navle/1836?chosen=0Ztdg77cp3gaVcyt47SqX1vdoRLmclo74iC42jx4KRY%3D&utm_source=Zukureview+Subscribers&utm_campaign=72802b1fd8-EMAIL_CAMPAIGN_2024_06_18_01_55&utm_medium=email&utm_term=0_1c9568dbdc-72802b1fd8-%5BLIST_EMAIL_ID%5D

How well did you know this?
1
Not at all
2
3
4
5
Perfectly
44
Q

A 12-year-old female spayed Labrador Retriever dog is presented for dental cleaning.
After she is induced under general anesthesia, her pulse oximetry drops below normal at 91%.
Thoracic radiographs show the following changes (see images below).
What is the most clinically significant finding evident on thoracic radiographs from this dog?

A - Spondylosis deformans is present
B - Heart is globoid in appearance
C - Left cranial lung lobe has alveolar infiltrates
D - There is a gastric foreign body
E - Osteochondritis dissecans is evident in the left shoulder

A

Answer: Left cranial lung lobe has alveolar infiltrates.

This dog has pneumonia of the left cranial lung lobe. There are diffuse alveolar infiltrates within the left cranial lung lobe with air bronchograms.
You should cancel the anesthetized procedure and wake up the patient because she is oxygenating poorly. Recheck radiographs are recommended once she is awake to rule out any component of atelectasis due to recumbency and sedation.
There is an incidental gastric foreign body (rounded metal opacity within the stomach that is in close association with additional irregular metal opacity).
The dog does have spondylosis deformans, which is a very common and incidental radiographic finding in older dogs.

https://www.merckvetmanual.com/nervous-system/diseases-of-the-spinal-column-and-cord/degenerative-diseases-of-the-spinal-column-and-cord-in-animals?autoredirectid=16745

Although these changes are present, they are not the most clinically important findings on these radiographs.
Click here to see normal canine thoracic radiographs.
Radiographic interpretation and images courtesy, Dr A. Zwingenberger and Veterinary Radiology.
Normal radiograph links courtesy, Imaging Anatomy, Univ. of Illinois Vet Med.

Pneumonia in Dogs and Cats

Concepts

Pneumonia is an inflammation of the pulmonary parenchyma (small airways, interstitium, and alveoli) caused by various infectious agents, including bacteria, viruses, fungi, and protozoa, or by aspiration of foreign materials.

Etiology

•	Bacterial: Common pathogens include Streptococcus, Staphylococcus, E. coli, and Bordetella bronchiseptica.
•	Viral: Canine distemper virus, adenovirus types 1 and 2, parainfluenza virus, and feline calicivirus.
•	Fungal: Cryptococcus, Aspergillus, and Blastomyces.
•	Parasitic: Filaroides, Aelurostrongylus, and Paragonimus spp.
•	Protozoal: Toxoplasma gondii and Pneumocystis jiroveci.
•	Aspiration: Gastric contents, medications, or food.

Symptoms

•	General: Lethargy, anorexia, cough (dry or productive), fever, leukocytosis, respiratory distress, hypoxia.
•	Auscultation: Crackles or dull areas indicating lung consolidation.
•	Species Differences:
•	Dogs: More prone to mycotic granulomatous pneumonia and tuberculous pneumonia.
•	Cats: Cryptococcal pneumonia is more common.

Pathogenesis

•	Inflammatory Response: Infiltration of inflammatory cells, cytokine release, increased mucus production, and consolidation of lung tissue.
•	Aspiration Pneumonia: Secondary bacterial infection following inhalation of foreign material, leading to severe inflammatory response.

Diagnostic Methods

•	History and Clinical Findings: Recent anesthesia, severe vomiting, or persistent cough.
•	Thoracic Radiography: Interstitial and/or alveolar changes, increased lung density.
•	Cytologic Examination: Neutrophilia, presence of bacteria.
•	Bacterial Culture and Sensitivity: To guide antibiotic therapy.
•	Bronchoalveolar Lavage: For cytology and culture.

Treatment

•	Supportive Care: Oxygen therapy (40%-60%), pulmonary physiotherapy, bronchodilators.
•	Antimicrobial Therapy: Empirical broad-spectrum antibiotics, adjusted based on culture results.
•	Monitoring: Frequent reexaminations, repeat chest radiographs every 2 weeks to monitor progress.

Pathological Findings

•	Bacterial Pneumonia: Neutrophilic infiltration, bacterial colonies.
•	Viral Pneumonia: Interstitial infiltrates, viral inclusions.
•	Fungal Pneumonia: Granulomatous inflammation, fungal hyphae or yeast.
•	Parasitic Pneumonia: Eosinophilic infiltration, larvae or adult parasites.
•	Protozoal Pneumonia: Infiltration by protozoa, associated with immune suppression.
How well did you know this?
1
Not at all
2
3
4
5
Perfectly
45
Q

An adult box turtle is presented for marked swelling at the left tympanic membrane with visible caseous material.
Which one of the following choices can be an underlying cause for this problem?
A - Vitamin A deficiency
B - Pasteurella multocida
C - Nutritional secondary_hyperparathyroidism
D - Mycoplasmosis
E - Septicemic cutaneous ulcerative disease (SCUD).

A

Answer: Vitamin A deficiency

This is an aural abscess, which can occur secondary to vitamin A deficiency.
Captive terrestrial box turtles are most at risk, usually due to diets containing little vitamin A.
Other presentations of hypovitaminosis A include froth from the nose (a sign of respiratory disease)
and renal disease.
For information on wild turtle conservation (and stunning photography), have a look at the Turtle Conservancy.

https://www.merckvetmanual.com/exotic-and-laboratory-animals/reptiles/bacterial-diseases-of-reptiles

https://www.merckvetmanual.com/exotic-and-laboratory-animals/reptiles/management-and-husbandry-of-reptiles?redirectid=31134

https://www.turtleconservancy.org/news/

How well did you know this?
1
Not at all
2
3
4
5
Perfectly
46
Q

A four-month-old Jack Russell terrier puppy was playing outdoors. Later that evening the owner noticed that the puppy was wheezing and coughing.
Based on the radiographs, what would be the best step to take next?

A - Barium swallow
B - Trans-tracheal wash
C - Edrophonium challenge test
D - Bronchoscopy
E - Dental prophylaxis

A

Answer: Bronchoscopy.

Bronchoscopy is the next best step. The radiographs show an alveolar pulmonary pattern; tracheal narrowing and intraluminal opacity that may indicate edema, mucus, or foreign material; and a dilated pharynx and aerophagia that support an upper airway obstruction.
In more detail: there is an alveolar pulmonary pattern in the left cranial and right middle lung lobes, consistent with aspiration pneumonia, and patchy increased opacity in the remaining lobes (seen best on the left), most consistent w/ non-cardiogenic pulmonary edema.
The trachea is narrowed at the thoracic inlet, with an apparent linear intraluminal opacity. There is dilation of the pharynx with air, and gas within the esophagus and gastrointestinal tract - a common finding in patients w/ respiratory distress from upper airway obstruction.
The dog received a diagnosis of bronchopneumonia, and a seven-inch tracheal foreign body.
Click here to see normal canine thoracic radiographs.
Radiographic interpretation and images courtesy, Dr A. Zwingenberger and Veterinary Radiology.
Normal radiograph links courtesy, Imaging Anatomy Univ. of Illinois Vet Med.

https://www.merckvetmanual.com/respiratory-system/aspiration-pneumonia-in-large-animals/aspiration-pneumonia-in-large-animals

https://vetmed.illinois.edu/imaging_anatomy/canine/thorax/ex02/thorax02.html

http://mirc.veterinaryradiology.net/storage/ss1/docs/9999.120659355827952154282177226211514216670/MIRCdocument.xml

How well did you know this?
1
Not at all
2
3
4
5
Perfectly
47
Q

A four-year-old male whippet is presented with an acute onset of swelling and pain in one of his testicles.
A post-op image is shown below.
Which test might have helped to diagnose the cause of the swelling before surgery?
A - Nuclear scintigraphy scan
B - Ultrasonography
C - Lateral radiography
D - Positive contrast cystography
E - Magnetic resonance imaging.

A

Answer: Ultrasonography

Use ultrasonography to identify testicular torsion in dogs with acute onset testicular swelling and pain. Other Ddx: scrotal hernia and epididymitis (usually bilateral).
Doppler ultrasonography of a torsed testicle will show reduced or absent perfusion, testicular enlargement, decreased parenchymal echogenicity, and enlargement of the epididymis and spermatic cord.
Clinical signs with testicular torsion are very suggestive. The contralateral testis is normal in size and shape, which differs from testicular tumors that often alter secretion of sex hormones, causing atrophy of the unaffected testicle.
Additionally, dogs with testicular torsion are in severe pain; often the patient needs very deep sedation for palpation and ultrasonography. Give appropriate doses of opioids and/or ketamine early if suspect testicular torsion.
Tx with castration to remove the affected testicle. This post-op image shows a hemorrhagic and infarcted right testicle (compared to normal left, consistent with torsion.

https://www.merckvetmanual.com/reproductive-system/reproductive-diseases-of-the-male-small-animal/orchitis-and-epididymitis-in-dogs-and-cats

https://zukureview.com/zuku-qod/navle/1821?chosen=Ir2r4R%2BtdyJJoCr7Hb1Ygw%3D%3D

48
Q

These larvae shown below have been removed from the topline of beef cattle in the early winter.
Which one of the following treatments is the most appropriate in both lactating dairy and beef cattle?
A - Rapid hand expulsion/squeezing.
B - None. These are self-limiting
C - Chloramphenicol
D - Pour-on moxidectin
E - Organophosphate spray

A

Answer: Pour-on moxidectin

These are the larvae of Hypoderma bovis or H. lineatum (the common and northern cattle grub, respectively) and pour-on moxidectin or eprinomectin are approved for use in both lactating dairy and beef cattle.
Other products are approved for non-lactating and beef cattle.
Withdrawal times must be followed.
Organophosphates are not approved for this use in cattle in the United States.
While the grubs can be manually expulsed/squeezed out, care must be taken to prevent rupture of the grubs.
Crushed larvae release antigens leading to a systemic anaphylactic reaction in cattle.
These pests are less common now since macrocyclic lactones gained widespread use in the last few decades.

The adult flies (gad flies or heel flies) lay eggs on the lower limbs of cattle over the summer.
The larvae penetrate the skin and migrate to their respective preferred locations.
By early winter, the larvae arrive in the subcutaneous tissues of the back and make breathing pores.
This cyst/warble stages lasts four to eight weeks, culminating in 3rd-stage larvae dropping to the ground and completing the life cycle.
Clinical presentation of the larval infections range from localized tissue trauma around the warbles/cysts to periostitis, osteomyelitis, and neurological dysfunction.
Death of 1st-stage larvae in the esophagus or spinal canal can lead to location-specific clinical signs.
Cattle bothered by adult flies in summer show â??gadding behaviorâ?? a?? stamping and running with tails in the air.
Dx: Identify 3rd stage larvae.

Key Information on Hypoderma spp. (Cattle Grubs)

Etiology and Life Cycle

•	Causative Agents: Hypoderma bovis and Hypoderma lineatum.
•	Hosts: Cattle, occasionally horses, sheep, goats, and humans.
•	Lifecycle Stages: Egg deposition on hair, larval penetration, migration to resting sites (esophagus for H. lineatum, spinal canal for H. bovis), subcutaneous warbles on back.

Clinical Findings and Pathogenesis

•	Signs: Warbles, reduced productivity, carcass trimming, hide depreciation, gadding behavior.
•	Complications: Dysphagia, bloat, paralysis if larvae die in resting sites.

Diagnosis

•	Methods: Clinical examination for warbles, serologic testing for antibodies.

Treatment and Control

•	Medications: Macrocyclic lactones (e.g., ivermectin).
•	Management: Treat at the end of fly season, avoid mechanical removal of larvae.
49
Q

A 15-year-old Quarter horse mare is presented for right front lameness grade 3/5 on the AAEP lameness scale (lameness consistently visible at the trot). She becomes sound after an abaxial nerve block.
Radiographs of the distal limb look are shown below.
What is the common name for the top differential diagnosis?
A - High ringbone
B - Exostosis of the second metacarpal
C - Pedal osteitis
D - Spavin
E - Chronic proliferative synovitis

A

Answer: High ringbone

This is an example of high ringbone (i.e., degenerative joint disease [DJD] of the proximal interphalangeal joint). Very common in horses. Can be due to chronic wear and tear, overuse, or secondary to a traumatic episode.
Follow this link to see a radiograph of arthrodesis of the proximal interphalangeal joint to resolve
DJD.
Osselets are traumatic arthritides of the metacarpophalangeal joints.
Pedal osteitis is demineralization of the solar margin of the distal phalanx, usually due to inflammation.
Bone spavin is osteoarthritis of the distal intertarsal, tarsometarsal, and less commonly, the proximal intertarsal joints.

Osteoarthritis of the Proximal Interphalangeal Joint in Horses

Definitions:

•	Osteoarthritis (OA): Degenerative joint disease affecting the proximal interphalangeal (pastern) joint, commonly referred to as “high ringbone.”

Causative Agents:

•	Primary Causes: Trauma, wear and tear, overuse.
•	Secondary Causes: Infection, developmental orthopedic disease.

Physiopathology:

•	Cartilage Degeneration: Loss of cartilage, new bone formation around the joint.
•	Chronic Changes: Subchondral bone sclerosis, joint space narrowing.

Symptoms:

•	Subtle lameness progressing to more noticeable lameness as disease advances.

Clinical Changes:

•	Radiographic Findings: New bone formation, subchondral lysis, joint space loss.

Assessment:

•	Diagnostic Analgesia: Localizes lameness.
•	Flexion Tests: May reveal pain.

Treatment:

•	Medical: Anti-inflammatories (NSAIDs, corticosteroids), intra-articular injections (e.g., ethanol).
•	Surgical: Arthrodesis using plates and screws for severe cases.

Medications:

•	NSAIDs: Reduce inflammation and pain.
•	Corticosteroids: Potent anti-inflammatory effects.
•	Biologic Therapies: Platelet-rich plasma, stem-cell therapy.

https://zukureview.com/zuku-qod/navle/1777?chosen=gU3O87sa%2Buyqgo7azsEKAw%3D%3D&utm_source=Zukureview+Subscribers&utm_campaign=f9eddcc10e-EMAIL_CAMPAIGN_2024_05_24_01_50&utm_medium=email&utm_term=0_1c9568dbdc-f9eddcc10e-%5BLIST_EMAIL_ID%5D

50
Q

A horse presents with distal limb edema in all 4 limbs (as seen below), petechiae of the mucous membranes, depression, and a low-grade fever of 101.9°F/38.8°C (N = 99-101°F, 37.2-38.3°C).
Multiple horses in the stable recently suffered from an upper respiratory infection of undiagnosed etiology.
Which type of immune reaction is most likely responsible for the vasculitis in this horse?

A - Cell-mediated immune reactions (type IV).
B - Antibody-mediated cytotoxic reactions (type II).
C - Immediate hypersensitivity and atopy_(type I).
D - Antigen-antibody_complex (type III).
E - None of these

A

Answer: Antigen - antibody complex Type III

In type Ill reactions such as purpura hemorrhagica, antigen-antibody complexes are deposited on

the endothelium, stimulating complement and a neutrophilic inflammatory response and vascular

damage. The petechiae and distal limb edema in this horse are consistent with vasculitis.
Classic type III diseases include:

Glomerulonephritis

Hypersensitivity pneumonitis (think moldy hay)

Purpura hemorrhagica (think post-strangles infection, but possible with other infections as well)
Anterior uveitis

https://zukureview.com/zuku-qod/navle/1776?chosen=2WC76OIttc0kQZcE0uORIhb1thKz4G47KGSX%2BemM0vArrJLRwspYIto6oaP1OtC8&utm_source=Zukureview+Subscribers&utm_campaign=94fba8eb06-EMAIL_CAMPAIGN_2024_05_23_01_50&utm_medium=email&utm_term=0_1c9568dbdc-94fba8eb06-%5BLIST_EMAIL_ID%5D

51
Q

A client with a small backyard flock presents a chicken carcass for necropsy. She says the bird had lost significant weight, was slow to move around, and was limping.
Necropsy reveals various-sized granulomatous lesions distributed throughout the spleen, liver, intestines, and bone marrow, as seen below.
What is the presumptive diagnosis?
A - Ulcerative enteritis
B - Candidiasis
C - Avian tuberculosis
D - Infectious coryza
E - Colibacillosis

A

Answer: Avian tuberculosis.

This is classic avian tuberculosis (usually caused by Mycobacterium avium subsp. avium), which can affect all types of birds.
It is a chronic, granulomatous bacterial infection that is slow to spread and is characterized by gradual weight loss. M. avium is prevalent worldwide and is spread by infective feces.
It is ZOONOTIC in immune-compromised people.
Dx: difficult. Positive wattle or skin tuberculin test confirms exposure but negative does not rule out disease. Suspicion can be based on seeing lots of acid-fast bacteria on smears from lesions.
Rx: none. Prevent by careful purchasing and/or quarantine (at least 6 months!) of new birds.
Improved sanitation and rapid flock turnover in commercial flocks has mostly eliminated this infection from these groups.
Candidiasis is an intestinal mycotic disease typically characterized by thickened mucosa and white, raised pseudomembranes lesions in the crop and esophagus.
Colibacillosis (Escherichia coli) is common in chickens and can cause subacute pericarditis, airsacculitis, salpingitis, peritonitis, and acute fatal septicemia. It can occasionally cause granulomas in the liver and spleen (but not the intestine, bone marrow like in this chicken). Lameness would also be less common in the history for colibacillosis.
Infectous coryza is a respiratory disease of chickens that acutely causes nasal discharge, sneezing, and facial swelling.
Ulcerative enteritis (Clostridium colinum) causes depression, anorexia, and diarrhea.

https://zukureview.com/zuku-qod/navle/1775?chosen=PAlciwsYYapQcMDF/4yizGuaAFFmbpAogokKFm9EIn4%3D&utm_source=Zukureview+Subscribers&utm_campaign=1e46ba7409-EMAIL_CAMPAIGN_2024_05_22_01_50&utm_medium=email&utm_term=0_1c9568dbdc-1e46ba7409-%5BLIST_EMAIL_ID%5D

52
Q

Cytological evaluation of a blood smear from a sick horse reveals the following (see below).
Which constellation of signs would you expect in a typical acute case of a horse with this disease?

A - Nystagmus, head tilt, strabismus
B - Fever, petechiations, hemolytic anemia
C - Limb edema, ventricular arrhythmias, ataxia
D - Elevated serum liver enzymes, hyperammonemia, head-pressing.
E - Weight loss,_partial anorexia, poor performance

A

Answer: Fever,petechiations, hemolytic anemia

Fever, petechiations, hemolytic anemia
This cytology shows Babesia caballi, one of the causative agents of equine piroplasmosis (the other is Theileria equi - looks like a maltese cross under the microscope) and acute disease is characterized by fever, petechiations, and signs consistent with hemolytic anemia.
Piroplasmosis is characterized by hemolytic anemia and is spread primarily by ixodid ticks or blood contamination. It is endemic in tropical, subtropical, and some temperate regions.
It is REPORTABLE in the USA.
Clinical infection can be acute, chronic, or inapparent, depending on strain, the horse’s health, and geographic region.
Acute clinical signs include those described here along with lethargy, anorexia, pale or icteric mucous membranes, and with worsening disease - tachycardia, tachypnea, weakness, and hemoglobinuria. Weight loss, poor performance, and partial anorexia can be seen along with mild anemia in horses with chronic piroplasmosis.
Dx: Demonstrate intra-erythrocytic organisms on blood smear and/or serology.
Rx: Imidocarb dipropionate and supportive care. There is a different duration of Tx depending on whether in endemic or nonendemic region (because trying to treat clinical signs in endemic region vs. clear disease in nonendemic region).
Limb edema, ventricular arrhythmias, and ataxia (the latter two uncommonly) can be seen with Anaplasma phagocytophila infections in horses, along with fever, petechiations, icterus, and depression.

https://zukureview.com/zuku-qod/navle/1761?chosen=9XS83s9tyJMi%2BCsyDIqqJzjVjqCaNOTKRr/0H1RLMBBhEI1dcPaNOZz2u%2B4y5I4X&utm_source=Zukureview+Subscribers&utm_campaign=c0ab8357b6-EMAIL_CAMPAIGN_2024_05_16_01_51&utm_medium=email&utm_term=0_1c9568dbdc-c0ab8357b6-%5BLIST_EMAIL_ID%5D

53
Q

A pale-colored Egyptian Arabian foal is presented for evaluation within a few hours of a dystocia with assisted vaginal delivery.
The foal has opisthotonos, nystagmus, limb rigidity and paddling, and has not yet stood and nursed.
What is the presumptive diagnosis based on the information available?

A - Botulism
B - Overo lethal white condition
C - Lavender foal syndrome
D - Myasthenia gravis
E - Portosystemic vascular anomaly

A

Answer: Lavender Foal Syndrome

This presentation is consistent with lavender foal syndrome (LFS), an inherited lethal neurologic disorder that affects Arabian foals
with a dilute coat color.

LFS is a recessive genetic disorder and newborn foals present as in this case. Affected foals are typically euthanized within days of birth. There is no treatment.
A genetic test is available and affected animals should not be bred.

Botulism in foals less than ~1 month old (i.e., â? ?shaker foal syndromeâ??) is characterized by progressive symmetric motor paralysis with muscular tremors.

Affected foals cannot stand for more than a few minutes at a time.

Overo lethal white syndrome is characterized by colic and meconium impaction due to ileocolonic agangliosis. It occurs in Paint foals with blue eyes that are the result breeding two Overos (a type of coat pattern in Paint horses).

Myasthenia gravis is not reported in horses.

Neurologic signs seen with hyperammonemia associated with portosystemic vascular anomalies (PSVAs, also referred to as portosystemic shunts) are not seen until affected foals are 2-3 months old and eating more grain and forage.

54
Q

A horse is presented in severe respiratory distress in southern California after a suspected snake bite.
There is significant nasal swelling (see below), flared nostrils, abdominal movement with each breath, stridor, and tachypnea. Aside from local tissue damage and associated upper airway obstruction, what is another major complication of crotalid envenomation?

A - Diarrhea
B - Acute hepatic necrosis
C - Immune suppression
D - Chronic visual impairment
E - Coagulopathy

A

Answer: Coagulopathy

Coagulopathies are common in association with crotalid (e.g., rattlesnake) envenomation.

Additional concerns include thrombocytopenia, neurotoxicity, and cardiovascular toxicity.

Temporary tracheostomy should be performed immediately in a horse in severe respiratory distress due to upper airway obstruction. The physical examination and further supportive care can be delayed until after an airway is obtained.

Additional diagnostics include CBC/blood smear/serum biochemistry/coagulation profile. See hypofbrinogenemia, thrombocytopenia, prolonged prothrombin time, and increased FDPs. Might see echinocytes in heparin or citrate samples, and severity of echinocytosis is associated with severity of envenomation.

Rx: IV fluids for hypotension, NSAIDs for analgesia, steroids are controversial (1 dose early may be beneficial), tetanus prophylaxis, antimicrobial prophylaxis, antivenin if owner can afford it/it is available, and occasional blood transfusion. If you look closely at the picture of this horse, you can see he is receiving a blood transfusion.

Snakebites in Animals

Overview

Snakebites in animals can result in a variety of clinical signs and outcomes depending on the species of snake, the amount of venom injected, and the location of the bite. Prompt recognition and appropriate treatment are crucial for improving the prognosis.

Types of Venomous Snakes

1.	Pit Vipers (Crotalinae):
•	Examples: Rattlesnakes, copperheads, cottonmouths (water moccasins).
•	Venom: Hemotoxic, causing tissue destruction, coagulopathy, and systemic effects.
2.	Elapids:
•	Examples: Coral snakes, cobras, kraits.
•	Venom: Neurotoxic, causing paralysis and respiratory failure.

Pathophysiology

•	Venom Composition: Contains enzymes, proteins, and peptides causing local and systemic effects.
•	Hemotoxic Venom:
•	Local Effects: Pain, swelling, hemorrhage, tissue necrosis.
•	Systemic Effects: Coagulopathy, hypotension, shock.
•	Neurotoxic Venom:
•	Local Effects: Minimal, often no significant swelling or pain.
•	Systemic Effects: Neuromuscular blockade, respiratory paralysis, cardiovascular collapse.

Clinical Changes and Symptoms

1.	Local Signs:
•	Pain and Swelling: Rapid onset, progressing over several hours.
•	Ecchymosis and Hemorrhage: Bruising and bleeding at the bite site.
•	Necrosis: Tissue death, potentially leading to severe infection.
2.	Systemic Signs:
•	Weakness and Collapse: Often early signs in severe envenomation.
•	Hypotension and Shock: Decreased blood pressure, tachycardia.
•	Coagulopathy: Prolonged bleeding, petechiae, hematuria.
•	Respiratory Distress: Particularly in neurotoxic envenomation.
•	Neurologic Signs: Paralysis, ataxia, altered mental status.

Diagnosis

1.	Clinical Examination:
•	History: Details of the bite incident, snake identification if possible.
•	Physical Examination: Assessing the bite site, systemic signs, and vital parameters.
2.	Laboratory Tests:
•	CBC: Leukocytosis, thrombocytopenia.
•	Coagulation Profile: Prolonged clotting times, decreased fibrinogen.
•	Serum Biochemistry: Elevated creatine kinase, renal function tests for myoglobinuria.
•	Urinalysis: Hematuria, myoglobinuria.
3.	Imaging:
•	Radiography/Ultrasound: Assessing for tissue damage and complications such as compartment syndrome.

Management Strategies

1.	Immediate Care:
•	Immobilization: Keep the animal calm and immobilize the affected limb to slow venom spread.
•	Avoid Tourniquets and Incisions: These can cause more harm than benefit.
2.	Supportive Care:
•	Intravenous Fluids: To maintain blood pressure and hydration.
•	Pain Management: Opioids (e.g., morphine), avoiding NSAIDs due to potential coagulopathy.
•	Antihistamines: To manage allergic reactions if needed.
3.	Antivenom Therapy:
•	Indications: Moderate to severe envenomation with systemic signs.
•	Types: Species-specific or polyvalent antivenoms.
•	Administration: Intravenous infusion, monitoring for allergic reactions.
4.	Additional Therapies:
•	Antibiotics: For secondary bacterial infections, particularly in necrotic wounds.
•	Corticosteroids: Use is controversial; may be considered in severe cases with significant inflammation.
•	Tetanus Prophylaxis: Recommended for animals with unknown vaccination status.
5.	Monitoring and Aftercare:
•	Frequent Monitoring: Vital signs, coagulation status, renal function.
•	Wound Care: Cleaning, debridement, and dressing changes for necrotic tissue.
•	Rehabilitation: Physical therapy for long-term recovery if needed.

Prognosis

•	Variable: Depends on the species of snake, the amount of venom injected, the location of the bite, and the timeliness of treatment.
•	Early and Appropriate Treatment: Significantly improves outcomes, reducing mortality and long-term complications.

Summary for Veterinary Professionals

•	Snakebites can cause severe local and systemic effects, with pit vipers and elapids being the primary venomous snakes of concern.
•	Clinical signs vary with the type of venom and include pain, swelling, coagulopathy, respiratory distress, and neurologic deficits.
•	Diagnosis involves clinical examination, laboratory tests, and imaging to assess the extent of envenomation and complications.
•	Management includes immobilization, supportive care, antivenom therapy, pain management, and wound care.
•	Prognosis is variable but can be improved with early and appropriate treatment.
55
Q

Which of the following choices correctly identifies the leukocyte in this image of a blood smear from a horse, and the most common types of diseases to which it responds?

A - Monocyte; viral/bacterial infection, and chronic inflammation
B - Lymphocyte; immune disease and viral infection
C - Macrophage; bacterial and viral infection
D - Neutrophil; wound repair and bacterial infection
E - Basophil; allergy and parasitic infestation

A

Answer: Basophil

This is a basophil, typically associated with allergy or parasite infestation. Basophils (BSO), eosinophils (EOS), and neutrophils (PMNs) are the three types of granulocytic white blood cells.

Phagocytosis is an important function of EOS and PMNs but this is not a primary role of BSOs.
Granulocytic phagocytes have segmented nuclei and cytoplasmic granules - orange/pink in EOS;
purple/dark blue in BSOs; pale pink in PMNs.

The granules of basophils contain many mediators of inflammation and large amounts of histamine.
They commonly respond to allergic conditions and parasite infestation, similar to eosinophils.
For more info and images, see the Cornell Clin Path website: Hematology Atlas, and Equine blood cells.

White blood cells (WBCs) in animals, relevant for your NAVLE preparation:

Phagocytes:

•	Mononuclear Phagocytes: Arise from marrow, circulate as monocytes, and become macrophages in tissues. They play a role in antigen presentation and produce substances like interleukin-1.
•	Granulocytes: Include neutrophils, eosinophils, and basophils. Neutrophils are short-lived and move from blood to tissues. Eosinophils respond to histamine and are involved in parasite defense and inflammation modulation. Basophils contain histamine and other inflammation mediators.

Lymphocytes:

•	T Lymphocytes: Differentiated into “helper” T4 cells (involved in humoral antibody production) and “cytotoxic” T8 cells (responsible for immune responses against specific antigens). These cells are crucial for cellular immunity.
•	B Lymphocytes: Responsible for humoral immunity through antibody production. B cells differentiate into plasma cells that produce immunoglobulins (IgA, IgM, IgG, IgE), each with specific functions (e.g., IgE in allergic reactions).
•	Natural Killer Cells: Destroy foreign cells like neoplastic cells without prior sensitization.

Immune Responses:

•	Appropriate Response: Activation of the immune system during infections or vaccinations.
•	Inappropriate Response: Immune-mediated diseases (e.g., immune-mediated hemolytic anemia) and allergies, which involve the immune system attacking host tissues or reacting to allergens (e.g., anaphylaxis).

Diseases and Conditions:

•	Neutropenia: Caused by marrow failure, infections, or immune-mediated destruction, leading to increased risk of bacterial infections.
•	Lymphocytosis: Can occur due to epinephrine release or chronic infections, while lymphopenia is often a response to glucocorticoid secretion.
•	Lymphoproliferative Disorders: Include lymphomas, and leukemias, often manifesting as abnormal proliferation of lymphocytes.

Medications:

•	Glucocorticoids: Used to manage inappropriate immune responses by reducing inflammation and suppressing the immune system.

Diagnostic and Treatment Approaches:

•	Assessment: Blood tests, bone marrow analysis, and histopathology to assess WBC counts, morphology, and function.
•	Treatment: Tailored to underlying causes, including immunosuppressants, antibiotics for secondary infections, and supportive care.

https://www.merckvetmanual.com/circulatory-system/hematopoietic-system-introduction/white-blood-cells-in-animals

56
Q

You are helping a local goat dairy owner interpret the results of semi-annual ELISA serological testing of her herd for caprine arthritis encephalitis (CAE).
One two-month-old kid is CAE-positive by ELISA.
Another two-month-old kid is CAE-negative by
ELISA.
How do you interpret these results?

A - Cannot say if CAE-positive kid has CAE, CAE-negative kid is not infected
B - Both kids are infected with CAE
C - CAE-positive kid has CAE, cannot say if CAE-negative kid is infected or not
D - CAE-positive kid has CAE, CAE-negative kid does not have CAE
E - Cannot tell if either kid is truly CAE positive or negative

A

Answer: ** E - Cannot tell if either kid is truly CAE positive or negative**

You cannot tell if either kid is truly CAE positive or negative yet. Should probably retest both after 90 days, or at next semiannual herd testing.

A positive ELISA test for caprine arthritis encephalitis (CAE) in a kid less than 90 days old may reflect maternal antibody or a false positive. A negative test in a kid less than 90 days old does not rule out CAE (though most kids infected at birth from colostrum develop a measurable antibody response by 4-10 weeks after infection).
Note lots of qualifiers about CAE testing: A positive test in an adult goat implies infection, but does not confirm that clinical signs were caused by CAE virus. Negative tests do NOT rule out CAE infection.

DEFINITIVE Dx is to find characteristic lesions by biopsy/necropsy. Virus isolation can substantiate
Dx, but may not be practical on herd basis.

Caprine Arthritis and Encephalitis (CAE) in Goats

Concepts

CAE is a persistent viral infection caused by the Caprine Arthritis Encephalitis Virus (CAEV), a lentivirus affecting goats. It manifests in various forms, including arthritis, leukoencephalomyelitis, mastitis, and interstitial pneumonia.

Etiology

•	Infectious Agent: CAEV, an enveloped, single-stranded RNA lentivirus in the Retroviridae family.
•	Transmission: Primarily through ingestion of infected colostrum or milk by kids. Horizontal transmission occurs via direct contact, fomites, and contaminated equipment.

Pathogenesis

•	Virus Spread: Infected macrophages in colostrum and milk are absorbed, spreading the virus systemically via mononuclear cells. Periodic replication induces characteristic lymphoproliferative lesions in target tissues like the lungs, synovium, choroid plexus, and udder.
•	Immune Response: The infection elicits strong humoral and cell-mediated immune responses, but these are not protective.

Symptoms

•	Polysynovitis-Arthritis: Most common in adult goats, leading to joint swelling, lameness, weight loss, and poor hair coats.
•	Leukoencephalomyelitis: Mainly in kids aged 2-6 months, causing weakness, ataxia, proprioceptive deficits, progressing to paralysis.
•	Interstitial Pneumonia: Chronic, leading to progressive dyspnea in adult goats.
•	Indurative Mastitis: “Hard udder” syndrome, causing firm, swollen mammary glands with low milk production.

Diagnostic Methods

•	Clinical Signs: Based on observable symptoms.
•	Serologic Testing: Agar gel immunodiffusion and ELISA tests to determine herd CAEV status.
•	Histopathology: Definitive diagnosis through characteristic lesions in biopsy or post-mortem samples.
•	Differential Diagnoses: Includes infectious arthritis (Mycoplasma spp.), traumatic arthritis, enzootic ataxia, spinal cord abscesses, cerebrospinal nematodiasis, polioencephalomalacia, listeriosis, and rabies.

Treatment and Control

•	No Vaccine or Specific Treatment: Supportive care includes regular foot trimming, additional bedding, NSAIDs for arthritis, and good nursing care for encephalomyelitis.
•	Management Practices: Isolation of kids at birth, feeding heat-treated colostrum and pasteurized milk, semi-annual serologic testing, segregation, and culling of seropositive goats.
•	Supportive Care: High-quality, readily digestible feed, antibiotics for secondary infections.

https://www.merckvetmanual.com/generalized-conditions/caprine-arthritis-and-encephalitis/caprine-arthritis-and-encephalitis

57
Q

Which one of the following choices includes the cardinal sign of trigeminal neuritis?

A - Paralyzed eyelid, ear, or lip on one or both sides of the face
B - Dysphagia, dysphonia and stridor
C - Inability to close the mouth
D - Masseter muscle pain associated with chewing.
E - Circling and head tilt toward side of lesion, no other signs

A

Answer: Inability to close the mouth

Idiopathic trigeminal neuritis, due to inflammation of cranial nerve 5 (CN 5), is characterized by acute onset of flaccid jaw paralysis. Affected animals cannot close their mouth and have difficulty eating and drinking. Seen occasionally in dogs, rare in cats.

Idiopathic facial nerve paralysis, affecting cranial nerve 7 (CN 7), results in the inability to move the eyelid, lip, or ear and dryness of the eyes and mouth.

Masticatory myositis is characterized by pain on opening the mouth and swelling of the muscles of mastication (acute) or atrophy of the temporalis and masseter muscles with the inability to open the mouth due to fibrosis (chronic).

Dysphagia, dsyphonia, and stridor are most often associated with dysfunction of the vagus nerve, cranial nerve 10 (CN 10).

Circling and head tilt toward the side of the lesion with no other signs is a common presentation of vestibulocochlear nerve, cranial nerve 8 (CN 8) lesions.

Concurrent CN 7 paralysis and Horner syndrome (ptosis, miosis, enophthalmos) may be present with middle- and inner-ear infections.

Acquired Myasthenia Gravis:

•	Etiology: Autoimmune destruction of acetylcholine receptors at the neuromuscular junction.
•	Clinical Forms: Generalized (exercise-induced weakness), focal (facial, pharyngeal weakness), fulminant (acute paralysis).
•	Diagnosis: Edrophonium chloride test, serum antibody detection.
•	Treatment: Pyridostigmine (1-3 mg/kg, PO, 2-3 times/day), immunosuppressive therapy, management of complications like megaesophagus.

Acute Idiopathic Polyradiculoneuritis:

•	Etiology: Immune-mediated, often triggered by raccoon bites or vaccination.
•	Clinical Signs: Flaccid tetraparesis, respiratory paralysis, normal pain perception.
•	Diagnosis: CSF analysis, electromyography.
•	Treatment: Supportive care, recovery within 2-6 months.

Chronic Inflammatory Demyelinating Polyneuropathy:

•	Etiology: Unknown, immune-mediated suspected.
•	Clinical Signs: Tetraparesis, cranial nerve dysfunction.
•	Diagnosis: Nerve biopsy, slowed conduction on electromyography.
•	Treatment: Corticosteroids, with potential relapses.
58
Q

An outbreak of diarrheal disease of piglets has occurred which affected the healthiest animals in the herd, one to two weeks after weaning.
Some affected piglets had no signs except peracute death.
Other affected piglets exhibit diarrhea, ataxia, paralysis, and recumbency.
What condition is at the top of the differential diagnosis list?

A - Epidemic transmissible gastroenteritis (TGE).
B - Streptococcus suis
C - Porcine proliferative enteritis
D - Clostridium perfringens type C enteritis
E - Edema disease

A

Answer: Edema disease

Edema disease is caused by shiga toxin-producing E. coli (STEC).
Look for severe acute illness ranging from peracute death with no signs to CNS involvement with ataxia, paralysis, and recumbency in healthiest pigs one to two weeks after weaning.

Streptococcus suis could cause sudden death but would not cause diarrhea and would see polyserositis on necropsy.

Clostridium perfringens type C
enteritis, also called
enterotoxemia in other animals, is characterized by a HEMORRHAGIC diarrhea in one-to three-day-old piglets.

Porcine proliferative enteritis is principally a diarrheal disease of growing finishing (40- to 80-|b) pigs and young breeding pigs.

Epidemic transmissible gastroenteritis (TGE) in non-immune pig herds characterized by HIGH MORBIDITY and HIGH MORTALITY in piglets less than one week old.

** Edema Disease in Pigs - Comprehensive Study Guide**

Definition

•	Edema Disease: Peracute vasculitis causing facial swelling, enteritis, and sometimes focal encephalomalacia in post-weaning pigs.

Causative Agent

•	Escherichia coli (E. coli): Specifically hemolytic strains producing F18 fimbriae and Shiga toxin 2e (Stx2e).

Pathophysiology

•	Transmission: Fecal-oral route from contaminated environments or infected pigs.
•	Infection Mechanism: Colonization of the intestine followed by toxin production. Toxin binds to specific receptors on vascular endothelium, leading to cell death and vascular damage.

Clinical Signs

•	Peracute Death: Often with no prior signs.
•	CNS Signs: Ataxia, paralysis, recumbency.
•	Visible Edema: Especially in the face, eyelids, and larynx.
•	Other Signs: Anorexia, dyspnea, submandibular swelling.

Lesions

•	Gross Lesions: Subcutaneous edema, stomach submucosa edema, mesocolon edema.
•	Microscopic Lesions: Degenerative angiopathy, necrosis of arterial smooth muscle cells, focal encephalomalacia in the brainstem.

Diagnosis

•	Clinical Signs: Characteristic facial edema and sudden death.
•	Necropsy: Edema in characteristic sites.
•	Laboratory Confirmation: Isolation and PCR identification of Stx2e and F18-positive E. coli.

Treatment and Control

•	Treatment: Often ineffective for affected pigs due to rapid disease progression. Oral antimicrobials for unaffected pigs in infected herds based on sensitivity testing.
•	Control Measures:
•	Feed Management: High-fiber, low-protein diets.
•	Vaccination: Various commercial vaccines targeting different aspects of the E. coli strain.
•	Passive Immunity: Systemic immunization or oral immunization with anti-F18 antibodies.

St reptococcus suis Infection in Pigs: NAVLE Study Guide

Definitions and Causative Agents

•	Streptococcus suis (S. suis): A gram-positive, facultatively anaerobic coccus causing septicemia, meningitis, arthritis, and endocarditis in pigs, particularly post-weaned piglets.
•	Serotypes: 29 serotypes recognized, with serotype 2 being most common in Europe and Asia.

Pathogenesis

•	Transmission: Colonization occurs at birth from vaginal secretions and nursing. Spread via direct contact, fomites, and possibly through the intestinal route.
•	Virulence Factors: Capsular polysaccharide, muramidase-released protein, extracellular factor, and hemolysin (suilysin).

Clinical Findings

•	Acute Signs: Fever, septicemia, meningitis (opisthotonos, convulsions, paddling), arthritis, endocarditis, and sudden death.
•	Lesions: Neutrophilic meningitis, polyserositis, fibrinopurulent exudates, endocarditis, and splenomegaly.

Diagnosis

•	Methods: Based on clinical signs, bacterial culture, serotyping, PCR assays, and histopathology.
•	Differential Diagnoses: Include polyserositis, meningitis from other pathogens, endocarditis, septicemia, and polyarthritis from other bacteria.

Treatment and Prevention

•	Antibiotics: Beta-lactams (penicillin, ampicillin, amoxicillin). Resistance may vary.
•	Anti-inflammatories: To reduce tissue inflammation.
•	Vaccination: Autogenous vaccines with varying effectiveness.
•	Management: Early detection, environmental control (overcrowding, ventilation), and prompt treatment.

Zoonotic Risk

•	Human Infection: Can cause septicemia, meningitis, endocarditis, and arthritis. Transmission through contact with infected pigs or consumption of raw pork products. Higher risk in Southeast Asia.

For detailed information, refer to the Merck Veterinary Manual article on Streptococcus suis Infection in Pigs.

https://zukureview.com/zuku-qod/navle/1963?chosen=MNS4vGCLeUF0Fos0r4TrHXpAgmG221Bs%2BLmK7gsUi9UxWK%2Bb/qHdMuk793PQQTDn&utm_source=Zukureview+Subscribers&utm_campaign=dce840d132-EMAIL_CAMPAIGN_2024_08_15_01_52&utm_medium=email&utm_term=0_1c9568dbdc-dce840d132-%5BLIST_EMAIL_ID%5D

59
Q

Which nerve block would most specifically relieve lameness resulting from fracture of the navicular bone?

A - Tibial and peroneal
B - Median and ulnar
C - Palmar digital
D - Abaxial sesamoidean
E - Low four-point

A

Answer: Palmar digital

The palmar digital nerve block (also called a PD or heel block) would anesthetize at least the palmar third of the foot, including the navicular bone.

Historically, the PD block was considered to anesthetize the palmar/plantar 1/3 of the foot and sole. However, it has been shown that it desensitizes 70-80% of the foot in most horses.
All of the other nerve blocks can desensitize the navicular bone too, but would not be as specific as a palmar digital nerve block because they also desensitize a larger region.

The abaxial sesamoidean nerve block (ASNB, also called a pastern or foot block) would anesthetize the entire foot and much of the
pastern. Sometimes an ASNB is needed to completely eliminate or diagnose navicular lameness, but it is less specific than the PD block.

A low four-point nerve block (also called low palmar or volar block) would desensitize the fetlock and areas distal to it.

The median and ulnar nerve block would anesthetize the carpus and areas distal to it.
Click here to see forelimb nerve block landmarks.

The tibial and peroneal nerve block would anesthetize the tarsus and areas distal to it.
Click here and scroll down to see pelvic limb nerve block landmarks.

** Regional Anesthesia in Equine Lameness**

  1. Palmar (or Plantar) Digital Nerve Block• Usage: Localizes lameness in the heel or sole.
    • Pathology: Navicular disease, heel pain.
    • Localization: Medial and lateral palmar digital nerves at the pastern level.
    • Medication: Mepivacaine HCl (2%).
    • Dose: 1-2 mL per site.
    • Technique: Injection proximal to the collateral cartilage.
    • Advantages: High specificity.
    • Complications: Rare, but include infection or nerve damage.
  2. Abaxial Sesamoid Nerve Block• Usage: Localizes lameness in the pastern or foot.
    • Pathology: Pastern osteoarthritis, ringbone.
    • Localization: Abaxial to the proximal sesamoid bones.
    • Medication: Mepivacaine HCl (2%).
    • Dose: 1-2 mL per site.
    • Technique: Injection at the base of the proximal sesamoid bones.
    • Advantages: Effective for pastern lameness.
    • Complications: Infection, inadvertent tendon sheath puncture.
  3. Low Palmar (or Plantar) Nerve Block (Low Four-Point Block)• Usage: Localizes lameness in the fetlock or below.
    • Pathology: Fetlock arthritis, flexor tendon injuries.
    • Localization: Medial and lateral palmar nerves and palmar metacarpal nerves.
    • Medication: Mepivacaine HCl (2%).
    • Dose: 2-3 mL per site.
    • Technique: Injection at the level of the distal ends of the splint bones.
    • Advantages: Broad area of desensitization.
    • Complications: Rare, but include hematoma.
  4. High Palmar (or Plantar) Nerve Block (High Four-Point Block)• Usage: Localizes lameness in the metacarpal or metatarsal region.
    • Pathology: Suspensory ligament injuries.
    • Localization: Medial and lateral palmar nerves and palmar metacarpal nerves.
    • Medication: Mepivacaine HCl (2%).
    • Dose: 3-5 mL per site.
    • Technique: Injection just below the carpus or hock.
    • Advantages: Effective for high suspensory issues.
    • Complications: Hematoma, inadvertent nerve damage.
  5. Median and Ulnar Nerve Block• Usage: Localizes lameness in the carpal region or below.
    • Pathology: Carpal arthritis, severe tendon injuries.
    • Localization: Median nerve (medial aspect of the radius), ulnar nerve (distal to the accessory carpal bone).
    • Medication: Mepivacaine HCl (2%).
    • Dose: 10-15 mL per site.
    • Technique: Separate injections for median and ulnar nerves.
    • Advantages: Provides complete desensitization of the lower limb.
    • Complications: Risk of hitting blood vessels, hematoma.
  6. Deep Branch of the Lateral Plantar Nerve Block• Usage: Localizes lameness in the proximal suspensory ligament.
    • Pathology: Proximal suspensory desmitis.
    • Localization: Deep branch of the lateral plantar nerve at the level of the hock.
    • Medication: Mepivacaine HCl (2%).
    • Dose: 5-10 mL.
    • Technique: Injection near the origin of the suspensory ligament.
    • Advantages: Specific for suspensory ligament issues.
    • Complications: Difficult to perform, risk of damaging surrounding structures.
  7. Tibial and Peroneal Nerve Block• Usage: Localizes lameness in the tarsal region or below.
    • Pathology: Tarsal arthritis, severe lower limb injuries.
    • Localization: Tibial nerve (medial aspect of the hock), peroneal nerve (lateral aspect of the hock).
    • Medication: Mepivacaine HCl (2%).
    • Dose: 10-15 mL per site.
    • Technique: Separate injections for tibial and peroneal nerves.
    • Advantages: Provides extensive desensitization.
    • Complications: Hematoma, inadvertent vascular puncture.
  8. Intra-Articular Blocks• Usage: Directly desensitize specific joints.
    • Pathology: Joint arthritis, synovitis.
    • Localization: Specific joints (e.g., carpal, fetlock, hock).
    • Medication: Mepivacaine HCl (2%), sometimes combined with steroids.
    • Dose: Varies by joint size.
    • Technique: Sterile injection directly into the joint capsule.
    • Advantages: Highly specific.
    • Complications: Infection, joint flare.
  9. Epidural Anesthesia• Usage: Provides hindlimb analgesia.
    • Pathology: Severe hindlimb pain, dystocia.
    • Localization: Epidural space, typically at the first coccygeal interspace.
    • Medication: Mepivacaine HCl (2%), sometimes combined with opioids.
    • Dose: 10-20 mL.
    • Technique: Injection into the epidural space.
    • Advantages: Effective for hindlimb analgesia.
    • Complications: Infection, ataxia, urinary retention.

https://zukureview.com/zuku-qod/navle/1962?chosen=kV2JYGNMqa9Ox8TRt1AFOdVAmy8Je4pmL7CKypvRHHM%3D&utm_source=Zukureview+Subscribers&utm_campaign=4125963086-EMAIL_CAMPAIGN_2024_08_14_01_52&utm_medium=email&utm_term=0_1c9568dbdc-4125963086-%5BLIST_EMAIL_ID%5D

60
Q

An obese eight-year-old Labrador Retriever is presented for annual vaccinations.
The owners claim that they have restricted his diet and try to exercise him but he still keeps the weight on.
There is alopecia on the dorsum of his tail and he has an unusual worried facial expression.
Which one of the following choices is the most likely diagnosis?

A - Sarcoptic mange
B - Hyperadrenocorticism
C - Hypoadrenocorticism
D - Male pattern alopecia
E - Hypothyroidism

A

Answer: Hypothyroidism

Tail alopecia and tragic facial expression are a classic signs of hypothyroidism in dogs.

Other areas of bilateral alopecia that are common in hypothyroidism are the ventral and lateral trunk, caudal thighs, ventral neck and dorsum of the nose.

Hyperadrenocorticism commonly has truncal alopecia and also presents with obesity. It is more common in toy dog breeds.

** Comprehensive NAVLE Preparation: Hypothyroidism in Dogs**

Definitions & Etiology:

•	Hypothyroidism: A common endocrine disorder in dogs, primarily due to the destruction of the thyroid gland (primary hypothyroidism) via autoimmune lymphocytic thyroiditis or idiopathic atrophy. Secondary and tertiary hypothyroidism, caused by pituitary or hypothalamic dysfunction, are rare.
•	Primary Hypothyroidism: Accounts for most cases and is often linked to immune-mediated processes or idiopathic thyroid atrophy. Autoantibodies to thyroglobulin (TgAA) are markers of autoimmune thyroiditis.

Pathophysiology:

•	Destruction of thyroid tissue leads to decreased production of thyroid hormones (T4 and T3). This causes a reduction in metabolic rate, affecting various body systems, including the cardiovascular, nervous, reproductive, and dermatologic systems.

Signalment:

•	Most commonly affects middle-aged dogs (average age ~7 years), with certain breeds, like Golden Retrievers, Doberman Pinschers, and Beagles, showing increased susceptibility. Gender predisposition is minimal, and congenital hypothyroidism is rare.

Clinical Signs:

•	General Signs: Weight gain, lethargy, exercise intolerance, and cold intolerance. These signs are often subtle and gradual in onset.
•	Dermatologic Signs: Alopecia (often bilaterally symmetrical), seborrhea, hyperpigmentation, pyoderma, and myxedema (leading to a “tragic” facial expression).
•	Neurological Signs: Peripheral neuropathy (generalized weakness, ataxia, proprioceptive deficits) and, less commonly, central nervous system involvement (nystagmus, paresis).
•	Cardiovascular Signs: Bradycardia, weak pulses, decreased myocardial contractility, and rare cases of reversible congestive heart failure.
•	Reproductive Signs: Infertility in females, prolonged parturition, and poor neonatal outcomes. Males are less affected.

Diagnostic Tests:

•	Laboratory Findings: Common findings include hypercholesterolemia, hypertriglyceridemia, mild nonregenerative anemia, and occasionally elevated creatine kinase.
•	Thyroid Function Tests:
•	Total T4: Often the initial test; decreased in >90% of hypothyroid dogs. Not reliable in dogs with nonthyroidal illness or on certain medications.
•	Free T4 (fT4): More accurate than total T4, especially using equilibrium dialysis, less affected by nonthyroidal illness or medications.
•	TSH (Thyroid-Stimulating Hormone): Elevated in 65-75% of cases. Combining TSH with T4 or fT4 improves diagnostic accuracy.
•	Thyroid Autoantibodies: TgAA indicates autoimmune thyroiditis and is useful when thyroid function tests are equivocal.

Differential Diagnoses:

•	Conditions that mimic hypothyroidism include hyperadrenocorticism, alopecia X, metabolic and neurologic disorders, primary cardiac disease, and other endocrine disorders.

Treatment:

•	Levothyroxine: The treatment of choice, administered at 0.02 mg/kg once daily. Dosage adjustments are based on therapeutic monitoring of clinical response and serum T4 levels. Dogs with concurrent diseases (e.g., diabetes, renal failure) may require lower starting doses.
•	Therapeutic Monitoring: Clinical improvement is the primary measure, with follow-up thyroid function tests (T4 levels) performed after 6-8 weeks of treatment.

Prognosis:

•	Excellent for Most Dogs: With proper treatment, most dogs show complete recovery. Dogs with congenital hypothyroidism may have long-term complications like mental retardation or skeletal abnormalities. The prognosis is guarded for dogs with myxedema coma.

https://zukureview.com/zuku-qod/navle/1951?chosen=j9rMj3DK0cdgoRQ8lU2ryg%3D%3D&utm_source=Zukureview+Subscribers&utm_campaign=4ec80037ca-EMAIL_CAMPAIGN_2024_08_13_01_52&utm_medium=email&utm_term=0_1c9568dbdc-4ec80037ca-%5BLIST_EMAIL_ID%5D

61
Q

Where, anatomically, is the predominant abnormality located in these radiographs from an 11-year-old English sheepdog?

A - Spleen
B - Stomach
C - Liver
D - Right kidney
E - Pancreas

A

Answer: Stomach

The stomach is markedly enlarged and filled with fluid and gas. There is mineral opacity material with a sharp edge in the body and pylorus.

There are small mineral opacities (gravel sign) in the dependent portion of the stomach. The enlarged stomach is causing a mass effect and displacing the other organs caudally, however peritoneal detail is good.

The radiographs show evidence of pyloric outflow obstruction such as a mass, stricture, polyp, or granuloma.

https://vetmed.illinois.edu/imaging_anatomy/canine/ab/ex03/ab03.html

https://zukureview.com/zuku-qod/navle/1950?chosen=egOLPEU36gVtqFMf04Kf6A%3D%3D&utm_source=Zukureview+Subscribers&utm_campaign=bf327a26fb-EMAIL_CAMPAIGN_2024_08_12_01_52&utm_medium=email&utm_term=0_1c9568dbdc-bf327a26fb-%5BLIST_EMAIL_ID%5D

62
Q

A hobbyist fish owner is concerned because, within just a few hours, many of the fish in his freshwater tank have developed cloudy eyes, increased mucus, and are gasping at the surface. Fish also seem to be moving slowly.
Based on this history, which of the following would be the most relevant question to ask?

A - Were these fish treated with ivermectin, levamisole, or praziquantel?
B - Did you use chlorine or other chemicals near the tank/ equipment or do a recent water change with tap water?
C - How often do you measure ammonia levels in the tank water and what was the last reading?
D - Do you feed frozen or live brine shrimp or bloodworms, and if so, a) how often are these fed and b) are they rinsed prior to use?
E - What are the levels of vitamin A, thiamine, and vitamin E in the feed, and how do you store the feed?

A

Answer: Did you use chlorine or other chemicals near the tank/ equipment or do a recent water change with tap water?

Ask: Did you use chlorine or other chemicals near the tank/equipment or do a recent water change with tap water?

Chlorine is commonly used to disinfect tanks and equipment but is highly toxic to fish. This is a common problem because both chlorine and chloramine (type of chlorine stabilized by amination) are frequently found in municipal water sources.

Chronic and subacute chlorine exposure result in signs as described here: cloudy eyes, inflammation and necrosis of the gills, excess mucus covering gills, and lethargy. Acute chlorine toxicosis can cause sudden death.

There is a colorimetric test readily available from pool supply stores that measures chlorine (free and total); levels should be nondetectable when tested on site. If the sample of water needs to be transported for testing, use a sealed glass bottle with no air space (it may dissipate); false negatives are possible. Need to measure both free and total to get the most accurate results.

Tx with dechlorination products (often containing sodium thiosulfate) available at pet stores and then recheck free and total chlorine levels. Increase aeration because of gill compromise.

Addition of 1-3 ppt salt may also help with osmoregulatory issues.

Check out this article on
“Emergency Care of A Tancho Koi (Cyprinus carpio) Suffering From Acute Chlorine Toxicity” by Bakal et al. from North Carolina State
University.

https://www.merckvetmanual.com/exotic-and-laboratory-animals/aquatic-systems/environmental-diseases-of-aquatic-animals-in-aquatic-systems?autoredirectid=21586

https://www.vin.com/apputil/content/defaultadv1.aspx?pId=11114&catId=29070&id=3981363

63
Q

This three-year-old horse is presented for evaluation of this dry, horny, wart-like mass on the distal forelimb. This horse also has smaller wart-like lesions on its muzzle. The other young horses in the herd have similar lesions.
Which one of the following is the most likely diagnosis?

A - Pediculosis
B - Dermatophilosis
C - Trombiculosis
D - Papillomatosis
E - Melanomatosis

A

Answer: Papillomatosis

These are warts, or papillomas, which is related to infection with equine papilloma virus.

Equine papillomatosis occurs most frequently in young horses and are most commonly found on the inner surfaces of the pinna, muzzle, distal limbs, and genitalia.

They are contagious, and since they usually resolve spontaneously, treatment is rarely pursued.

These differ from equine aural plaques, which are papillomas of the inner ear associated with fly bites that do not typically self-resolve.

https://www.merckvetmanual.com/integumentary-system/tumors-of-the-skin-and-soft-tissues/epidermal-and-hair-follicle-tumors-in-animals

https://zukureview.com/zuku-qod/navle/1750?chosen=umoM3v1aLC9sEzeqnYPe0w%3D%3D&utm_source=Zukureview+Subscribers&utm_campaign=9057429c8c-EMAIL_CAMPAIGN_2024_05_13_01_51&utm_medium=email&utm_term=0_1c9568dbdc-9057429c8c-%5BLIST_EMAIL_ID%5D

64
Q

In September in the Southeastern U.S., a two-year-old Angus heifer died the previous night. She had a three-week history of poor appetite, unthriftiness, and diarrhea.
Several other younger cows in this herd also look unthrifty and have diarrhea. The cattle are vaccinated yearly and dewormed twice a year with pour-on moxidectin.
Necropsy reveals that the abomasum is edematous and covered in small umbilicated nodules, 1-2 mm in diameter (cobblestone appearance).

What is the most correct action to take next?

A - Treat affected cows with 2 classes of dewormer
B - Vaccinate against Clostridium perfringens C and D with a multivalent bacterin
C - Medicate for liver flukes
D - Provide free-choice monensin and thiamine-supplemented feed
E - Provide loose trace mineral salt to the entire group

A

Answer: ** A - Treat affected cows with 2 classes of dewormer**

The correct action is to treat affected cattle with 2 classes of dewormer. An abomasum with a cobblestone or “Moroccan leather” appearance in the fall in the southern U.S. is pathognomonic for Type I| ostertagiasis caused by Ostertagia ostertagi.

This finding in a herd with routine deworming suggests anthelmintic resistance. Current
recommendations are to deworm with two different classes of dewormer concurrently combined with a refugia program. By deworming only the affected animals (targeted treatment), the other animals serve as a source of refugia.

https://www.merckvetmanual.com/digestive-system/gastrointestinal-parasites-of-ruminants/overview-of-gastrointestinal-parasites-of-ruminants

https://www.fda.gov/media/85805/download

65
Q

What is the most common cause of iron deficiency?

A - Copper toxicosis
B - Dietary zinc deficiency
C - Immune-mediated hemolytic anemia
D - Chronic external blood loss
E - Renal disease

A

Answer: ** D - Chronic external blood loss**

Chronic external blood loss (i.e., gastrointestinal hemorrhage) is the most common cause of iron-deficiency anemia.

Dietary iron or copper deficiency or excessive zinc may also cause iron deficiency anemia.
Look for microcytic, hypochromic anemia which may be regenerative or non-regenerative. Also look for evidence of red blood cell (RBC) fragmentation - i.e., acanthocytes, schistocytes, and keratocytes.

The heme group of hemoglobin contains iron and therefore deficiency causes decreased hemoglobin production. The most common cause of iron deficiency anemia is chronic external blood loss (e.g., chronic gastrointestinal bleeding).

https://www.merckvetmanual.com/circulatory-system/anemia/nonregenerative-anemias-in-animals?mredirectid=2750

https://eclinpath.com/hematology/anemia/causes-of-anemia/

66
Q

A pale-colored Egyptian Arabian foal is presented for evaluation within a few hours of a dystocia with assisted vaginal delivery.
The foal has opisthotonos, nystagmus, limb rigidity and paddling, and has not yet stood and nursed.
What is the presumptive diagnosis based on the information available?

A - Botulism
B - Overo lethal white condition
C - Lavender foal syndrome
D - Myasthenia gravis
E - Portosystemic vascular anomaly_

A

Answer: **Lavender Foal Syndrome **

This presentation is consistent with lavender foal syndrome (LFS), an inherited lethal neurologic disorder that affects Arabian foals
with a dilute coat color.
LFS is a recessive genetic disorder and newborn foals present as in this case. Affected foals are typically euthanized within days of birth. There is no treatment.
A genetic test is available and affected animals should not be
bred.

Botulism in foals less than ~1 month old (i.e., â? ?shaker foal syndromeâ??) is characterized by progressive symmetric motor paralysis with muscular tremors.

Affected foals cannot stand for more than a few minutes at a time.

Overo lethal white syndrome is characterized by colic and meconium impaction due to ileocolonic agangliosis. It occurs in Paint foals with blue eyes that are the result breeding two Overos (a type of coat pattern in Paint horses).
Myasthenia gravis is not reported in horses.

Neurologic signs seen with hyperammonemia associated with portosystemic vascular anomalies (PSVAs, also referred to as portosystemic shunts) are not seen until affected foals are 2â??3 months old and eating more grain and forage.

https://www.ncbi.nlm.nih.gov/pmc/articles/PMC2855325/?tool=pubmed

Botulism in Animals - Detailed Veterinary Summary

Etiology and Pathogenesis

•	Causative Agent: Clostridium botulinum types A-G
•	Transmission: Ingestion of toxin in decaying carcasses or spoiled plant material; sometimes via wounds
•	Affected Species: Horses, cattle, birds, and others; variable susceptibility among species

Clinical Findings

•	Symptoms: Flaccid paralysis, progressive motor paralysis, disturbed vision, difficulty chewing/swallowing, paresis, respiratory failure
•	Lesions: No characteristic gross/histologic lesions; general paralytic action on muscles

Diagnosis

•	Tests:
•	Mouse Inoculation Test: Traditional but not always reliable
•	ELISA: Detects toxin in samples
•	Clinical Signs: Suggestive but not confirmatory
•	Differential Diagnosis: Rule out other motor paralysis causes

Treatment and Control

•	Supportive Care: Hydration, electrolyte correction, respiratory support
•	Medications:
•	Botulinum Antitoxin: Varies by type and species, e.g., 30,000 IU type B for foals
•	Prevention: High-quality feed, remove decaying materials, vaccination (types C and D toxoid)

Key Points

•	Botulism often results from toxin ingestion rather than infection
•	Diagnosis is challenging; primarily via clinical signs and ruling out other conditions
•	Effective treatment includes antitoxin administration and supportive care
•	Prevention focuses on feed quality and environmental management

https://www.merckvetmanual.com/generalized-conditions/clostridial-diseases/botulism-in-animals?mredirectid=916

https://www.merckvetmanual.com/digestive-system/congenital-and-inherited-anomalies-involving-the-digestive-system/congenital-and-inherited-anomalies-of-the-small-and-large-intestines-in-animals?autoredirectid=20437

https://www.merckvetmanual.com/digestive-system/hepatic-disease-in-large-animals/miscellaneous-hepatic-disorders-in-large-animals

67
Q

Which group is most at risk of developing the lesion evident in this image?

A - Sheep or goats pastured above 2000 meters; saanens
B - Young horses ingesting Perilla mint; standardbreds
C - Obese cats, any breed; Burmese cats
D - Old, small-breed dogs; cavalier King Charles Spaniel
E - Cattle without reticular magnets; Charolais

A

Answer: ** D - Old, small-breed dogs; cavalier King Charles Spaniel**

This is endocardiosis (a.k.a. myxomatous degeneration) of the mitral valve and older, small breed dogs are predisposed. It is an inherited disease in the cavalier King Charles spaniel and dachshund.
Note the nodular thickening of the valve margins which results in ineffective closure of the valve and resultant regurgitation of blood from the ventricle into the atrium.
Degenerative valve disease accounts for 75% of all canine cardiac disease. Approx. 60% of cases involve only the mitral valve,

30% have mitral and tricuspid lesions, and < 10% only have tricuspid involvement.

Think of myxomatous mitral valve disease in older, small-breed dogs with a heart murmur. It is an inherited disease in cavalier King Charles spaniels and dachshunds.

Don’t confuse endocardiosis with infective endocarditis which is due

to bacterial infection of the cardiac valves (typically the atrioventricular or aortic valve) and endocardial muscle.

https://www.merckvetmanual.com/circulatory-system/various-heart-diseases-in-dogs-and-cats/myxomatous-atrioventricular-valve-degeneration-in-dogs-and-cats

https://www.merckvetmanual.com/circulatory-system/various-heart-diseases-in-dogs-and-cats/infectious-endocarditis-in-dogs-and-cats

68
Q

Which one of the following choices includes the cardinal sign of trigeminal neuritis?

A - Paralyzed eyelid, ear, or lip on one or both sides of the face
B - Dysphagia, dysphonia and stridor
C - Inability to close the mouth
D - Masseter muscle pain associated with chewing.
E - Circling and head tilt toward side of lesion, no other signs

A

Answer: ** B - Dysphagia, dysphonia and stridor**

Idiopathic trigeminal neuritis, due to inflammation of cranial nerve 5
(CN 5), is characterized by acute onset of flaccid jaw paralysis.
Affected animals cannot close their mouth and have difficulty eating and drinking. Seen occasionally in dogs, rare in cats.

Idiopathic facial nerve paralysis, affecting cranial nerve 7 (CN 7), results in the inability to move the eyelid, lip, or ear and dryness of the eyes and mouth.

Masticatory myositis is characterized by pain on opening the mouth and swelling of the
muscles of mastication (acute) or atrophy of the temporalis and masseter muscles with the inability to open the mouth due to fibrosis (chronic).

Dysphagia, dsyphonia, and stridor are most often associated with
dysfunction of the vagus nerve, cranial nerve 10 (CN 10).
Circling and head tilt toward the side of the lesion with no other signs is a common presentation of vestibulocochlear nerve, cranial nerve 8 (CN 8) lesions.

Concurrent CN 7 paralysis and Horner syndrome (ptosis, miosis, enophthalmos) may be present with middle- and inner-ear infections.

** Trigeminal Neuritis in Dogs and
Cats**

Idiopathic trigeminal neuropathy is common in dogs and uncommon in cats. It is characterized by acute onset of flaccid jaw paralysis. Affected animals cannot close the mouth and have difficulty eating and drinking. Horner syndrome, facial paresis, and decreased facial sensation are also possible. The cause is unknown.
Histopathologically, there is bilateral nonsuppurative inflammation and demyelination in the motor branches of the trigeminal nerve. Affected animals usually recover spontaneously within 3-4 weeks. Fluid therapy and nutritional support may be necessary.

https://www.merckvetmanual.com/nervous-system/diseases-of-the-peripheral-nerves-and-neuromuscular-junction/inflammatory-disorders-of-the-peripheral-nerves-and-neuromuscular-junction-in-animals#Sensory-Ganglioneuritis_v26304956

** Facial Paralysis in Animals**

1.	Definitions and Causative Agents:
•	Facial Paralysis: Involves paralysis of muscles of facial expression due to lesions of the facial nerve (cranial nerve VII).
•	Causes: Trauma, otitis media, guttural pouch infections, hypothyroidism, neoplasia, idiopathic.
2.	Physiopathology:
•	Lesions affecting the facial nerve or its nucleus in the brainstem cause dysfunction in facial muscle movement, tear and saliva production.
3.	Symptoms and Clinical Changes:
•	Inability to blink, drooping lips, ear droop, reduced tear production, deviation of the nose, drooling, food accumulation in the mouth.
4.	Diagnosis:
•	Based on clinical signs, neurologic examination, otoscopic examination, thyroid testing, imaging (CT/MRI), and possibly CSF analysis for infections.
5.	Treatment and Medications:
•	Underlying Cause: Address primary condition (e.g., antibiotics for infections, surgery for neoplasia).
•	Supportive Care: Artificial tears, manage corneal ulcers, corrective surgery for nostril collapse.
•	Medications: Antibiotics for infections (based on culture), thyroid supplements for hypothyroidism, corticosteroids for inflammation.
6.	Prognosis:
•	Variable, depending on etiology. Early treatment increases recovery chances. Idiopathic cases may partially recover.

https://www.merckvetmanual.com/nervous-system/facial-paralysis/facial-paralysis-in-animals?autoredirectid=14324

https://www.merckvetmanual.com/musculoskeletal-system/myopathies-in-small-animals/masticatory-myositis-in-dogs-and-cats

69
Q

A hobbyist fish owner is concerned because, within just a few hours, many of the fish in his freshwater tank have developed cloudy eyes, increased mucus, and are gasping at the surface. Fish also seem to be moving slowly.
Based on this history, which of the following would be the most relevant question to ask?

A - Were these fish treated with ivermectin, levamisole, or praziquantel?
B - Did you use chlorine or other chemicals near the tank/ equipment or do a recent water change with tap water?
C - How often do you measure ammonia levels in the tank water and what was the last reading?
D- Do you feed frozen or live brine shrimp or bloodworms, and if so, a) how often are these fed and b) are they rinsed prior to use?
E - What are the levels of vitamin A, thiamine, and vitamin E in the feed, and how do you store the feed?

A

Answer: ** B - Did you use chlorine or other chemicals near the tank/ equipment or do a recent water change with tap water?**

Ask: Did you use chlorine or other chemicals near the
tank/equipment or do a recent water change with tap water?
Chlorine is commonly used to disinfect tanks and equipment but is highly toxic to fish. This is a common problem because both chlorine and chloramine (type of chlorine stabilized by amination) are frequently found in municipal water sources.

Chronic and subacute chlorine exposure result in signs as described here: cloudy eyes, inflammation and necrosis of the gills, excess mucus covering gills, and lethargy. Acute chlorine toxicosis can cause sudden death.

There is a colorimetric test readily available from pool supply stores that measures chlorine (free and total); levels should be nondetectable when tested on site. If the sample of water needs to be transported for testing, use a sealed glass bottle with no air space (it may dissipate); false negatives are possible. Need to measure both free and total to get the most accurate results.
Tx with dechlorination products (often containing sodium
thiosulfate) available at pet stores and then recheck free and total
chlorine levels. Increase aeration because of gill compromise.

Addition of 1-3 ppt salt may also help with osmoregulatory issues.

Check out this article on
“Emergency Care of A Tancho Koi (Cyprinus carpio) Suffering From Acute Chlorine Toxicity” by Bakal et al. from North Carolina State
University.

https://www.vin.com/apputil/content/defaultadv1.aspx?pId=11114&catId=29070&id=3981363

70
Q

The calf shown below was presented with ataxia, intention tremors, and hypermetria; all signs were present since birth.
The second image shows a normal bovine calf brain on the left and the brain from this calf on the right.
Which one of the following choices is the most likely cause of the disorder in this calf?

A - Bovine viral diarrhea virus
B - Copper deficiency
C - Border disease
D - Caudal occipital malformation syndrome
E - Organophosphate toxicity

A

Answer: Bovine Viral Diarrhea Virus

Bovine viral diarrhea virus (BVD).

In utero infection with BVD is a common cause of cerebellar hypoplasia in cattle.
Other causes are bluetongue, Akabane, or Wesselsbron viruses.

Caudal occipital malformation syndrome is a disorder seen mainly in Cavalier King Charles
Spaniels.

The malformed occipital bone causes crowding at the caudal fossa and cerebellar herniation at the foramen magnum.

Comprehensive Information on Bovine Viral Diarrhea (BVD) for NAVLE Preparation

Definition & Etiology:

•	Bovine Viral Diarrhea (BVD): A disease caused by Bovine viral diarrhea virus (BVDV), a Pestivirus in the Flaviviridae family.
•	Transmission: Mainly through direct contact with infected animals or contaminated fomites. Vertical transmission can lead to persistently infected (PI) calves.

Pathophysiology:

•	BVDV can cause immunosuppression, leading to secondary infections.
•	Cerebellar Hypoplasia: In utero infection during mid-gestation can result in cerebellar hypoplasia in calves, characterized by ataxia, tremors, and inability to stand properly.

Clinical Signs:

•	Diarrhea, fever, nasal discharge, immunosuppression, and mucosal disease in PI animals.
•	Neurological signs in calves with cerebellar hypoplasia include ataxia, intention tremors, and hypermetric gait.

Diagnosis:

•	PCR and virus isolation are used to detect BVDV.
•	Serology to identify antibodies in the dam or calf.

Prevention:

•	Vaccination: Prophylactic vaccination of breeding animals is essential to prevent vertical transmission and cerebellar hypoplasia in calves.

Treatment:

•	No specific antiviral treatment; focus on supportive care and prevention of secondary infections.

https://www.merckvetmanual.com/nervous-system/congenital-and-inherited-anomalies-of-the-nervous-system/congenital-and-inherited-cerebellar-disorders-in-animals?redirectid=30646

71
Q

A 12-week-old chicken from a young backyard flock is presented for evaluation because of weight loss, decreased appetite, and diarrhea.
The irises in the affected bird are lighter than normal with irregular pupillary margins, and the hen does not have a normal pupillary light reflex. Several other young birds appear to be growing poorly compared to the rest of the flock.
The image below is an example of what the affected bird’s eye looks like, vs. a normal eye on the left.
Based on the presumptive diagnosis, how can this problem be prevented going forward?

A - Isolate unaffected animals; breed from survivors of disease
B - Use only_ mycoplasma-resistant bloodlines for new additions to flock
C - Buy only vaccinated chicks
D - Cull birds from the same genetic line
E - Collect pharyngeal swabs from affected birds; Tx for entire flock with antimicrobials based on culture and sensitivity

A

Answer: Buy only vaccinated chicks

Buy only chicks vaccinated against
Marek disease. This chicken has ocular evidence of Marek disease.

There is no treatment, but the problem is preventable when chicks are vaccinated at hatching or before 2 weeks of age.
Diffuse infiltration of mononuclear cells can turn the irises of affected chickens from a normal yellow color to a pale tan to gray color (a.k.a. “gray eye”). Cellular infiltration can also occur in many other tissues (skin, nerves, viscera), and there may be palpable tumors in the muscle and skin.

Marek disease is common in commercial poultry. Dx usually based on enlarged nerves and lymphoid tumors in the viscera.
Lymphoid leukosis is a key differential.

Absence of bursal tumors helps distinguish Marek disease from leukosis. But if you find bursal tumors, Marek disease is still on the DDX list pending other tests, like immunochemistry. Chicks as young as 3 weeks can show Marek disease, but lymphoid leukosis is typically seen in chickens more than 14 weeks old.

** Comprehensive Information on Marek’s Disease for NAVLE Preparation**

Definition & Etiology:

•	Marek’s Disease (MD): A highly contagious viral disease in chickens caused by an alphaherpesvirus known as Marek’s disease virus (MDV). It leads to the formation of tumors and affects the nervous system.

Pathophysiology:

•	MDV infects lymphocytes, leading to the development of T-cell lymphomas. The virus causes immunosuppression and can also result in neurological damage, leading to paralysis.

Clinical Signs:

•	Progressive paralysis, especially of the legs and wings.
•	Ocular form causes irregularly shaped pupils and blindness.
•	Cutaneous form manifests as skin nodules.

Diagnosis:

•	Based on clinical signs, postmortem findings of enlarged nerves, and histopathology showing lymphocytic infiltration.

Prevention:

•	Vaccination: Administered in ovo or at hatch, providing lifelong immunity. Biosecurity measures are also critical to prevent the spread of MDV.

https://www.merckvetmanual.com/exotic-and-laboratory-animals/backyard-poultry/common-infectious-diseases-in-backyard-poultry

https://www.merckvetmanual.com/poultry/neoplasms-in-poultry/marek-s-disease-in-poultry?autoredirectid=23789&autoredirectid=23789&autoredirectid=16879#Control_v3342456

https://www.merckvetmanual.com/poultry/neoplasms-in-poultry/avian-leukosis-in-poultry?autoredirectid=23791

https://zukureview.com/zuku-qod/navle/1984?chosen=hO3J/VsV5gKHka9efFXLsuWBtiF2Wy48rVT5xmlvUyFpZB4rAyfp8KPbuHyEbpZN4zWd27P7/%2BJ5bEl0B4w9yg%3D%3D&utm_source=Zukureview+Subscribers&utm_campaign=b3c53e896a-EMAIL_CAMPAIGN_2024_08_28_01_51&utm_medium=email&utm_term=0_1c9568dbdc-b3c53e896a-%5BLIST_EMAIL_ID%5D

72
Q

Several 12-month-old sheep and goats are sick at a petting zoo that has cows, horses, pigs, bison, and white-tailed deer. One of the deer is also affected.
The rest of the animals appear well.
Affected sheep are febrile, listless, and off-feed, with serous or mucopurulent nasal discharge and rectal temperatures ranging from
105-107.5ºF (40.5-42°C) [N=100.9-103.8ºF (38.3-39.9°C)].
Physical exam shows swollen muzzles with edema and congestion of the lips, nose, and face with small hemorrhages and ulcers on the mucous membranes.
The ulcers appear where the teeth contact the swollen lips and tongue.
Two affected animals are lame.
What is the diagnosis?

A - Contagious ecthyma
B - Caprine arthritis encephalitis (CAE).
C - Ovine progressive pneumonia (OPP).
D - Bluetongue
E - Peste des petits ruminants (PPR).

A

Answer: Blue tongue

Swollen, sore muzzles with mucous membrane erosions, high fevers, and lameness suggests infection with bluetongue virus.

Signs are worse in young lambs (up to 30% mortality). Remember infection of pregnant dams produces fetuses with hydrancephaly or porencephaly causing ataxic and blind lambs.
Bluetongue is almost exclusively seen in sheep, though white-tailed deer, pronghorn antelope, and desert bighorn sheep in North America can be severely affected.

Rare in cows. REPORTABLE.

There are at least 24 serotypes of the bluetongue virus (genus
Orbivirus in the family Reoviridae). Worldwide
distribution mimics its vector’s
(Culicoides biting midges)
distribution. Dx: serology (ELISA or AGID), virus isolation from blood of febrile animals, and PCR to identify the specific isolate.

Treatment is supportive care.
Prevention relies on controlling vectors and vaccination, the latter of which is used in endemic regions (but no cross-protection between different serotypes) or during outbreaks in non-endemic regions.

Bluetongue is clinically indistinguishable from foot and mouth disease (FMD), so that is a good second choice. But FMD is unlikely in the scenario above because FMD mainly affects pigs and cattle.

Ovine progressive pneumonia
(OPP is a chronic disease of wasting and dyspnea in sheep and goats, caused by a lentivirus (retrovirus family). OPP is most common in animals older than four years and rarely occurs in sheep and goats under two years of age.

** Bluetongue in Ruminants - Comprehensive Study Guide**

Definition

•	Bluetongue: A viral disease of ruminants caused by Bluetongue virus (BTV), affecting sheep, cattle, and other ruminants.

Causative Agents

•	Bluetongue Virus (BTV): An Orbivirus in the family Reoviridae, with at least 29 serotypes worldwide.

Pathophysiology

•	Transmission: Primarily by Culicoides biting midges. The virus infects endothelial cells, causing vascular damage, hemorrhage, and edema.
•	Lifecycle: Virus spreads through local lymph nodes, leading to viremia and systemic spread.

Symptoms

•	Sheep: Fever, edema of lips, tongue, face, nasal discharge, oral ulcers, lameness, and death in severe cases.
•	Cattle: Generally mild signs, fever, oral vesicles, and ulcers, increased respiratory rate, and stiffness.
•	Other Ruminants: Similar signs as in sheep and cattle, with occasional severe hemorrhagic disease.

Diagnosis

•	Clinical Signs: Edema, hemorrhages, and necrosis in characteristic locations.
•	Laboratory Tests: Virus isolation, PCR, serologic tests (AGID, competitive ELISA).

Treatment

•	Supportive Care: Rest, soft food, good husbandry, and treatment of secondary infections.
•	Control: Vaccination in endemic areas, vector control using insecticides, and management practices to reduce exposure to midges.

Prevention

•	Vaccination: Attenuated and inactivated vaccines available, use during non-vector seasons to avoid transmission of vaccine strains.
•	Vector Control: Reducing midge bites through insecticides and protective measures.

https://www.merckvetmanual.com/generalized-conditions/bluetongue/bluetongue-in-ruminants?autoredirectid=17866

https://www.merckvetmanual.com/generalized-conditions/foot-and-mouth-disease/foot-and-mouth-disease-in-animals

https://zukureview.com/zuku-qod/navle/1983?chosen=9LEARvvRrESZW%2Bls3Vkge%2BbCBdMnHYlcXw2nRw6pT60%3D&utm_source=Zukureview+Subscribers&utm_campaign=e11519ea73-EMAIL_CAMPAIGN_2024_08_27_01_51&utm_medium=email&utm_term=0_1c9568dbdc-e11519ea73-%5BLIST_EMAIL_ID%5D

73
Q

A five-year-old boxer is presented for routine vaccinations.
The owner points out a small, smooth mass on the right ear. Fine needle aspiration cytology is shown below.
Which one of the following choices is the most likely diagnosis?

A - Aural hematoma
B - Bacterial infection
C - Histiocytoma
D - Degranulating mast cell tumor
E - Lymphoma

A

Answer: Bacterial infection

Bacterial infection.
There are both intracellular and extracellular bacteria present in this cytology.
The abscess should be drained and the animal placed on appropriate antibiotics.
Mast cell tumors are round cell tumors with basophilic granules that are smaller than bacteria.

https://www.merckvetmanual.com/emergency-medicine-and-critical-care/wound-management-in-small-animals/drains-in-wound-management-of-small-animals?mredirectid=821

https://www.merckvetmanual.com/integumentary-system/tumors-of-the-skin-and-soft-tissues/lymphocytic-histiocytic-and-related-cutaneous-tumors-in-animals

74
Q

A 13-week-old puppy is presented for trauma after being attacked by another dog.
After stabilization, a series of radiographs show various issues (see below).
Which one of the following choices is the best next step to do for this patient?

zukureview.com

A - Thoracocentesis and chest tube
B - Computerized tomography
C - Aspirate subcutaneous emphysema and apply snug body wrap
D - Suture lacerations and insert drains
E - Place an intravenous catheter and start IV fluids

A

Answer: Thoracocentesis and chest tube

This puppy has a pneumothorax and needs immediate thoracocentesis and chest tube placement. The left hemithorax appears gas distended with marked rightward mediastinal shift.

A large, loculated area of subcutaneous emphysema is seen over the caudal dorsolateral thorax; however, a communication with the thoracic cavity or the external body wall cannot be identified.

A triangular wedge of soft tissue opacity is seen in the mid left thorax and is consistent with collapsed left lung.

A smaller, poorly demarcated area of increased soft tissue opacity is also seen superimposed over the 10th left rib adjacent to the body
wall at the level of the most severe subcutaneous emphysema. There is mild pleural effusion in the right hemithorax.

The right pulmonary parenchyma is difficult to evaluate, but appears to be within normal limits. The musculoskeletal structures appear within normal limits.

This is severe left sided tension pneumothorax with collapse of the left lung lobes.

Click here to see normal canine thoracic radiographs Radiographic interpretation and images courtesy, Dr A. Zwingenberger and Veterinary Radiology. Normal radiograph links courtesy, Imaging Anatomy Univ. of Illinois Vet Med.

Trauma in Emergency Medicine in Small Animals - Comprehensive Veterinary Information

Definitions and Terminology:
- Trauma: Physical injuries caused by external forces.
- Blunt Trauma: Associated with bleeding, organ rupture, fractures, and neurologic injuries.
- Penetrating Trauma: Localized injuries from objects.

Causes:
- Blunt Trauma: Car accidents, falls.
- Penetrating Trauma: Bites, sharp objects.

Clinical Findings:
- Thoracic Trauma: Pulmonary contusions, pneumothorax, rib fractures, diaphragmatic hernia.
- Abdominal Trauma: Hemorrhage, organ rupture, herniation.
- Neurologic Injuries: Spinal fractures, head trauma.

Physiopathology:
- Initial Response: Airway, breathing, circulation assessment; control hemorrhage, pain relief.
- Secondary Survey: Evaluate nervous system, thorax, abdomen, integument, musculoskeletal systems.

Assessment:
- Physical Examination: Palpation for pain, fluid, hernias; auscultation for lung sounds.
- Diagnostic Imaging: Radiographs, ultrasound, CT for internal injuries.

Treatment:
- Fluid Resuscitation: Isotonic crystalloids, blood products.
- Analgesia: Pain management.
- Surgical Intervention: Indications include ongoing hemorrhage, organ rupture.

Monitoring:
- Vital Signs: Temperature, pulse, respiration, blood pressure.
- Laboratory Tests: PCV, total solids, blood gases, electrolytes, lactate.

Medications:
- Antibiotics: For penetrating wounds.
- Antiarrhythmics: Lidocaine for cardiac arrhythmias.

Prognosis:
- Severity Dependent: Based on initial injury severity, timely intervention, and ongoing monitoring.

For more detailed information, visit the Merck Veterinary Manual.

https://vetmed.illinois.edu/imaging_anatomy/canine/thorax/ex02/thorax02.html

75
Q

A 13-week-old puppy is presented for trauma after being attacked by another dog.
After stabilization, a series of radiographs show various issues (see below).
Which one of the following choices is the best next step to do for this patient?

zukureview.com

A - Thoracocentesis and chest tube
B - Computerized tomography
C - Aspirate subcutaneous emphysema and apply snug body wrap
D - Suture lacerations and insert drains
E - Place an intravenous catheter and start IV fluids

A

Answer: Thoracocentesis and chest tube

This puppy has a pneumothorax and needs immediate thoracocentesis and chest tube placement. The left hemithorax appears gas distended with marked rightward mediastinal shift.

A large, loculated area of subcutaneous emphysema is seen over the caudal dorsolateral thorax; however, a communication with the thoracic cavity or the external body wall cannot be identified.

A triangular wedge of soft tissue opacity is seen in the mid left thorax and is consistent with collapsed left lung.

A smaller, poorly demarcated area of increased soft tissue opacity is also seen superimposed over the 10th left rib adjacent to the body
wall at the level of the most severe subcutaneous emphysema. There is mild pleural effusion in the right hemithorax.

The right pulmonary parenchyma is difficult to evaluate, but appears to be within normal limits. The musculoskeletal structures appear within normal limits.

This is severe left sided tension pneumothorax with collapse of the left lung lobes.

Click here to see normal canine thoracic radiographs Radiographic interpretation and images courtesy, Dr A. Zwingenberger and Veterinary Radiology. Normal radiograph links courtesy, Imaging Anatomy Univ. of Illinois Vet Med.

Trauma in Emergency Medicine in Small Animals - Comprehensive Veterinary Information

Definitions and Terminology:
- Trauma: Physical injuries caused by external forces.
- Blunt Trauma: Associated with bleeding, organ rupture, fractures, and neurologic injuries.
- Penetrating Trauma: Localized injuries from objects.

Causes:
- Blunt Trauma: Car accidents, falls.
- Penetrating Trauma: Bites, sharp objects.

Clinical Findings:
- Thoracic Trauma: Pulmonary contusions, pneumothorax, rib fractures, diaphragmatic hernia.
- Abdominal Trauma: Hemorrhage, organ rupture, herniation.
- Neurologic Injuries: Spinal fractures, head trauma.

Physiopathology:
- Initial Response: Airway, breathing, circulation assessment; control hemorrhage, pain relief.
- Secondary Survey: Evaluate nervous system, thorax, abdomen, integument, musculoskeletal systems.

Assessment:
- Physical Examination: Palpation for pain, fluid, hernias; auscultation for lung sounds.
- Diagnostic Imaging: Radiographs, ultrasound, CT for internal injuries.

Treatment:
- Fluid Resuscitation: Isotonic crystalloids, blood products.
- Analgesia: Pain management.
- Surgical Intervention: Indications include ongoing hemorrhage, organ rupture.

Monitoring:
- Vital Signs: Temperature, pulse, respiration, blood pressure.
- Laboratory Tests: PCV, total solids, blood gases, electrolytes, lactate.

Medications:
- Antibiotics: For penetrating wounds.
- Antiarrhythmics: Lidocaine for cardiac arrhythmias.

Prognosis:
- Severity Dependent: Based on initial injury severity, timely intervention, and ongoing monitoring.

For more detailed information, visit the Merck Veterinary Manual.

https://vetmed.illinois.edu/imaging_anatomy/canine/thorax/ex02/thorax02.html

76
Q

A hobbyist fish owner is concerned because, within just a few hours, many of the fish in his freshwater tank have developed cloudy eyes, increased mucus, and are gasping at the surface. Fish also seem to be moving slowly.
Based on this history, which of the following would be the most relevant question to ask?

A - Were these fish treated with ivermectin, levamisole, or praziquantel?
B - Did you use chlorine or other chemicals near the tank/ equipment or do a recent water change with tap water?
C - How often do you measure ammonia levels in the tank water and what was the last reading?
D - Do you feed frozen or live brine shrimp or bloodworms, and if so, a) how often are these fed and b) are they rinsed prior to use?
E - What are the levels of vitamin A, thiamine, and vitamin E in the feed, and how do you store the feed?

A

Answer: ** B - Did you use chlorine or other chemicals near the tank/ equipment or do a recent water change with tap water?**

Ask: Did you use chlorine or other chemicals near the tank/equipment or do a recent water change with tap water?

Chlorine is commonly used to disinfect tanks and equipment but is highly toxic to fish. This is a common problem because both chlorine and chloramine (type of chlorine stabilized by amination) are frequently found in municipal water sources.

Chronic and subacute chlorine exposure result in signs as described here: cloudy eyes, inflammation and necrosis of the
gills, excess mucus covering gills, and lethargy. Acute chlorine toxicosis can cause sudden death.

There is a colorimetric test readily available from pool supply stores that measures chlorine (free and total); levels should be nondetectable when tested on site. If the sample of water needs to be transported for testing, use a sealed glass bottle with no air space (it may dissipate); false negatives are possible. Need to measure both free and total to get the most accurate results.
Tx with dechlorination products (often containing sodium
thiosulfate) available at pet stores and then recheck free and total
chlorine levels. Increase aeration because of gill compromise.

Addition of 1-3 ppt salt may also help with osmoregulatory issues.

Check out this article on
“Emergency Care of A Tancho Koi (Cyprinus carpio) Suffering From Acute Chlorine Toxicity” by Bakal et al. from North Carolina State
University.

https://zukureview.com/zuku-qod/navle/1949?chosen=5cewrhrCdz9kJT3k0XUM18K/llgpQxfC/Z1kpaEv%2BJy5gFTklxnnQ6%2BpLCwAfM2YNFskNWyZEvvmyGttECXtd5qtDguS/rmYiTtp2eiYHPs%3D&utm_source=Zukureview+Subscribers&utm_campaign=43bbd5ca1c-EMAIL_CAMPAIGN_2024_08_09_01_52&utm_medium=email&utm_term=0_1c9568dbdc-43bbd5ca1c-%5BLIST_EMAIL_ID%5D

77
Q

A four-month-old calf from a small beef herd in Louisiana presents with a fracture of the right metatarsus. He was normal last night. The owner found the calf this morning after rounding up the herd for deworming. This is the fifth calf this season with fractured bones.
One had a vertebral fracture, two calves had humeral fractures, and one other had a cannon bone fracture.

Which one of the following choices is the most likely problem in this herd?

A - Hypovitaminosis D
B - Low dietary protein
C - Low dietary copper
D - High dietary calcium
E - High dietary phosphorus

A

Answer: Low dietary copper

Low dietary copper levels (Cu) causes bones of growing animals to be brittle because they do not develop the normal trabecular
structure.

Higher fracture rates are seen in Cu deficient than in Cu replete animals.

Pathologic fractures are more common during periods of exertion, such as the morning round up. Rough handling exacerbates the number of
fractures.

Treatment of Cu deficiency includes copper glycinate injections, and feeding of mineral mixes with Cu. Copper boluses can also be given.

In areas with high levels of molybdenum (Mb) in the soil, Cu levels are often low. Mb interferes with absorption of Cu, hence Mb excess results in Cu deficiency.

https://www.merckvetmanual.com/management-and-nutrition/nutrition-beef-cattle/nutrient-requirements-of-beef-cattle?autoredirectid=22126

https://www.merckvetmanual.com/management-and-nutrition/nutrition-beef-cattle/nutrient-requirements-of-beef-cattle?autoredirectid=22126

https://zukureview.com/zuku-qod/navle/1948?chosen=bvwW297qRVQpH4PHNu64IUKmTXfRGirgYkxSdB%2BxUx4%3D&utm_source=Zukureview+Subscribers&utm_campaign=9eaee92738-EMAIL_CAMPAIGN_2024_08_08_01_52&utm_medium=email&utm_term=0_1c9568dbdc-9eaee92738-%5BLIST_EMAIL_ID%5D

78
Q

A four-month-old calf from a small beef herd in Louisiana presents with a fracture of the right metatarsus. He was normal last night. The owner found the calf this morning after rounding up the herd for deworming. This is the fifth calf this season with fractured bones.
One had a vertebral fracture, two calves had humeral fractures, and one other had a cannon bone fracture.

Which one of the following choices is the most likely problem in this herd?

A - Hypovitaminosis D
B - Low dietary protein
C - Low dietary copper
D - High dietary calcium
E - High dietary phosphorus

A

Answer: Low dietary copper

Low dietary copper levels (Cu) causes bones of growing animals to be brittle because they do not develop the normal trabecular
structure.

Higher fracture rates are seen in Cu deficient than in Cu replete animals.

Pathologic fractures are more common during periods of exertion, such as the morning round up. Rough handling exacerbates the number of
fractures.

Treatment of Cu deficiency includes copper glycinate injections, and feeding of mineral mixes with Cu. Copper boluses can also be given.

In areas with high levels of molybdenum (Mb) in the soil, Cu levels are often low. Mb interferes with absorption of Cu, hence Mb excess results in Cu deficiency.

https://www.merckvetmanual.com/management-and-nutrition/nutrition-beef-cattle/nutrient-requirements-of-beef-cattle?autoredirectid=22126

https://www.merckvetmanual.com/management-and-nutrition/nutrition-beef-cattle/nutrient-requirements-of-beef-cattle?autoredirectid=22126

https://zukureview.com/zuku-qod/navle/1948?chosen=bvwW297qRVQpH4PHNu64IUKmTXfRGirgYkxSdB%2BxUx4%3D&utm_source=Zukureview+Subscribers&utm_campaign=9eaee92738-EMAIL_CAMPAIGN_2024_08_08_01_52&utm_medium=email&utm_term=0_1c9568dbdc-9eaee92738-%5BLIST_EMAIL_ID%5D

79
Q

A pig farm has several sick and dying adult pigs.
Affected animals are febrile and depressed.
Some seem constipated and others have diarrhea. A few are ataxic.

A necropsy on one of the dead pigs shows
widespread petechial and ecchymotic hemorrhages in the kidneys (“turkey egg kidneys”), bladder, spleen, and larynx.
Which one of the following choices is the most likely diagnosis?

A - Postweaning multisystemic wasting syndrome
B - Anthrax
C - Glaesserella parasuis (formerly Haemophilus parasuis).
D - Mycoplasma hyopneumoniae
E - Classical swine fever

A

Answer: Classical swine fever

Hemorrhages on the kidneys and other organs are characteristic of both classical swine fever (CSF, also called “hog cholera”) and African swine fever (ASF).

African swine fever cannot be differentiated from classical swine fever based on clinical and postmortem signs alone.

Confirmation is based on either
PCR or ELISA antigen testing.

Both are reportable diseases.

Classical swine fever was last reported in North America in the 1970s.

Click here to see more images of CSF.
Erysipelas is characterized by fever, painful joints, and sometimes, urticarial diamond-shaped skin lesions.

Glaesserella parasuis (formerly Haemophilus parasuis; cause of Glasser’s disease) occurs mainly in young piglets.

Pigs with postweaning multisystemic wasting syndrome (PMWS) typically have enlarged, pale lymph nodes, growth retardation, wasting, and dyspnea.

Mycoplasma hyopneumoniae is a common cause of pneumonia in adults pigs; present with a barking cough and slow growth.

Ref: Jackson and Cockcroft, Handbook of Pig Medicine, pp. 182-84 and Zimmerman et al., Diseases of Swine, 11th ed.

Images courtesy, USDA and the Center for Food Security and Public Health.

https://zukureview.com/zuku-qod/navle/1947?chosen=%2BJeZ2rEjQ1PM4SJWhaNqQuiQGe/i2YbcgwM8J5MckNZHee9yrwsZj1FkO2D0/GQM&utm_source=Zukureview+Subscribers&utm_campaign=7bd7bc98ab-EMAIL_CAMPAIGN_2024_08_07_01_52&utm_medium=email&utm_term=0_1c9568dbdc-7bd7bc98ab-%5BLIST_EMAIL_ID%5D

** Classical Swine Fever (CSF) Comprehensive Overview**

Etiology and Pathogenesis

•	Causative Agent: Classical swine fever virus (CSFV), an enveloped RNA virus in the genus Pestivirus, family Flaviviridae.
•	Pathogenesis:
•	Infects pigs via direct contact, contaminated feed, or fomites.
•	Virus replicates in tonsils and lymph nodes, spreading through the bloodstream.
•	Causes vasculitis leading to hemorrhages and necrosis.

Epidemiology

•	Distribution: Endemic in Central and South America, the Caribbean, and parts of Asia. Free in Australia, New Zealand, Canada, the US, and some European countries.
•	Transmission:
•	Direct contact with infected pigs or pork products.
•	Wild boars and Suidae family members are reservoirs.
•	Can survive in pork products for extended periods, especially when frozen.

Clinical Signs and Lesions

•	Acute Form:
•	Incubation: 3-7 days.
•	Symptoms: High fever (>41°C), hemorrhages, lethargy, diarrhea, vomiting, purple skin discoloration.
•	Lesions: Petechial hemorrhages, splenic infarction, “button” ulcers in the intestine.
•	Chronic Form:
•	Symptoms: Prolonged fever, weight loss, diarrhea, skin discoloration.
•	Lesions: Atrophy of the thymus, depletion of lymphoid follicles.
•	Subclinical Form: Poor reproductive performance, congenital tremors in piglets.

Diagnosis

•	RT-PCR and RT-qPCR: Detect viral RNA in tissues, blood, serum, or oronasal fluids.
•	Antigen Detection: Immunofluorescence on tissue sections, ELISA for herd screening.
•	Serological Tests: ELISA, virus neutralization assays for detecting antibodies.
•	Virus Isolation: In cell cultures using tissue suspensions or fluids from suspected cases.

Differential Diagnosis

•	Similar Diseases: African swine fever (ASF), other hemorrhagic diseases, systemic infections.

Control and Prevention

•	No Treatment Available.
•	Vaccination:
•	Live, attenuated vaccines (LAVs) provide rapid immunity.
•	Oral vaccines for wild boars.
•	New marker vaccines allow differentiation of infected from vaccinated animals (DIVA).
•	Biosecurity Measures: Prohibit swill feeding, stringent farm hygiene, and control of animal movements.
•	Outbreak Response: Quarantine and culling of exposed herds, emergency vaccination in ring around outbreaks.

Key Points for Veterinary Professionals

•	CSF is a highly infectious, often fatal viral disease of swine.
•	Early diagnosis, biosecurity, and vaccination are critical for control.
•	Differentiation from similar diseases is necessary for accurate diagnosis.

** African Swine Fever (ASF), comprehensive overview:**

Etiology and Pathogenesis

•	Causative Agent: ASF virus (ASFV), a large DNA virus in the Asfarviridae family.
•	Pathogenesis: Infection through contact, ingestion, or tick bites. It replicates in macrophages and causes systemic infection, leading to hemorrhagic fever.

Clinical Signs and Lesions

•	Acute Form: High fever, cyanosis, hemorrhages, vomiting, diarrhea, and sudden death.
•	Chronic Form: Weight loss, respiratory issues, joint swelling.

Diagnosis

•	Laboratory Tests: PCR, ELISA, virus isolation from blood, spleen, or lymph nodes.
•	Necropsy Findings: Hemorrhages in organs, enlarged spleen, and lymph nodes.

Treatment and Control

•	No treatment; focus on biosecurity, quarantine, and culling.
•	Prevention: Strict biosecurity measures, including movement control, cleaning, and disinfection.
80
Q

A farmer recently purchased a sheep from a salebarn. The sheep soon became ill, with progressive depression, lethargy, and inappetance.

He had respiratory problems, breathing hard with abdominal effort, and died after three days with a frothy nasal discharge just prior to death.
At necropsy, the lungs had the appearance shown below.

What was the most likely cause of death?

A - Perilla frutescens
B - Ovine progressive pneumonia
C - Dictyocaulus filaria
D - Corynebacterium pseudotuberculosis
E - Mannheimia haemolytica

A

Answer: ** E - Mannheimia haemolytica**

Mannheimia haemolytica
pneumonia typically has a cranioventral distribution - as demonstrated by the hepatized lung (dark red, liver-like) seen in the bottom half of this image. M. haemolytica and Pasteurella multocida are commensals in the upper airway of sheep.

Mannheimia haemolytica of sheep and goats is a different strain (type A2) from that in cattle (type A1).

Infection is more likely if there is a primary viral (e.g., parainfluenza-3) or bacterial (e.g., Mycoplasma ovipneumoniae) infection.
Pneumonia with these pathogens is most common after recent stress or comingling with new animals. Prevention involves decreasing stress and exposure.

There are currently no vaccines available for respiratory disease in sheep and goats and cattle vaccines have not been shown to be effective.

Ovine progressive pneumonia causes chronic wasting with respiratory distress and occasional indurative mastitis.

Necropsy findings include lungs with mottled gray and brown areas of consolidation and very enlarged mediastinal and tracheobronchial lymph nodes.

Corynebacterium pseudotuberculosis causes caseous lymphadenitis in sheep and goats.
Multifocal abscesses would be seen on necropsy.

Click here to see
C. pseudotuberculosis hepatic abscesses.
Dictyocaulus filaria can cause bronchitis or pneumonia in sheep and infection is characterized by tachypnea and coughing.
Hepatization of the lungs would be an uncommon finding at necropsy.

Perilla frutescens causes acute bovine pulmonary emphysema and edema in cattle.

https://zukureview.com/zuku-qod/navle/1935?chosen=qL5FVi9IipslWn/VS9F0poViWRjrmzFawdvQPBWmzHQ%3D&utm_source=Zukureview+Subscribers&utm_campaign=c9b48097ca-EMAIL_CAMPAIGN_2024_08_06_01_53&utm_medium=email&utm_term=0_1c9568dbdc-c9b48097ca-%5BLIST_EMAIL_ID%5D

81
Q

An emaciated cow is presented. She appears weak and anorexic, with enlarged superficial cervical lymph nodes.
Value
Normal
T=102.9ºF (39.5°C) 100.4?102.8°F, 38.0?39.3°C HR=60 bpm
48-84 bpm
BR=30 brpm
26-50 brpm
The cow dies the next day. On necropsy, the following lesions in the lungs are evident.
What is the diagnosis?

A - Johne’s disease
B - Contagious bovine pleuropneumonia
C - Bovine tuberculosis
D - Bovine lymphoma
E - Actinobacillosis

A

Answer: Bovine Tuberculosis

These are the classic granulomas and abscesses of bovine tuberculosis.

Most commonly caused by Mycobacterium bovis or
Mycobacterium tuberculosis in cattle.
TB is reportable.

Look for cows with chronic debilitation, emaciation, but without the telltale watery diarrhea of Johne’s disease.
It is transmitted via aerosol route.
Here is an excellent image set of
TB lesions in 3 animals.

Actinobacillosis causes wooden tongue in cows.

Overview of Tuberculosis in Animals

Definitions and Terminology

•	Tuberculosis (TB): A chronic, infectious disease caused by various species of the genus Mycobacterium.
•	Mycobacterium bovis (M. bovis): The primary causative agent of bovine tuberculosis.
•	Mycobacterium tuberculosis (M. tuberculosis): Causes tuberculosis in humans but can also infect animals.
•	Mycobacterium avium complex (MAC): Includes species causing tuberculosis-like diseases in birds and occasionally in other animals.

Causative Agents and Characteristics

•	Mycobacterium bovis: Acid-fast, rod-shaped bacterium, part of the Mycobacterium tuberculosis complex.
•	Transmission: Primarily through inhalation of aerosols from infected animals, ingestion of contaminated feed or water, or direct contact with lesions.

Pathophysiology

•	Infection Process:
•	Inhalation or ingestion leads to initial infection in the lungs or alimentary tract.
•	Macrophages engulf bacteria, but Mycobacterium species resist destruction, leading to granuloma formation.
•	Granulomas, or tubercles, are composed of macrophages, lymphocytes, and fibrous tissue, often with a necrotic center.
•	Dissemination: Bacteria can spread through lymphatic or hematogenous routes to other organs, leading to generalized tuberculosis.

Clinical Changes and Symptoms

•	Cattle:
•	Early Stages: Often asymptomatic.
•	Progressive Disease: Coughing, dyspnea, weight loss, decreased milk production, lymph node enlargement.
•	Other Species:
•	Similar respiratory and systemic signs; species-specific variations in symptom presentation.

Diagnosis

•	Tuberculin Skin Test (TST): Intradermal injection of purified protein derivative (PPD) tuberculin; swelling at the injection site indicates a positive reaction.
•	Interferon-Gamma Release Assay (IGRA): Measures the release of interferon-gamma by T-cells in response to Mycobacterium antigens.
•	Culture and PCR: Definitive diagnosis through isolation and identification of Mycobacterium species from tissue samples.
•	Radiography and Ultrasound: Imaging to detect granulomas in the lungs or other organs.

Management Strategies

1.	Cattle:
•	Test-and-Slaughter Programs: Regular testing and culling of positive animals to control and eradicate tuberculosis.
•	Biosecurity Measures: Preventing introduction of infected animals, controlling wildlife reservoirs, and ensuring proper sanitation.
2.	Wildlife:
•	Management Practices: Controlling wildlife populations, vaccination programs in some regions, and reducing interactions between wildlife and livestock.
3.	Pets (Dogs and Cats):
•	Diagnosis and Treatment: Similar diagnostic methods; long-term antibiotic therapy (e.g., combination of isoniazid, rifampin, and pyrazinamide) may be attempted but is often impractical.

Medications

•	Antibiotics:
•	Isoniazid: Inhibits mycolic acid synthesis, crucial for mycobacterial cell wall.
•	Rifampin: Inhibits RNA synthesis.
•	Pyrazinamide: Disrupts mycobacterial cell membrane metabolism.
•	Ethambutol: Inhibits arabinosyl transferases involved in cell wall synthesis.
•	Doses: Vary by species and specific case, often requiring combination therapy for extended periods (6-12 months or more).

Prevention

•	Vaccination: Limited use in animals due to variable efficacy and interference with diagnostic tests. Bacille Calmette-Guérin (BCG) vaccine is used in some wildlife management programs.
•	Biosecurity Measures: Quarantine and testing of new animals, minimizing contact between livestock and wildlife, maintaining hygiene and sanitation.

Summary for Veterinary Professionals

•	Tuberculosis is a significant zoonotic disease requiring vigilant control measures.
•	Diagnosis relies on skin tests, blood assays, culture, and molecular techniques.
•	Management includes test-and-slaughter programs, biosecurity, and, in some cases, antibiotic therapy.
•	Prevention focuses on vaccination in wildlife, biosecurity in livestock, and proper management practices.

https://zukureview.com/zuku-qod/navle/1987?chosen=dt9WMq21kThLlgTrycVgcA%3D%3D&utm_source=Zukureview+Subscribers&utm_campaign=59bc1682c4-EMAIL_CAMPAIGN_2024_09_02_01_51&utm_medium=email&utm_term=0_1c9568dbdc-59bc1682c4-%5BLIST_EMAIL_ID%5D

82
Q

An eight-year-old female spayed Boston terrier is presented with a solitary well-circumscribed mass two-cm in diameter on the left hip area, shown below. The owner reports that the mass occasionally appears inflamed, and that the dog has a history of allergies.
A fine needle aspirate of the mass stained with Diff-Quik is also shown below.
Based on the clinical presentation, cytological findings, and presumptive diagnosis, the owner opts for surgical resection.
What pre-operative treatment is indicated?

A - No pre-operative treatment is indicated
B - H1 histamine receptor antagonist; corticosteroids
C - Nonsteroidal anti-inflammatory analgesics; H2 histamine blocker
D - Bactericidal beta-lactam antibiotics; calcium channel blocker (vasodilator).
E- Adrenerair aannict: hronchodilatorc

A

Answer: ** B - H1 histamine receptor antagonist; corticosteroids**

This is a well-differentiated mast cell tumor (MCT), which should be treated with an H1 histamine receptor anatagonist (a.k.a. blocker - e.g., diphenhydramine) and a corticosteroid (e.g., prednisone) prior to surgery.

De before surgical excision: CBC, serum chemistry panel, urinalysis, and regional lymph node aspiration (regardless of node size). CBC changes can include eosinophilia, basophilia, and regenerative or nonregenerative anemia. If there is lymph node metastasis, tumor recurrence, or peritumoral edema/bruising also consider abdominal ultrasound +/- hepatic and splenic aspirates and bone marrow cytology.
Complete excision requires at least two-cm lateral margins and one fascial plane.

Mast cell tumors can occur in any age or breed. Predisposed breeds include Boston terrier, pug, Shar-pei, Staffordshire bull terrier, golden and Labrador retrievers, and Jack Russell terriers.

Treat pre-surgically to shrink the tumor, minimize degranulation, and block the effects of histamine and other vasoactive mediators.
Risk of histamine-induced gastric ulceration can be decreased with proton pump inhibitors (e.g., omeprazole) +/- H2 histamine blockers (e.g., famotidine).

1.	Lymphoid Tumors of the Skin: Includes extramedullary plasmacytomas and cutaneous lymphosarcoma, found mainly on the head and extremities, affecting primarily middle-aged to senior dogs. Treatments involve surgical excision, radiation, and chemotherapy.
2.	Cutaneous Mast Cell Tumors: Common in dogs, appearing as raised, nodular masses often on limbs, abdomen, and thorax. Diagnostic tools include fine-needle aspirates and biopsy. Treatments range from surgical excision with wide margins to chemotherapy and radiation, depending on the tumor grade.
3.	Tumors with Histiocytic Differentiation: Covers diseases like histiocytoma and malignant histiocytosis, mainly affecting younger dogs. These tumors often require surgical removal, with chemotherapy used for more aggressive forms.
4.	Transmissible Venereal Tumors: Contagious tumors primarily affecting the external genitalia of dogs and transmitted by contact. Treatment typically involves chemotherapy.
5.	Cutaneous Lymphoma: Occurs less frequently but is notable for its two forms, epitheliotropic (involving skin cells directly) and nonepitheliotropic. Treatment options vary widely from topical therapies to systemic chemotherapy.

For a more comprehensive understanding, ensure to review the detailed descriptions, symptoms, diagnostic criteria, and therapeutic approaches provided on the Merck Veterinary Manual website. This will be essential for your BCSE test preparation.

83
Q

The night after Valentine’s day, a male neutered Schnauzer is presented with a combination of hyperactivity, urinary accidents, and diarrhea.
On physical exam there is muscle rigidity and chocolate wrappers visible in the diarrhea. The dog has a seizure on the exam table. An ECG is shown below.
Which one of the following choices is the best treatment plan?

0101:42
Value
Normal
T=103.2ºF (39.6°C) 99.5-102.5ºF (37.2-39.2°C)
HR=184 bpm
60-120
RR=40 brpm
15-34
A - Apomorphine,-gastric lavage, Na-bicarbonate IV
B - Ipecac, theophylline, digoxin
C - Phenobarbital, glycopyrrolate
D - Activated charcoal, sodium sulfate, methocarbamol IV
E - Diazepam, lidocaine drip

A

Answer: ** E - Diazepam, lidocaine drip**

Treat the ventricular tachycardia with a lidocaine drip and address hyperactivity/seizures with diazepam (Valium ®) or methocarbamol. If response to diazepam inadequate, consider barbiturates like phenobarbital (given slowly).

This is a classic presentation of severe chocolate toxicity, due to toxic methylxanthine alkaloids (e.g., theobromine, theophylline, caffeine).

Common around Halloween, Christmas, and Valentine’s Day holidays, see excitement, seizures, and arrhythmias. This dog’s ECG shows ventricular tachycardia (V-tach).
Most chocolate toxicities are milder - hyperactivity, vomiting, urinating, diarrhea.

Decontamination with emesis (induced with apomorphine) can be helpful up to eight hours after exposure in asymptomatic dogs.

If animal is sedated because of seizures, consider gastric lavage. If vomiting is controlled, give activated charcoal (AC) only if the dog ate a lethal dose. Chocolate will increase the risk of hypernatremia from AC, so the benefit must outweigh the risk. If AC is given, monitor for hypernatremia and associated signs (tremor, ataxia, seizures).

Tx arrhythmias depending on type
- usually a tachycardia;
give LIDOCAINE for V-tach or
BETA-BLOCKERS (propranolol/esmolol) for supraventricular tachycardias.

Chocolate toxicity from most toxic to least toxic: cacao beans and baking chocolate are worse than semisweet chocolate which is worse than milk chocolate. 49 grams of baking chocolate (one 2-oz bar) can kill a 7-kg dog. It would take 420 grams of milk chocolate (about eight 2-oz bars) bars to kill a 7-kg dog.

https://zukureview.com/zuku-qod/navle/1763?chosen=BLwcgF0L/VtuOP3VJnlEx7arFJLD6Zpt7mCgrVuDH0lbZIADKgpUVq2Twv2KGS6p&utm_source=Zukureview+Subscribers&utm_campaign=751612a1fe-EMAIL_CAMPAIGN_2024_05_20_01_51&utm_medium=email&utm_term=0_1c9568dbdc-751612a1fe-%5BLIST_EMAIL_ID%5D

84
Q

A two-year-old beagle is presented with patchy alopecia, papules, and thick crusts, on the ventral abdomen, chest, ears, and elbows.
The owner reports intense pruritus and a positive pinnal-pedal reflex is observed.
A flea comb does not reveal any fleas.
The image below shows what was discovered from a skin scraping.
Which one of the following organisms is depicted in the image?

A - Neotrombicula autumnalis
B - Cheyletiella yasguri
C - Demodex canis
D - Sarcoptes scabei
E - Straelensia cynotis

A

Answer: ** D - Sarcoptes scabei**

This is Sarcoptes scabei, a highly contagious mite.

A positive pinnal-pedal reflex (scratching action with the dog’s hind limb when the examiner firmly rubs the edge of the dog’s pinna between thumb and forefinger) is seen in 82% of dogs with sarcoptic mange but observed in only 7% of dogs with pruritus caused by other diseases.
Cheyletiellosis (walking dandruff) is another highly contagious mite.

Unlike Sarcoptes, they have very prominent mouthparts.

Image courtesy, Dr Kalumet.

https://zukureview.com/zuku-qod/navle/1759?chosen=UeXGnk5Xz9YqakYaok3orEv8lAWqVzfppkd6r2Doan4%3D&utm_source=Zukureview+Subscribers&utm_campaign=5c0ddd5c3a-EMAIL_CAMPAIGN_2024_05_14_01_51&utm_medium=email&utm_term=0_1c9568dbdc-5c0ddd5c3a-%5BLIST_EMAIL_ID%5D

85
Q

A four-month-old filly is presented with a four-day history of lethargy, heavy breathing, cough and decreased appetite.
Value
Normal
104.9°F (40.5°C) 99.0-100.6ºF (37.2-38.1°C)
HR=72 bpm
28-40 bpm
R=44 brpm
10-14 brpm
On lung auscultation asymmetrically-distributed crackles and wheezes are audible. Some areas have no breath sounds and a dull resonance on thoracic percussion. A lateral chest radiograph shows consolidated nodular lung lesions and mediastinal lymphadenopathy.
Pneumonia due to Rhodococcus equi infection is confirmed based on culture and polymerase chain reaction results on a transtracheal wash.
Which one of following choices is the most appropriate treatment?

A - Procaine penicillin G and gentamicin
B - Chloramphenicol and aminophylline
C - Clarithromycin and rifampin
D - Vancomycin and atropine (if bronchospasm is observed).
E - Tilmicosin

A

Answer: ** C - Clarithromycin and rifampin**

Start this filly on clarithromycin and rifampin.
In this case, a chest radiograph with consolidated nodular lung lesions and mediastinal lymphadenopathy is highly suggestive of Rhodococcus equi in a foal under five months of age.

Polyarthritis, polysynovitis, and diarrhea can also be seen in affected foals

Combination treatment of rifampin combined with either erythromycin, clarithromycin, or azithromycin is appropriate to treat R. equi pneumonia.

However, in one retrospective study, the combination of clarithromycin and rifampin was shown to be the most effective of
the three.

Tilmicosin, while a macrolide, is not very active against R. equi and swelling can occur at injection sites in foals; it is not
recommended for treatment of R. equi pneumonia.

Click here to see an ultrasound video from Merck of a three-month-old foal with rhodococcal pneumonia.

https://zukureview.com/zuku-qod/navle/1747?chosen=5XIg2ba95xnJGKRtSCb2n5mID4UOUaFGSi9qpaeTY9axFPrpBhXhbMPtRy3e28K4&utm_source=Zukureview+Subscribers&utm_campaign=5cd7044d13-EMAIL_CAMPAIGN_2024_05_08_01_51&utm_medium=email&utm_term=0_1c9568dbdc-5cd7044d13-%5BLIST_EMAIL_ID%5D

86
Q

Which species has ellipsoid erythrocytes that lack central pallor?

A - Chinchillas
B - Psittacines
C - Ferrets
D - Reptiles
E - Camelids

A

Answer: Camelids

Camelids (llamas, camels, alpacas) normally have ellipsoid red blood cells (RBCs, erythrocytes), which lack central pallor.
Birds and reptiles have nucleated, elliptical RBCs.

It is important to know what normal looks like for the main veterinary species.

https://zukureview.com/zuku-qod/navle/1744?chosen=USjLsVBVjd2i7MmRvQPNWA%3D%3D&utm_source=Zukureview+Subscribers&utm_campaign=5db7dbd7a6-EMAIL_CAMPAIGN_2024_05_07_01_51&utm_medium=email&utm_term=0_1c9568dbdc-5db7dbd7a6-%5BLIST_EMAIL_ID%5D

87
Q

For the last ten years, your state has had a mandatory vaccination program against
“pedunculated giblet disease” in fur-bearing turtles and the prevalence of this terrible disease has decreased markedly.
How does this decrease in prevalence affect the predictive value positive (PVP) of the best serologic test for pedunculated giblet disease?

A - PVP decreases as prevalence decreases
B - PVP depends on the number tested, not prevalence
C - PVP is affected by specificity, not prevalence
D - PVP increases as prevalence decreases
E - PVP stays the same as prevalence decreases

A

Answer: ** A - PVP decreases as prevalence decreases**

Don’t overthink this: As prevalence of a disease goes DOWN, PVP of your serologic test also goes DOWN.

That is, as your disease becomes more and more rare, the predictive value of your same old test gets WORSE.

That is all you need to know. Read more only if you want to see the math (but you don’t need it).

Let’s say prevalence of pedunculated giblet disease is 30% in 1000 turtles (that’s 300 infected, 700 disease-free, then).

A test with 90% sensitivity would correctly Dx 270 (cell “a”) with the disease (true pos) and correctly say 30 were negative (cell
“c”). A test with 90% specificity would correctly Dx 630 (cell “d”) as disease-free (true negs) and correctly say 70 were positive (cell
“b”): PVP=a/(a+b)=270/(270+70)=
79%.

Follow this link to See a PVP
Diagram.

Now, do the same math on 1000 turtles, but assume prevalence is now only ~1% after your vaccination program. You will see
PVP goes down to ~ 1%.

https://zukureview.com/zuku-qod/navle/1733?chosen=vECc2%2B8mvtDqTQSX81WWldWFRQc9vPaSbibsNibwe%2B51KzrHLEXGIHGD8T45ZyW3&utm_source=Zukureview+Subscribers&utm_campaign=40cb2d9fbc-EMAIL_CAMPAIGN_2024_05_06_01_51&utm_medium=email&utm_term=0_1c9568dbdc-40cb2d9fbc-%5BLIST_EMAIL_ID%5D

88
Q

Under what conditions is a very sensitive test used?

A - Lethal disease; highly_prevalent disease
B - Zoonoses; untreatable diseases
C - Rare disease; early diagnosis improves prognosis
D - Common disease; infectious diseases
E - Treatment does not affect prognosis; non-infectious diseases

A

Answer: C - Rare disease; early diagnosis improves prognosis

You need a very sensitive test if:
1. Disease is rare (e.g.: BSE)
2. Early Dx improves prognosis (e.g.: HIV in people)
3. The disease is highly lethal or consequences of missing a case are severe (e.g.: rabies, brucellosis, BSE, screw-worm, FMD, EIA)
Remember that a HIGHLY SENSITIVE test will have very FEW false negatives.

That means if a test is highly sensitive, you can TRUST a NEGATIVE TEST.
This sounds contradictory, but it makes more sense if you review this sensitivity diagram.
Sensitivity=a/(a+c). “a” are true positives. “c” are false negatives.

If sensitivity is HIGH then “c”
(FALSE negs) must be small.

Therefore, high sensitivity means you can really trust a NEGATIVE result to be correct.

https://zukureview.com/zuku-qod/navle/1732?chosen=Pp0VYwRosYKkjqbIcmTrJXvPwWTnxLwypeuHF6dIYeUnxxg6Roe8EzXOBdleIRD5&utm_source=Zukureview+Subscribers&utm_campaign=4c5b321d0f-EMAIL_CAMPAIGN_2024_05_03_01_51&utm_medium=email&utm_term=0_1c9568dbdc-4c5b321d0f-%5BLIST_EMAIL_ID%5D

89
Q

A rescued Pekin duck is presented for non-weight bearing lameness of the left pelvic limb and fever.

Which one of the following choices is the most likely diagnosis?

A - Bumblefoot
B - Egg binding
C - Panosteitis
D - Normal radiographs
E - Septic arthritis

A

Answer: E - Septic arthritis

The duck has septic arthritis.

There is diffuse soft tissue swelling of the left pelvic limb.

There also appears to be intracapsular soft tissue swelling of the left tibiotarsal joint causing the joint space to widen asymmetrically. An osseous fragment is noted at the medial aspect of the joint.

The articular margins of the tibiotarsus and the tarsometatarsus appear lytic and irregular.
Egg-binding signs are nonspecific but lameness is not a key finding typically. Sometimes the bird will adopt a penguin-like posture. Egg-binding is more often seen during the spring and summer.

https://www.merckvetmanual.com/poultry/disorders-of-the-skeletal-system-in-poultry/infectious-skeletal-disorders-in-poultry#v14343876

https://zukureview.com/zuku-qod/navle/2004?chosen=UVZCTJY8E3tIZbVbiOBdsA%3D%3D&utm_source=Zukureview+Subscribers&utm_campaign=6589dba74f-EMAIL_CAMPAIGN_2024_09_03_01_51&utm_medium=email&utm_term=0_1c9568dbdc-6589dba74f-%5BLIST_EMAIL_ID%5D

90
Q

An eight-month-old male intact German shepherd dog is presented for poor weight gain, chronic diarrhea, and polyphagia.

Fasting serum trypsin-like immunoreactivity (TLI) neasurement is <2.0 ugL (Normal = 5.7-45.2 kg/L).

What is the most likely cause of this dog’s clinical signs?

A - Inflammatory bowel disease
B - Acinar cell atrophy
C - Pituitary dwarfism
D - Acute pancreatitis
E - Small intestinal bacterial overgrowth

A

Answer: Acinar cell atrophy

This dog has exocrine pancreatic insufficiency (EPI) due to pancreatic acinar atrophy (PAA).
PAA is the most common cause of EPl in German shepherd dogs and is considered hereditary. It is also seen in rough-coated Collies and Eurasians.

In cats and older dogs EPl is most commonly caused by chronic pancreatitis.

Serum TLI is the best test for EPI in dogs and cats. A significantly low (<2.5 mg/L) fasting TLI is diagnostic for EPI.

In EPI there is impaired synthesis and release of digestive enzymes by pancreatic acinar cells, resulting in maldigestion, weight loss/failure to gain weight, diarrhea, and polyphagia.

Concurrent small intestinal dysbiosis and cobalamin deficiency are common.

Treat with oral enzyme replacement and cobalamin supplementation if deficient.
Patients with EPI require life-long enzyme replacement. Prognosis is good with appropriate treatment.

https://vetmed.tamu.edu/gilab/service/assays/tli/

https://zukureview.com/zuku-qod/navle/1731?chosen=8Z9%2BxwvLaX1TNXtc4whNRK4pdGe1VTXJ9z/9duXMj6w%3D&utm_source=Zukureview+Subscribers&utm_campaign=f9ddd25514-EMAIL_CAMPAIGN_2024_05_02_01_51&utm_medium=email&utm_term=0_1c9568dbdc-f9ddd25514-%5BLIST_EMAIL_ID%5D

91
Q

Retained placenta and metritis can predispose to which one of the following secondary conditions in horses?

A - Contagious equine metritis
B - Laminitis
C - Postpartum dysgalactia syndrome
D - Colic
E - Cystic endometriosis

A

Answer: Laminitis

In horses, retained placenta and septic metritis can lead to acute laminitis.

There are four main types of laminitis:
- sepsis/SIRS-related (following septic metritis, pleuropneumonia, colitis, grain overload, etc.; in research they induce this with black walnut (Juglans nigra) or carbohydrate overload)
- endocrinopathic (horses with insulin dysregulation/equine metabolic syndrome or equine Cushing’s disease; also associated with lush spring grasses)
- traumatic (a.k.a. “road founder”)
- support-limb (e.g., severe lameness on one limb requiring over-dependency on opposite leg, which then develops laminitis)

Parturition in Horses

Overview

Parturition, or foaling, in horses is a critical process that requires careful monitoring and management to ensure the health and safety of both the mare and the foal. Understanding the stages of labor, recognizing signs of impending foaling, and being prepared to assist if complications arise are essential for successful outcomes.

Stages of Parturition

1.	Stage I: Preparation
•	Duration: 1-4 hours.
•	Signs:
•	Restlessness, sweating, frequent urination or defecation.
•	Mild colic-like symptoms, getting up and down frequently.
•	Tail switching and kicking at the abdomen.
•	Physiological Changes:
•	Cervical dilation.
•	Uterine contractions begin to position the foal for delivery.
•	End of Stage I: Rupture of the allantochorion (“water breaking”), releasing allantoic fluid.
2.	Stage II: Delivery of the Foal
•	Duration: 15-30 minutes (can be up to 60 minutes).
•	Signs:
•	Strong, forceful abdominal contractions.
•	Appearance of the amniotic sac at the vulva.
•	The foal’s forelimbs should be visible within the amniotic sac, followed by the head.
•	Normal Presentation: Front legs extended with one slightly ahead of the other, head resting on the forelimbs.
•	Progression:
•	Delivery of the shoulders, then the rest of the body.
•	End of Stage II: Complete expulsion of the foal.
3.	Stage III: Expulsion of the Placenta
•	Duration: 1-3 hours.
•	Signs:
•	Mild uterine contractions to expel the placenta.
•	Mare may show mild discomfort.
•	Normal Outcome: Entire placenta should be expelled intact.
•	Importance: Retained placenta can lead to serious complications, such as metritis or laminitis.

Monitoring and Assistance

1.	Pre-Foaling Preparation:
•	Environment: Clean, quiet, and safe foaling area.
•	Supplies: Clean towels, antiseptic solution, gloves, umbilical cord clamps, and emergency veterinary contact information.
2.	Observation:
•	Mare Behavior: Monitor for signs of impending foaling (waxing teats, relaxed pelvic ligaments, changes in behavior).
•	Stage I and II Monitoring: Continuous observation, especially as labor progresses to ensure timely intervention if necessary.
3.	Assistance During Foaling:
•	Intervention Indications: Prolonged labor, malpresentation, mare exhaustion, or signs of distress in the mare or foal.
•	Actions: Gently assist the foal’s passage if necessary, ensuring proper presentation and avoiding excessive force.
•	Emergency Situations: Contact a veterinarian immediately for dystocia (difficult birth), incorrect foal positioning, or failure of the foal to progress through the birth canal.

Post-Partum Care

1.	Mare:
•	Observation: Monitor for signs of complications such as retained placenta, hemorrhage, or colic.
•	Placenta Examination: Ensure the entire placenta is expelled and inspect for completeness.
•	Vital Signs: Check the mare’s temperature, heart rate, and respiratory rate.
2.	Foal:
•	Immediate Care: Clear the foal’s airways, dry the foal, and encourage bonding and nursing.
•	Vital Signs: Monitor for normal breathing, heart rate, and activity levels.
•	Umbilical Care: Disinfect the umbilical stump with antiseptic.
•	Colostrum Intake: Ensure the foal nurses within the first 1-2 hours to receive colostrum for passive immunity.

Common Complications

1.	Dystocia:
•	Definition: Difficult or abnormal labor due to malpresentation, oversized foal, or uterine inertia.
•	Management: Veterinary intervention for repositioning the foal or performing an emergency cesarean section.
2.	Retained Placenta:
•	Definition: Failure to expel the placenta within 3 hours post-foaling.
•	Management: Administration of oxytocin to stimulate uterine contractions, manual removal by a veterinarian if necessary, and antibiotic therapy to prevent infection.
3.	Post-Partum Hemorrhage:
•	Definition: Excessive bleeding following delivery.
•	Management: Immediate veterinary care to control bleeding and stabilize the mare.

Summary for Veterinary Professionals

•	Parturition in horses consists of three stages: preparation, delivery of the foal, and expulsion of the placenta.
•	Close monitoring of the mare during foaling is crucial for timely intervention if complications arise.
•	Post-partum care includes monitoring the mare for complications, ensuring the foal receives colostrum, and inspecting the placenta for completeness.
•	Common complications include dystocia, retained placenta, and post-partum hemorrhage, requiring prompt veterinary attention.

https://zukureview.com/zuku-qod/navle/1730?chosen=uTXBjaj2v89zO7U56tbwVXlUdpnkqGiHdjRr4/MT4d0%3D&utm_source=Zukureview+Subscribers&utm_campaign=613dee4e88-EMAIL_CAMPAIGN_2024_05_01_01_51&utm_medium=email&utm_term=0_1c9568dbdc-613dee4e88-%5BLIST_EMAIL_ID%5D

92
Q

A flock from a turkey farm is presented with a mysterious illness. Numerous dead birds are noted. Sick turkeys are listless, with drooping wings, unkempt feathers, yellow droppings. Some birds are emaciated. Necropsy shows a marked thickening of the cecal wall, cecal ulcerations, and yellowish, caseous exudate distending the cecal lumen (cecal core). A typical liver looks like the image below.

What is the diagnosis?

A - Avian spirochetosis
B - Necrotic enteritis
C - Histomoniasis
D - Coronaviral enteritis of turkeys
E - Hemorrhagic enteritis of turkeys

A

Answer: Histomoniasis

This is histomoniasis; the combination of characteristic “bull’s-eye” lesions on liver and cecal changes are pathognomonic.

Caused by the protozoa Histomonas meleagridis, and transmitted in eggs of cecal nematode Heterakis gallinarum.

Expect a presentation of depression/diarrhea.
Expect to see more sudden death with necrotic enteritis caused by Clostridium perfringens.

Follow this link to see the so-called
“Turkish towel” intestinal pseudomembrane of necrotic enteritis.

Signs of avian spirochetosis are highly variable, may be absent: see listlessness, shivering, increased thirst, green/yellow diarrhea with increased urates early on. Caused by a tick-borne Borrelia. Look for characteristic enlarged, mottled spleen with petechial hemorrhages, similar to Marble spleen disease of pheasants.
Expect depression, bloody droppings, substantial mortality with hemorrhagic enteritis of turkeys.

Expect diarrheal presentation with coronaviral enteritis of turkeys but NOT the characteristic cecal/liver lesions described on necropsy above.

https://zukureview.com/zuku-qod/navle/1729?chosen=Vq0o2/cB/zYhqJvz4kUDcjUJIyzUCQF7SPfNyEehyUc%3D&utm_source=Zukureview+Subscribers&utm_campaign=1654ef6ff1-EMAIL_CAMPAIGN_2024_04_30_01_51&utm_medium=email&utm_term=0_1c9568dbdc-1654ef6ff1-%5BLIST_EMAIL_ID%5D

93
Q

When a foal is being treated with erythromycin (for Rhodococcus equi, for example), the mare is at risk of developing enterocolitis due to which one of the following organisms?

A - Clostridioides difficile (formerly _Clostridium difficile).
B - Clostridium novyi
C - Rhodococcus equi
D - Escherichia coli
E - Lawsonia intracellularis (a.k.a. proliferative enteropathy).

A

Answer: A - Clostridioides difficile (formerly _Clostridium difficile).

A history of recent antimicrobial therapy is common in cases of Clostridioides difficile (formerly Clostridium difficile associated diarrhea.

Adult horses exposed to erythromycin are particularly at risk for C. difficile enterocolitis.
Mares are very close with their foals and thus, if the foal is receiving a medication, the mare is likely ingesting some as well.

Clostridium novyi is the cause of infectious necrotic hepatitis, which is primarily seen in sheep but can also be seen in cattle, hogs, and horses.

Lawsonia intracellularis is the cause of proliferative enteropathy, resulting in diarrhea and hypoproteinemia in foals and swine.

Rhodococcus equi is a notable cause of pneumonia in older foals characterized by pulmonary abscessation as well as some extrapulmonary manifestations.

Escherichia coli can be a cause of septicemia and diarrhea in foals and calves.

https://zukureview.com/zuku-qod/navle/1718?chosen=iwXtlldTbc98CQrBM563LpYyy9ovLECYXQaGqFI4dpQentT61eAr%2BI/vVAPVJKcKhIWMof8hW3iiV8NiFV7QLA%3D%3D&utm_source=Zukureview+Subscribers&utm_campaign=d80584f334-EMAIL_CAMPAIGN_2024_04_29_01_51&utm_medium=email&utm_term=0_1c9568dbdc-d80584f334-%5BLIST_EMAIL_ID%5D

94
Q

Damage to which tissues can cause increases in serum alanine aminotransferase (ALT) levels in dogs and cats?

A - Pancreas and intestine
B - Spleen and adrenals
C - Liver and muscle
D - Heart and kidneys
E - Red blood cells and brain

A

Answer: C - Liver and muscle

In dogs and cats, damage to liver and muscle cells may cause increases in serum alanine aminotransferase (ALT).

ALT is a “leakage” enzyme meaning that the enzyme is intracellular. It leaks out of cells and ultimately into the circulation following damage to the cell membrane.

In other species (e.g., horses, ruminants, pigs, and birds), ALT is less useful because there are not high intracellular levels. Aspartate aminotransferase (AST), sorbitol dehydrogenase (SDH), and glutamate dehydrogenase (GLDH)
are the predominant leakage enzyme in these animals (availability is lab-dependent).

The magnitude of increase of leakage enzymes may be deceiving. Severe damage to a healthy liver may result in very high levels, while low levels may be seen when significant atrophy or fibrosis of the liver is present and few cells are left.

Differentiate ALT from muscle vs. liver by evaluation of the clinical picture and other serum biochemistry results. Other muscle enzymes include creatine phosphokinase (CK) and AST; and other liver enzymes are AST, SDH, GLDH, gamma glutamyl transferase (GGT), and alkaline phosphatase (ALP).

Leakage enzymes do NOT provide an estimate of liver function. This is evaluated by measurement of substances dependent on the efficacy of liver function such as bile acids, blood urea nitrogen, albumin, and glucose.

https://zukureview.com/zuku-qod/navle/1717?chosen=HdsMGv9Um0JThs9LcshD1hdiuELqCe6sDDeU1e4t5%2B0%3D&utm_source=Zukureview+Subscribers&utm_campaign=1b87d1b03e-EMAIL_CAMPAIGN_2024_04_26_01_51&utm_medium=email&utm_term=0_1c9568dbdc-1b87d1b03e-%5BLIST_EMAIL_ID%5D

95
Q

A stray dog is presented after being hit by a car.
The dog has hypotonic forelimbs and spastic paresis in the hinds. All four limbs have proprioceptive deficits and sensation loss-signs are worse in the forelimbs.

Where is the lesion?

A - Cervicothoracic: C6-T2
B - Cranial cervical: C1-C5
C - Cannot say without cutaneous trunci reflex results
D - Thoracolumbar T3-L3
E - Lumbosacral L4-S3

A

Answer: A - Cervicothoracic: C6-T2

Weak, hypotonic (i.e., lower motor neuron a.k.a. “LMN”) forelimbs and spastic paresis (i.e., upper motor neuron a.k.a. “UMN”) hindlimbs are signs of a cervicothoracic (C6-T2) lesion.

May see worse signs in forelimbs than hindlimbs.

NOTE: This is the OPPOSITE presentation as Schiff-Sherrington syndrome - i.e., SEVERE acute spinal cord trauma in the region of
T3-L3 and, when in lateral recumbency, the thoracic limbs are rigid and extended and the pelvic limbs appear flaccid in comparison.

However, pelvic limb reflexes are normal to increased, as would be expected with an upper motor neuron lesion.

Can localize T3-L3 lesions by checking cutaneous trunci reflex - the lesion is usually 1-2 vertebrae cranial to where the reflex disappears.

With C1-C5 would expect UMN signs in all four limbs, usually worse in hinds.

With a T3-L3, would see UMN hind limb signs and normal forelimbs.

https://zukureview.com/zuku-qod/navle/1716?chosen=vYs5tF55m5fsVtjvqgWMctlbI/Gnw5M7IvUr2UDu86k%3D&utm_source=Zukureview+Subscribers&utm_campaign=ea8cc60593-EMAIL_CAMPAIGN_2024_04_25_01_52&utm_medium=email&utm_term=0_1c9568dbdc-ea8cc60593-%5BLIST_EMAIL_ID%5D

96
Q

One description of a typical heart sound is “lub-dub.” What makes the first heart sound (S1) (i.e., the “lub’”)?

A - Aortic and pulmonic valves shut and the mitral and tricuspid valves open
B - Atrial contraction
C - Closure of atrioventricular valves, opening of semilunar valves
D - End of rapid ventricular filling_ period
E - Rapid atrial and auricular expansion due to cranial and caudal vena cavae contraction

A

Answer: C - Closure of atrioventricular valves, opening of semilunar valves

The first heart sound (S1 or the “lub” in “lub-dub”) is caused by closure of the atrioventricular valves (AV valves, mitral and tricuspid) and opening of the pulmonic and aortic valves (semilunar valves). This is the end of diastole and start of systole.

The second sound (S2) is the closure of the aortic and pulmonic valves and opening of the AV valves. This is the end of systole and start of diastole.

A third sound (S3) is the end of rapid ventricular filling and a fourth sound (S4) is atrial systole (atrial contraction).

You can often hear all 4 sounds in horses, but typically hear only S1 and S2 in cattle and small animals.

https://zukureview.com/zuku-qod/navle/1715?chosen=8H%2BIQzgOqJ3Ok9K%2BKKhg/lA6Koa9E3ze9cihxcE7hIM6ALll9XmAdiG/kSORx6Cai9U7LH4P%2B3uUlAaK4R/to2WF%2BK9A1ic3hu%2BOyvEfqPM%3D&utm_source=Zukureview+Subscribers&utm_campaign=4ad426cd19-EMAIL_CAMPAIGN_2024_04_24_01_52&utm_medium=email&utm_term=0_1c9568dbdc-4ad426cd19-%5BLIST_EMAIL_ID%5D

97
Q

A five-year-old female spayed Cocker Spaniel is presented with a drooping right ear and lip and drooling.

There is ptosis OD (right eye) but the remainder of the physical exam is unremarkable. She is otherwise bright, alert, and responsive.

What anatomic structure is damaged?

A - Facial nerve
B - Trigeminal nerve
C - Left side inner ear
D - Right side cranial cervical ganglion
E - Left side medulla, motor tract

A

Answer: Facial Nerve

Think of facial nerve paralysis (C7) with a unilaterally droopy face. Remember the facial nerve is motor to the muscles of facial expression (causing drooped ear, lip, and eyelid) and innervates the lacrimal and salivary glands.
Loss of lacrimal innervation can lead to dry eye and exposure keratitis if dog loses ability to close eyelid from damage to facial nerve innervation of the orbicularis oculi
muscle.

Idiopathic in 75% of canine cases (25% of cats).

Can also see these signs with middle ear damage (from otitis media), facial nerve trauma (ear surgery in dogs, or pressure from halter buckles in anesthetized horse), or neoplasia.

Follow this link to see a horse with facial nerve damage: note the nose pulled to horse’s left. (means facial nerve damaged on right).

Think more of a dropped jaw with trigeminal neuropathy (CN 5-dogs, horses).

https://zukureview.com/zuku-qod/navle/1714?chosen=%2BDjN/AlCLNJhEzHhEYZL8g%3D%3D&utm_source=Zukureview+Subscribers&utm_campaign=f65c309e3b-EMAIL_CAMPAIGN_2024_04_23_01_52&utm_medium=email&utm_term=0_1c9568dbdc-f65c309e3b-%5BLIST_EMAIL_ID%5D

98
Q

A one-year-old female spayed German Shepherd is presented for a two-week history of lameness.

Although the dog limps on the right foreleg as it enters the exam room, the owner reports that the dog was lame on the left hindleg last week.
On physical exam, the dog reacts painfully on palpation of the long bones of the right foreleg.

What is the most likely diagnosis?

A - Osteosarcoma
B - Hypertrophic osteodystrophy
C - Panosteitis
D - Multiple cartilaginous exostoses
E - Hypertrophic osteopathy

A

Answer: Panosteitis

This is panosteitis, an acute-onset shifting leg lameness with long bone pain most commonly seen in young (5-18 mos) medium to large breed dogs. German Shepherds are at the highest risk of developing panosteitis.

Hypertrophic osteodystrophy
(HOD) is a an uncommon
developmental orthopedic disease of young (2-8 mos), growing dogs.

Large and giant breeds, in particular the great Dane, are predisposed. It is characterized by symmetrical distal metaphyseal pain and swelling of long bones (most commonly the ulna and tibia), fever, anorexia, and depression.

https://zukureview.com/zuku-qod/navle/1709?chosen=LTO%2B3G5b006tyuXbvHfv6u7vQy19G12tP3nnjyivWaM%3D&utm_source=Zukureview+Subscribers&utm_campaign=1338f90fa8-EMAIL_CAMPAIGN_2024_04_22_01_52&utm_medium=email&utm_term=0_1c9568dbdc-1338f90fa8-%5BLIST_EMAIL_ID%5D

99
Q

A commercial pig farm experiences an explosive outbreak of respiratory disease with high mortality, primarily in young pigs under six months of age.

Affected pigs show severe respiratory distress, fever up to 107ºF (41.5°C), anorexia, and reluctance to move. Some animals display open-mouth breathing with a blood-stained, frothy nasal and oral discharge.

On necropsy, the lungs are bilaterally dark and swollen and ooze bloody fluid from the cut surface, as shown in the image below.

Which one of the following choices is the most likely diagnosis?

A - Mycoplasma hyopneumoniae
B - Actinobacillus pleuropneumoniae
C - Haemophilus parahaemolyticus
D - Swine influenza
E - Fusobacterium necrophorum

A

Answer: B - Actinobacillus pleuropneumoniae

This is a classic scenario of an outbreak of pleuropneumonia, caused by Actinobacillus pleuropneumoniae.

Primarily a growing pig disease, although adults can be affected as well, and sows can abort.
Mortality is high if untreated.

Survivors may experience lowered growth rates and have a persistent cough.

Image courtesy of Meghann Pierdon, VMD, DACAW.

https://zukureview.com/zuku-qod/navle/1708?chosen=RavNZiX4hv9VzibSd1ZIykb3fklGd3Tr5TgJbzMKX4w%3D&utm_source=Zukureview+Subscribers&utm_campaign=b0fbb1a3e0-EMAIL_CAMPAIGN_2024_04_19_01_52&utm_medium=email&utm_term=0_1c9568dbdc-b0fbb1a3e0-%5BLIST_EMAIL_ID%5D

100
Q

A six-year-old male neutered Yorkshire terrier is presented for routine examination. The owner has no current concerns.
Exam reveals a grade 3/6 left-sided systolic heart murmur. Pulse quality is strong and synchronous and there are no other abnormalities.

What is the most likely cause of the heart murmur?

A - Physiologic
B - Patent ductus arteriosus
C - Carnitine-deficient dilated cardiomyopathy
D - Aortic stenosis
E - Myxomatous mitral valve disease

A

Answer: Myxomatous mitral valve disease

Myxomatous mitral valve disease (MMVD) is the most common heart disease in dogs. It is the most common cause of a heart murmur in adult small-to medium-sized dogs, especially Cavalier King Charles spaniels, dachshunds, Yorkies, Chihuahuas, and mini poodles.

Myxomatous degeneration of the tricuspid valve can also occur.
Stage dogs with MMVD to determine the need for
intervention - use thoracic radiographs, +/- echocardiogram, +/- ECG. Asymptomatic dogs with a heart murmur and normal thoracic radiographs are
considered class B1. Do not treat class B1 dogs, but monitor for progression. Progression can be slow to negligible.

There is no Tx that slows or reverses MMVD; however, dogs with cardiomegaly (i.e., class B2) benefit from Tx with pimobendan to slow progression to congestive heart failure (CHF). If CHF is present, Tx to alleviate clinical signs and improve quality of life.

More invasive procedures, such as mitral valve annuloplasty, transcatheter edge-to-edge repair, and chordae tendineae replacement, have been undertaken in research settings and some client-owned dogs at a few facilities around the world.

The use of these procedures in veterinary medicine is still in its infancy, and as such, medical management of MVD remains the standard of care at this time.

Read the useful and informative
American College of Veterinary
Internal Medicine consensus guidelines for the diagnosis and treatment of mitral valve disease.

https://zukureview.com/zuku-qod/navle/1705?chosen=VILFHxx2QTtX0HGjmFFjUA%3D%3D&utm_source=Zukureview+Subscribers&utm_campaign=5b79cbf829-EMAIL_CAMPAIGN_2024_04_18_01_52&utm_medium=email&utm_term=0_1c9568dbdc-5b79cbf829-%5BLIST_EMAIL_ID%5D

101
Q

A six-year-old male neutered Yorkshire terrier is presented for routine examination. The owner has no current concerns.
Exam reveals a grade 3/6 left-sided systolic heart murmur. Pulse quality is strong and synchronous and there are no other abnormalities.

What is the most likely cause of the heart murmur?

A - Physiologic
B - Patent ductus arteriosus
C - Carnitine-deficient dilated cardiomyopathy
D - Aortic stenosis
E - Myxomatous mitral valve disease

A

Answer: Myxomatous mitral valve disease

Myxomatous mitral valve disease (MMVD) is the most common heart disease in dogs. It is the most common cause of a heart murmur in adult small-to medium-sized dogs, especially Cavalier King Charles spaniels, dachshunds, Yorkies, Chihuahuas, and mini poodles.

Myxomatous degeneration of the tricuspid valve can also occur.
Stage dogs with MMVD to determine the need for
intervention - use thoracic radiographs, +/- echocardiogram, +/- ECG. Asymptomatic dogs with a heart murmur and normal thoracic radiographs are
considered class B1. Do not treat class B1 dogs, but monitor for progression. Progression can be slow to negligible.

There is no Tx that slows or reverses MMVD; however, dogs with cardiomegaly (i.e., class B2) benefit from Tx with pimobendan to slow progression to congestive heart failure (CHF). If CHF is present, Tx to alleviate clinical signs and improve quality of life.

More invasive procedures, such as mitral valve annuloplasty, transcatheter edge-to-edge repair, and chordae tendineae replacement, have been undertaken in research settings and some client-owned dogs at a few facilities around the world.

The use of these procedures in veterinary medicine is still in its infancy, and as such, medical management of MVD remains the standard of care at this time.

Read the useful and informative
American College of Veterinary
Internal Medicine consensus guidelines for the diagnosis and treatment of mitral valve disease.

https://zukureview.com/zuku-qod/navle/1705?chosen=VILFHxx2QTtX0HGjmFFjUA%3D%3D&utm_source=Zukureview+Subscribers&utm_campaign=5b79cbf829-EMAIL_CAMPAIGN_2024_04_18_01_52&utm_medium=email&utm_term=0_1c9568dbdc-5b79cbf829-%5BLIST_EMAIL_ID%5D

102
Q

A one-year-old female llama is presented with hyperkeratosis around the nose and mouth.
What is the most likely diagnosis?

A - Dorsal nasal alopecia
B - Wry face
C - Dermatophilosis
D - Zinc-responsive dermatosis
E - Munge

A

Answer: Munge

This is the clinical picture of munge (idiopathic nasal/perioral hyperkeratotic dermatosis). Some affected animals have lesions on the bridge of the nose and around the eyes and ears.

Average age of onset is six months to two years old.

Tx - address secondary bacterial infections (e.g., daily 10% povidone iodine scrubs and apply 7% tincture of iodine). If minimal response, include a topical glucocorticoid or intralesional triamcinolone acetonide. If still no
response, evaluate immune function.

Dorsal nasal alopecia is characterized by alopecia over the bridge of the nose with normal to variably scaly, hyperpigmented, and thickened skin. It is more common in dark-haired animals. It can be secondary to fly bites or rubbing the nose.

Click here to see a llama with dorsal nasal alopecia.

Animals with idiopathic hyperkeratosis (zinc-responsive dermatosis) present with thickened, nonpruritic papules with tightly adherent crusts in the less densely haired areas of the perineum, ventral abdomen, inguinal region, medial thighs, and axilla. Signs can begin at any age.

https://zukureview.com/zuku-qod/navle/1702?chosen=wfOStFmgW9nO59xqKEV1833%2BR3qrkJdSjEMjbOtnJfc%3D&utm_source=Zukureview+Subscribers&utm_campaign=43456d44da-EMAIL_CAMPAIGN_2024_04_17_01_52&utm_medium=email&utm_term=0_1c9568dbdc-43456d44da-%5BLIST_EMAIL_ID%5D

103
Q

A female veterinarian is pregnant and her physician tests her serologically for toxoplasmosis.
IgM is negative.
IgG is positive.

What is the most appropriate interpretation?

A - Mother is safe, baby at risk
B - Both mother and baby are safe
C - Both mother and baby at risk
D - Mother at risk, baby is safe
E - Need to re-check in 2 weeks for rising titers

A

Answer: Both mother and baby are safe

Both mother and baby are safe.
The toxoplasmosis organism causes birth defects in a developing fetus if a mother is infected for the first time in her life while pregnant (ie: IgM positive while pregnant).

Toxoplasmosis is not generally dangerous to immune-competent people and a positive IgG result suggests a previous infection.
There are challenges to toxoplasmosis testing in pregnant women (false positives).

If a pregnant woman is IgM positive, confirmatory tests must be done.

Click here for more on pregnancy and toxoplasmosis.

https://zukureview.com/zuku-qod/navle/1693?chosen=880IhrDfWfwmivh/GPpo3Q9kdgXg86Zr7ZxFWq4pUeE%3D&utm_source=Zukureview+Subscribers&utm_campaign=b41fc05a7a-EMAIL_CAMPAIGN_2024_04_15_01_52&utm_medium=email&utm_term=0_1c9568dbdc-b41fc05a7a-%5BLIST_EMAIL_ID%5D

104
Q

During examination of a newborn male cria, adhesions between the free end of the penis and the prepuce are evident.

What is the most likely explanation?

A - Secondary to stress in utero, 5th month
B - Common congenital malformation of preputial ring, associated with phimosis
C - Normal finding.
D - Failure of separation of preputial diverticulum
E - Sequella of hypospadias; Heritable

A

Answer: Normal Finding

Normal male crias are born with adhesions between the free end of the penis and the prepuce. As they sexually mature these adhesions gradually detach, so puberty can be seen clinically when the penis can be completely exteriorized.

Castration prior to puberty may result in continued adhesions. In Peru, males are not generally used for breeding until 3 years of age.
Male llamas reach puberty at approximately 21 months (range 9-31 months). Alpacas mature earlier, around 12 months of age.

105
Q

A two-day-old calf has a GGT level of 500 times the adult upper reference range.
Which one of the following choices is the most likely interpretation?

A - Associated with neonatal cholestasis
B - Related to bone development
C - Caused by traumatic placental detachment
D - Suggestive of a congenital liver shunt
E - Due to colostrum absorption

A

Answer: Due to colostrum absorption

Very high GGT levels are seen with colostrum absorption.

In newborn calves, lambs, and pups, an increase in GGT up to a 1000 times is normal when they receive high levels of colostrum from the dam. In calves it takes four to six weeks for these GGT levels to return to the normal
adult reference range.

High levels of GGT are produced in the mammary epithelium during lactation. When evaluating bloodwork, make sure to utilize reference ranges specific for the age of the patient.

https://zukureview.com/zuku-qod/navle/1691?chosen=E5Q3%2BkptaJSR197I6v7Og7ObNqCo94K8KQL5ikadElCKk6VWaKjvM7w8HQjpEL2l&utm_source=Zukureview+Subscribers&utm_campaign=c8ffc18539-EMAIL_CAMPAIGN_2024_04_11_01_52&utm_medium=email&utm_term=0_1c9568dbdc-c8ffc18539-%5BLIST_EMAIL_ID%5D

106
Q

What recommendation should be given to the owner of a mare that has had Caslick’s vulvoplasty surgery?

A - Mare has a decreased risk for perineal laceration
B - Mare has a higher than normal risk for metritis
C - Mare should not be bred by a stallion
D - Higher risk of uterine torsion
E - Mare will need an episiotomy before foaling

A

Answer: Mare will need episiotomy before foaling

The mare will need an episiotomy (a surgically planned incision on the perineum and the posterior
vaginal wall) prior to foaling.

Sometimes an episiotomy may be needed prior to breeding by stallion if the remaining vulvar cleft is too small to permit intromission.

A Caslick operation (vulvoplasty, a superficial form of the episioplasty) is used to treat pneumovagina in horses to prevent genital infections and infertility.

https://www.merckvetmanual.com/management-and-nutrition/management-of-reproduction-horses/breeding-soundness-examination-of-the-mare

Click here to see a horse that has had a Caslick operation.

107
Q

When a sow is giving birth, what is the shortest time interval between passage of piglets that indicates dystocia?

A - 2 hours
B - 15 minutes
C - 4 hours
D - 1/2 hour
E - 1 hour

A

Answer: 1 hour

Dystocia in the sow is present when one hour or more has passed between piglets.

Intervention should be swift to prevent death of piglets from anoxia.

In sows, dystocia is often due to uterine inertia, similar to other animals that produce multiple offspring.

Other causes include fetal malposition, fetopelvic disproportion, obstruction of the pelvic canal, deviation of the uterus, and excitement in the dam.

Medical therapy can be given with oxytocin and/or calcium only when an obstructive dystocia is not present.

https://www.merckvetmanual.com/management-and-nutrition/management-of-reproduction-pigs/breeding-management-of-pigs?autoredirectid=16798

108
Q

A five-year-old female spayed cat is presented with a two-day history of vomiting and anorexia.
Physical exam reveals 5% dehydration and abdominal discomfort. Plain abdominal radiographs are shown below.
Which choice is the most appropriate step to take next?

A - IV fluids and broad-spectrum antibiotics
B - Gastroduodenoscopy with biopsy
C - Exploratory laparotomy_
D - Specific feline pancreatic-lipase assay
E - Non-iodinated contrast series

A

Answer: Exploratory Laparotomy

Exploratory laparotomy is the most appropriate next step for this linear foreign body. On the lateral radiographs, there is an area of the small intestine with very small gas bubbles where the walls of the small intestine appear undulating.
In this case, the linear foreign body was a bikini string.

A linear foreign body is a surgical emergency due to potential intestinal perforation and septic peritonitis. In severe cases, the
“sawing action” of the linear foreign body against the intestinal mucosa can cause extensive damage requiring extensive resection and anastomosis.

Occasionally, the intestinal damage is too extensive and euthanasia is the best course.
If a definitive Dx is not evident on radiographs, consider abdominal ultrasound to further evaluate the gastrointestinal tract.

Click here to see normal feline

https://zukureview.com/zuku-qod/navle/1672?chosen=nipbTi8rwOSqEhFTF2at/jmuPF9pm%2Bgy9P6ZJnvYP/g300lrUwJMiJsZjxcJ6Xla&utm_source=Zukureview+Subscribers&utm_campaign=e8a9336018-EMAIL_CAMPAIGN_2024_04_08_01_52&utm_medium=email&utm_term=0_1c9568dbdc-e8a9336018-%5BLIST_EMAIL_ID%5D

109
Q

Which diseases are associated with a high anion gap (AG)?

A - Prostatic adenocarcinoma, pleural effusion, lymphangiectasia
B - Hypercalcemia of malignancy, hypoadrenocorticism, displaced abomasum
C - Eosinophilic enteritis, hypothyroidism, gastrinoma
D - Grain overload, ethylene glycol toxicity, renal failure
E - Diabetes mellitus, pancreatitis, chronic diarrhea

A

Answer: D - Grain overload, ethylene glycol toxicity, renal failure

Think of an increased anion gap (AG) with renal failure, ethylene glycol toxicity, lactic acidosis from grain overload or strenuous exercise, and diabetic ketoacidosis.

The AG is an assessment of acid-base status and evaluates unmeasured anions such as ketones, lactate, uremic metabolites, and toxins (e.g., ethylene glycol).

Anion gap = (Na* + K*) - (HCO3 + CI)
If venous blood gas analysis is unavailable substitute TCO2 for HCO3

Here is more information from Cornell’s eClinPath website about measurement of the anion gap.

https://zukureview.com/zuku-qod/navle/1671?chosen=6T7pBWa3KzDoHUQmKskaeA8lytdDvhL%2BGEu7ycecGMLn7sj/%2BySuxcDXMrhsXNNFZm1xjC63PHDSjXYFjSHHJOnLsVbSKbX9In2IL8yOIY0%3D&utm_source=Zukureview+Subscribers&utm_campaign=6b810ea075-EMAIL_CAMPAIGN_2024_04_05_01_52&utm_medium=email&utm_term=0_1c9568dbdc-6b810ea075-%5BLIST_EMAIL_ID%5D

110
Q

What is the earliest stage post-breeding that an experienced practitioner can reliably rectally palpate fremitus in the uterine artery of the gravid uterine horn in a pregnant cow?

A - 60-75 days
B - 70-80 days
C - 90 -120 days
D - 120-150 days
E - 45-60 days

A

Answer: 120-150 days

An experienced practitioner can reliably feel fremitus (vibration in uterine artery of the gravid uterine horn) between 120 and 150 days.
Remember that fremitus in the uterine artery is a supportive sign of pregnancy but NOT conclusive evidence of pregnancy.

An experienced practitioner can reliably feel the membrane slip of chorioallantoic membranes rectally at 30-35 days and can feel placentomes at around 70-75 days.

Placentomes and membrane slip
ARE cardinal signs of pregnancy.

One way to remember these is that the EARLIEST occurs in
REVERSE alphabetical order ie:
Slip 30-35 days
Placentomes 70-75 days
Fremitus 120-150 days

https://zukureview.com/zuku-qod/navle/1670?chosen=Li8MEfMvRsyfP%2BKVqXOwqw%3D%3D&utm_source=Zukureview+Subscribers&utm_campaign=d2c0e8204d-EMAIL_CAMPAIGN_2024_04_04_01_52&utm_medium=email&utm_term=0_1c9568dbdc-d2c0e8204d-%5BLIST_EMAIL_ID%5D

111
Q

A five-year-old female spayed domestic short-haired cat is presented with chronic vomiting, anorexia, and weight loss.
Physical examination reveals icterus, dehydration, and fever. Abdominal palpation identifies thickened small intestinal loops, and serum biochemistry shows increased bilirubin and liver enzymes.
The cat is diagnosed with extrahepatic biliary obstruction with accompanying cholangitis and inflammatory bowel disease.

Which one of the following normal anatomic features may predispose cats to this triad of diseases?

A - Enlarged duodenal papilla in comparison to other species
B - Common opening of the pancreatic and common bile ducts into duodenum
C - Narrowing of the duodenum distal to the entry of the common bile and pancreatic ducts
D - Decreased proportion of alpha-smooth muscle actin in muscle fibers of the wall of the gall bladder in cats
E - Decreased diameter and increased branching of the hepatic ducts in comparison to other species

A

Answer: Common opening of the pancreatic and common bile ducts into duodenum

In cats, there is a common opening of the pancreatic and common bile ducts into the duodenum.

It is thought that this may predispose them to ascending cholangitis and pancreatitis after vomiting associated with inflammatory bowel disease, resulting in extrahepatic biliary obstruction.

https://www.merckvetmanual.com/digestive-system/hepatic-diseases-of-small-animals/extrahepatic-bile-duct-obstruction-in-small-animals?mredirectid=1156

Other possible etiologies of EHBO include neoplasia, biliary stricture, duodenal obstruction, diaphragmatic hernia, and parasitic infection.

In all cases, there is a lack of bile entry into the intestinal tract, leading to decreased absorption of fat and fat soluble vitamins such as
vitamin K, potentially resulting in coagulopathies.

Except in transient cases that are related to acute pancreatitis, need surgical intervention to relieve the obstruction as well as appropriate supportive care, antimicrobial treatment, and vitamin K administration.

Click here to see a good summary of extrahepatic biliary obstruction with images, courtesy of the American College of Veterinary Surgeons (ACVS).

https://www.acvs.org/small-animal/extrahepatic-biliary-tract-obstruction/

https://zukureview.com/zuku-qod/navle/1669?chosen=ESTUa%2BWkWQ9q/8R8E0RfErvQLpoWv/dhNAj/tuLFaEKKcrzGBvBF0FtaJL9443gzkXe8rVZAlO6dkJ0tZYelBw%3D%3D&utm_source=Zukureview+Subscribers&utm_campaign=179365ae71-EMAIL_CAMPAIGN_2024_04_03_01_52&utm_medium=email&utm_term=0_1c9568dbdc-179365ae71-%5BLIST_EMAIL_ID%5D

112
Q

A three-year-old white rat is presented with yellowing fur and a brownish, granular sebaceous secretion at the base of affected hair shafts.

What should the owner be told?

A - Normal age-related change
B - Common manifestation of chorioptic mange
C - Probable ringworm, resolves spontaneously with sunlight exposure
D - Staph dermatitis, responsive to antiseptic shampoo
E - Moisture-related dermatitis, cage needs more frequent cleaning.

A

Answer: A - Normal age-related change

These are normal age related changes. Rats normally live two to three years, though some may live to five years of age.

With age, the hair coat of white rats normally develop yellowing fur. They may also show a brownish, granular sebaceous secretion at the base of affected hair shafts which an owner may mistake for a skin parasite.

113
Q

A six-year-old male neutered Doberman pinscher is presented for a one-week history of coughing, depression, and exercise intolerance.
Exam reveals tachycardia with an irregularly irregular heart rhythm and pulse deficits.

Once the dog is stabilized which initial diagnostic step is most appropriate?

A - Abdominal focused assessment with sonography for triage,. trauma, and tracking rapid (AFAST) for hemoabdomen
B - CBC to identify anemia
C - Thoracic radiographs to evaluate for congestive heart failure
D - Ultrasound for pericardial effusion
E - Bronchoscopy for airway assessment

A

Answer: C - Thoracic radiographs to evaluate for congestive heart failure

This is a classic presentation for dilated cardiomyopathy (DCM); perform thoracic radiographs to evaluate for congestive heart failure. Perform electrocardiogram to better assess the cardiac rythm. Use echocardiography to confirm the diagnosis.

Consider veterinary point of care ultrasound (POCUS) to assess for pleural effusion and alveolar infiltrates.

Tx to control arrhythmia and diminish clinical signs. Therapy may include diuretics (e.g., furosemide), positive inotropes (e.g., pimobendan), angiotensin converting enzyme inhibitors (e.g., enalapril) and antiarrhythmics (e.g., diltiazem, sotalol).

Px is guarded, especially for Dobermans - with Tx may live six months to two years. Large and giant breeds are at increased risk, including Irish wolfhounds and great Danes. Boxers can develop arrhythmogenic right ventricular cardiomyopathy, distinct from DCM.

Dobermans have two genetic mutations that can lead to development of DCM: PDK4 and
DCM2. Each mutation independently can lead to DCM: dogs affected with both are 30 times more likely to develop DCM than a normal dog.

https://zukureview.com/zuku-qod/navle/1657?chosen=/aAohEOm/tj%2B8SkQ1bBK5W9gNbdZaeYSwy41OnRhFP4UfpWSdHkndGIAlthdIunFDGkMponqYgx1zyDmVy54y3t20nfLm%2BsTZcM6hD%2Bp%2BtkY7EoupLMie%2BJZWclPMT8bykHR80CLl3iF2VukGAzUXA%3D%3D&utm_source=Zukureview+Subscribers&utm_campaign=dd8bdf9f66-EMAIL_CAMPAIGN_2024_04_01_01_52&utm_medium=email&utm_term=0_1c9568dbdc-dd8bdf9f66-%5BLIST_EMAIL_ID%5D

114
Q

What behavior is the mare displaying in the image below?

A - Flehmen
B - Cribbing.
C - Lignophagia
D - Windsucking.
E - Aggression

A

Answer: Flehmen

This is flehmen. This is a behavior seen in the following circumstances: normal stallion response to pheromones, normal response of all horses to new smells (even foals do it!), sign of colic, or associated with granulosa cell tumors in mares.

Many other animals also show this response, for example here is a cat doing flehmen.
Image courtesy, Waugsberg.

https://zukureview.com/zuku-qod/navle/1656?chosen=R8MeipsSyLTwETdstETZYA%3D%3D&utm_source=Zukureview+Subscribers&utm_campaign=53142829a3-EMAIL_CAMPAIGN_2024_03_29_01_53&utm_medium=email&utm_term=0_1c9568dbdc-53142829a3-%5BLIST_EMAIL_ID%5D

115
Q

What behavior is the mare displaying in the image below?

A - Flehmen
B - Cribbing.
C - Lignophagia
D - Windsucking.
E - Aggression

A

Answer: Flehmen

This is flehmen. This is a behavior seen in the following circumstances: normal stallion response to pheromones, normal response of all horses to new smells (even foals do it!), sign of colic, or associated with granulosa cell tumors in mares.

Many other animals also show this response, for example here is a cat doing flehmen.
Image courtesy, Waugsberg.

https://zukureview.com/zuku-qod/navle/1656?chosen=R8MeipsSyLTwETdstETZYA%3D%3D&utm_source=Zukureview+Subscribers&utm_campaign=53142829a3-EMAIL_CAMPAIGN_2024_03_29_01_53&utm_medium=email&utm_term=0_1c9568dbdc-53142829a3-%5BLIST_EMAIL_ID%5D